You are on page 1of 294
MCAS TE Civil red fiteted ad lp 4 Reference VOLUME 2a Cory elitmered Cedar) to Past CE Board CU CCR Eee LAL: | and Transportation Engineering Hydraulics and Geotechnical Engineering Structural Engineering and Construction Civil Engineering Reference For Licensure Examinations Volume 2 Copyright © 2006 by Diego Inovencio Tapang Gillesania All rights reserved. No pant of this book may be reproduced, stored in a rewieval spstemt, or ‘transferred, in any form or by any ‘without the prior permission of the ISBN. 971-8614 Cover design by the author. Civil Engineering I Reference Vol. 2 Table of Contents CE Board May 2000. ‘Mathematics, Surveying, and Transportation Eogneeog at Solution... Hydraulics and Geotechnical Engineering Solution. Seructural Engineering and Construction... Solution. aa CE Board November 2000 Mathematics, Surveying, and Transportation Engineering. Sore Solution.. _ = — 38 Hydraulics and Geotechnical Engineering zoe era Solution, E 2 47 Structural Engincering and Conscructian 32 ‘Solution... fa “ Be Ss CE Board May 2001. fora Surveying, EE upron aes 63, Solution a eee ‘Hydraulics and Geotechnical Engineering. ae 76 ‘Solution. i ca i 80. ‘Structural Engineering and Construction — 87 ‘Solution... it eran 92 CE Board November 2001. seencieees 98 LO) at athena. Suneying and TranspertonEgetng ‘Solution, - = = re ‘Hydraulics and Geotechnical Engineering. aoe: ‘Solution. i ia ‘Structural Engncesng and ‘Construction vs ‘ mom ces nee 2S Solution. bee BL o Table of Comtents CE Board May 2002... 139 to 187 Mathematics, Surveying, and Transportation Enginecring.. 9 ee ‘anil Eng i Hydraulics and Geotechnic: ering... i a ae kl mee ae Structural Engineering and Construction = 169 Solution ei CE Board November 2002. ... 188 to 237 Matematis Sureing ad Transportation Engineering mmm ‘Solution... 192 Hydraulics and Geotechnical Engineering. a3 ‘Solution. intense OB ‘Structural Engineering and Construction - on 2B Solution. im - es aan 229 CE Board May 2003.. Mathematics, Surveying, and Transporation Engesring Solution... Hydraulics and Geotechnical Engineering. Solution ae aatass ‘Structural Engineering and Consemiction... aS 269, Solution. — = 280 CE Board November 2003. ‘Mathematics, Surveying, and Transportation gine 292 Solution. 293, Hydraulics and Geotechnical Engineering. 5 304 Solution ... ‘ ves omer 3b Seracrurl Enesco and Construction z got Solution... Civil Engineering = Reference Vol. 2 CE Board May 2004... 341 to 388 Mathematics, Surveying, and Transporration Enginceting.. mins aE Solution... Ses eees Hydraulics and Geotechnical Engince ti §..0..cnmener aS Solution aah Structural Engrering and Constr sion ata Solution a ‘CE Board November 2004 .. 389 to 436 Mathematics, Surveying, and Transportation EnginecTingvcnninsn 389 Solution... _ 394 Hydraulics and Geotechnical Engineering, ne AOS Solution.... 4 Structural Engineering and Construction Sepa nead Solution. 2 426 CE Board May 2005. 437 to 480 Mathematics, Surveying. and Transportation Engineering. 437 Solution... " - 42 “Hydraulics and Geotechnical Engineering : 453 Solution... = serene 5B ‘Structural Engineering and Construction “66 Solution........... = = es S74 CE Board November 2005... 481 to 526 ‘Mathematics, Surveying, and Transportation. ate é a1 Solution. 485 ‘Hydraulics and Geotechnical Eng 493. Solution. SOL ‘Structural Engineeri an Constr S10 ‘Solution... 57 Problem Index . Table of Comtemts = . Civil Engineering 1 Reference Vol. 2 ‘Seat Noz CIVIL ENGINEER Licensure Examination ‘Thursday, May 11,2000, 06:00 a.m. 01:00 p.m. SURVEYING & TRAN! x SETA ICTION: Select the correct answer for each of the following questions ansater for each item by shading the box corresponding to the letter of your choice on the answer sheet provided. STRICTLY NO FRASURES ALLOWED, Use pencil no. 2 only. MULTIPLE CHOICE ‘ Mark: 1. How long will take for money to quadruple iteelf if invested at 20% compounded quarterly? i ‘A. 10? years © 95 years B62 years D7 years 2 Find the area in 9, m. of a spherical triangle Of whose angles are 123%, 84", sand 75°, The radius of the sphere is 30m “A. 18633 19586 B 15708, D.1t02 3. Convert 405° to mils “A 2.800 mls G, 7200 mils E Di, 6,200 mils 4 ght circular cone of radius 4 em is 100.58 oq. cm Detstmine the slant height. ‘Bem © 6cm 3B Oem B idem 5 Five thousand pegs is dposied a the end of eachyyeur for 15 years uo an account earning 75% compounded continuously. Find the amount after 15 years, OK Piss 5413 © Pisg26s9 B ibz742 D,Pri2g541 6. There are four geometric means between 3 and 729. Find the fourth term, A 8L C28 tee Been ys Be 7. Loge975 =x. Find x Rs cs Baia D287 2 8. Evaluate [50 cose aa A. 15431 © c 1708 8 19.086 B wiz 8. A piece fired at an angle of 20- with she horizantal tthe top of a 0- High building. ‘The muzzle Geloity ie 300 m/s. What i the total Une of " A 3268 © 22s . B 37s D. i855 ‘ * z 0 . 2 a. Mw 5 17, 19. 2000 Mathematics, Surveying. 204. ‘Two vertices of a triangle are (6, -1) and (7-8) third vertex such that the centroid of the triangle ipd the ordinate af the ligat the origin. A 18 cB Ba D4 ‘A machine costing P480,000 has life expectancy of 12 years with a salvage Value of 10% of tit First cost. What isthe book value after five years using, the declinmg balance method? ‘A. P83, © P96 432 B Pisz738 D, P1785. ‘Two sides of a triangle measure 18 em and 6 cm. The third sce may be: ‘Az © 10 BB B11 "At what value of x will the lope of the curve? = 9x=y =Obe 18? a2 cs Ba D3 ‘The frustum of a regular tdangular pyrimid has equilateral triangles for its bases and has an altitude of 8 m. The 135 cus, whal isthe upper base edge? Alm Cim 85m Dm ‘A cylinder of radius 6 m hos its axis along the Yoaxia. A second eylinder af the’same radius has its axis along the Yeasis. Find the volume, in the first ‘octant, common to the two cylinders. wer base edge is Om. Ifthe volurne is em 14m ote Dam Raltonalize tse followings zoe Le is een ib ra A lighthouse i 2k la stright shore search gla he gthouse fectbes to a car moving along the shore. When the car is 1 km from the [point nearest to the lighthouse, the searchlight rotates 0.25 revfhour. Find the speed of the ear in kph. AN383 C22 B. 256 D, 387 ‘The total surface area of a closed cylindrical tank is 153.98 square meter. If the volume isto be maccanum, what is ils height in meters? ‘A. 68m C 36m =B 57m D. 45m ‘Determine the moment of inertinjabout the Yanus, of the area bounded by the curved =4y, the line x= 4 and the X-axis A. 985 © 1017 B 1324 D219 ‘Civil Engineering 3 Reference Vol. 2 20, From point A ona simple curve, the perpendicular distance to the tangent, at pase Ta The tangent paces Hough the PC The dstance from © to PC. is 260m. Find the length ofthe eurve from PC to A, ina ‘A 2037 © 208.56 B 35247 D, 2542 21 The ground makes uniorm sloge of 48% from STA 12-+ 18M to STA 12 + 240. “AUSTA 12 #180, the center height of Use roadway is 1.2 ma fll At the other station, the center height is 25 m cut. Find the length of cut in meters. A. 3085 C4632 3. 4054 D. 50.28 22. A vertical parnbolicsag curve has tangent grades of -1.2% and+0.6%, Ifthe ‘grade changes uniformly at 018% per 20 my fand the length of the curve. ‘A. 200m © 300m 8. 150m 250m 2. The observed interior angles of a angular piece of land ABC are ox ‘A= 35°14 97" Cm a8" 15°05" Bs eines The most probable value B is nearest to: ‘A. 96° 30°15" ©. 957 30" Os” 1B. 96°30" a3" . 95 30/12" 2A. Aclosed traverse has the following data: Line Bearing Distance (1m) AB = 6000 BC 7268 a 4433 DE 565 FA 50.00 Find the bearing of line BC. A Sas: 57 © Sse aE BL NAS? 11'E D.NAS*o0'E 25, The perimeter ofan ellipse is 28448 units. Ifthe majar exis 10 unit, what fslength of the mine axis? a8 ca we eee D.6 in a certain stretch of the north expressway 1200 vehicles hour. If Seeaiicdenaty ‘eles per Ktomter what the space ean speed A a8 ca mea ae D. 36 2, ‘The driver of cas travelling ata certain speed suddenly sees an obstruction Soe ee ‘of 25.acconds. Determine the car's speed of approach in kp A856 108 BL 953, D. 1243 25 A cera copier machine cost F500 wih adn val of PS afer es. Using the dactining balance method, what 4 the took value when the machine had made 300000 copia? A. PST C Pansat B. P64.896 1, PSS. 254 * Mathemacies, Surveying, and Civil Engineering 5 4 May ‘Transportation Englnceriog, Reference Vol. 2 Be Deeemine te equation fe rect oftheceg =U Solutions to May 2000 Examination rea=0 fe j cd eB Ah Be. B yea=0 De ee 90. Find the volume of aright circular cylinder whose fater ne mi aee ie Tee ni ©. 2053 | Let be the number of years: = . D, 12 Fe Pasay B, 15.69 4P = P+ 02/4" Losin 44 Qn 1.05) ~ ind; t= 7 years m2 aE Ame” a0 BS A+ B+ C=1B0" = 125 + 94° +73°=180"= 107° * ? i (90)? (100") Area = 52000009 5708 « gee) = 15708 a9, m. ms 405° x 10m 7 200 mils ma Apnarl 10053 © #(4) Li L=8.em ms pa Alea" Convert 75% compounded continuously to anmuall ER=ERe : e (+p tee- i= 007788 __ Sx 0417788)" —1) 3 P13 5413 me 729-39 ac=3Q)1= 81 m7 logs975 =x x log 975 / log 8= 3.31 Mathematics, Surveying, and 6 May 2000 “Transportation Engineering ° OE ie [et eon tom fru eee u= 3 sin 0; d= 3.08.0 pe Note: x/2= 90" = 19086 na) yr yt Vig 2 = G00 sin 20°): 4981) F @-20921~612=0 {#212 seconds m0 an Bv.= FCO ere dS oi (OWE -orre FC Fo ‘Bs =-499,000(1 0.1746) = PL83892 on otxr ls ‘Thus, among the choices, x may be 13 ou yar? =r Slope = dy/de = 3x? -9 = 18 B= Di ke3 v= tla, +As+JAiAal Ar = 33(9)(9) sin Aj= 35074 a? l= Vie sin 6 A= 043332 135= © |soq74-0430e" + fSS074(04837) 0433a24 98975-15.551=0, x=3m. The volume in the first coctant is 1/8 of the volume common to the two oplinders, The volumecommen to the two cylinders isa prismatodd. L ve Ehsan Ela +44. +4i] [=12 Av=0 As=0 Ae = 12012) = 144 v 2 {0+ 4(244) +0] = 1152 m> Required yolume = V/8 = 144 m! Masry 2000 ae mie aso fe dah axel m7 \ d0/dt = 0.25 rev/hour | ‘oats Ob rad /our = oe tan 0=2/2-05 x Boos a aH] When 150-2658 62 26.565" (05 9) = 05-7 a For a given total surface area, the volume is 8 maximum when the Thame) eal te eight) Ap= ED? 2+ aDH =2 H+ a(f(H) = 153.54 H=572m my be fps dA any? dam (ay? 8) dy fvetw-[ Civil Engineering 9 Reference Vol. 2 20 In right tangle ABO: R= 2608+ (R64)? = 6700+ RE= 12BR + 406 R= 560.125 m sin = 260/560.25 6- 7a" Lengthaf curve from PC toA: L=aRO/ 180° = n(500.125)(27.657") / 180° L=2037m on Sea 12240 rom th tring showe B Pasta Note: the slope of ofa parabola changes uniformly. The slope ofthe curve atany point isas shown in the grade diagram From the grade diagram shown: L 20 12+06 O18 £=200 m a Mathematics, SurveyiOg, nd “Transportation Enginectiag angles ~95° 14’ 97" + 96° 3 09 +457 15" 05" angles = 179.9975" = 179" 57°51" ‘Since the sum of the interior angles ofa triangle fs 180", the total error Eis, E= 180*- 179° 9 51” = 0" 009" Most probable value of B= 96°30" 09" + (0° 0' 09°)/3 Most probable value of A = 96° 30" 12" Solving forthe dosing line AC Tine Gearing Distance Latitude Departure CD SAP20E | -a4a3 ARTI 13.3562 DE s7rsoW 5645 18.7505 S82 FA N7@90W 5000133619 48.1815. ‘Sum 81828 88.0702 hatitude of AC = +48.1828 Departure of AC= +88 0702 Bearing of AC=N 61.317" E Length of AC (48.1828) +(88.0702)" = 100389 In triangle ABC: 100389! = 602 + 72.602 -2(60)(72.69) cos 8 p= 97.868" 28 . 03 8292 sina in7 B83" Gan Bac = 6131" 45.8297" = 15.4808" (Onc = B= Dan = 97.9683 - 15.4808 = 82.3873" Bearing of line BC™ S 82° 25! B Reference Vol. 2 a m2 Pads, minor ands = 2k= 8 p= 1300/25= 48kph S=ot Famv (2 (0 =28.8 m/s (1 km/ 1000 m)(3600s Brae ype roe mci) Poser 14 ro Kat = son neef 15,000 "i500 2 an mp = 150,01 - 0 = BV = POR 54S it ‘00000287823 ael6y dan 16 and Equation of directrix = y= ory +40 Ar= P= 7.068 r=1sm Am 2erk Ac=2n (1.5) k= 25918 yetneBe = Ab h= 7068022 viisarne wyanuiieans =| ~Ss«HVil Engineering 13 ees Reference Vol. 2 “P Aglindseal nk 2m i iv it diameter is divided into two equal chambers by artition along its diameter, and provided with 20 cm x 20 em orifice at the over geno ofthe partion Aa certain inet, dhe evel one chamber nat is ism higher than ies inthe other chambet. Hlow long rl i lake for the caval EXGINEER Lge Pana (200 p.m. - 06:00 ‘water surlace inthe two chambers vin the same level? € 0.6. sae © 36s 575 Bi. 14s beled, TGaL Ne SELA 82) tee taaputas Eee of saat eet proportion fe & bese width of Sm Te ym is nid ona slope s hus a roughness coefficient ry and the discharge is 3c correct answer for each of the following questions m/s when flowing full. If the same material were used for the most PNSTRUCTION: Select the (Oem by shading the box corresponding to the leter efficient trapezoidal section, by how much would the discharge be Me an ae te answer sheet provided tam ’ eer ee ERASERS ALLOWED. Use pellon 207 fh datavrs Boats MULTIPLE CHOICE FS REN tl 1 Jjanincatacoomonfuetan no eaten bs Rowor A bat side of a tanieeontaining © elevation 933m voir Bis at elevation 844 m. The properties of each 1A pressue gage at elevation @ mg arn aeads 100 KPs. What the pipe areas fllows: [r= 1500 m, Dy = 600 mm. f= 02; r= 1000 my Be = reads SOKPE. Another gags mim. f = 0.025; Ls = 900 m, Dy = 500 mm, fy = 0018. "A pressure gage at oa er cin Faction Preads A050 KPa. What is the flowin pipe 3 in m/s if D1 A. O91 3 mat water with 20 em projecting above the water B. 08s D. 076 2A block of wood Maus wate 3 Wil gat wrth 15cm projecting abot 1. Arstangolas footing sto support wo square caring excl 12" 12" and surface, I place rig the height of the block? spaced 12 feet on centers. column carries a load of 40 kips and the the oil surface. C. 50cm other carries a load of 50 kips. The footing is 2ft thick and its length should =p oan ponzanii sodsmendiamets pipe. Upstream ey nee swine vere Keer cong : ae di Upson of footing low the ground surface. Assume specifc gravity o Water a theyll comer gage @O0 m aparcshowed derence of [40 He, concrete and’ soil above the footing 10 be 24 and 178, respectively fro the com rom Ue connection ive gages 600 m apart showed 8 nesters Determine the length of the footing if the allowable soil bearing capi owt ee Ascuming f= 0015, how siach water 1s being 2000 pe ‘of 126 kPa. AL 1634 ft ©, 20,74 ft Pipe. “¢, 11L/s B, 15.21 ft D. 17.23 ft _ He 12 A spillway, 35.65 fret whose rest at elevation 400m elnses water fern eerste etic res tra. bccn ial vrcre Seven 2 of the same 4. Two pipes 1 and 2 of te = is 418 m, Howr long will t take for the water level to drop 1 meter? parallel, It f= 2/3, the flow in pipe ‘A 30m 20 min pipe 1? Cc. 1487, B. 15 mir D. 25min a eS D. ar inal, If the water 13. The asus wis crest ee im pean Head ninatt ee “ way. and into & ca reservoir 50 hectares in area. If the reservoir level drops from elevation 5 Miner ws oe a dep of ow ae OP? thoi man deer find hen pny nce Use comes Cab Pe rani Formula Assume cormant reservoir aura sre, Te supercritical uy sides sloping 45" has a base width of 2 m. B 478 D. 324 6. A trapezoidal ae Mihat is the hydraulic radius fr this condition’ 14, The ratio of the votume of water tothe volume of voids tie” aoe ae oe B, 075m 6 ottonn and 2 gh as ater 2b B. void ratio ee ing uyarostatic uplift, what i the soil pores ae yelght of masonry as 2400 Kg/e fhldeep on its vertical side pressure atthe heel in Kg i © 320 “a. 1580 34780 pay 14 May 2000 eect Eogineeront — oe Water Weight of moist Content soln Prot 8) ‘mold (grams 40 185 2 11606, 4 1696 16 \ 8 1741 20 L651 the volume of dhe mold for this tests 1/30 euble feet (916,000 cubne rmllimeters) Which of the following most nearly gives the maximum dey anit weight of the sail in grams/cc: 1S ew Bie D.14 16. Weer Bw in 10 fect wide rectangular carne tse of Sheu Set pesca. What isthe critical depth fortis condition? A108 fe © 1228 Bunt Biz 17. Avoid srorple has a moisture content of 30% and degree of 8 Soe" Tne atid eo spect gravy of 261. Determine the ry wit ‘reight ofthe soi in N/m 852 C94. Brit D. 1025 1B Ane ot moist clay i found to hae maisture comment of 400% 350 A. see Sauurition of 48%. The specif: grevity of the abide + 278 ‘Determine the voids ratia of this soil. ABD ©. 1028 5 sat D. 1299 19, A Supls of moat sand taken From te ill was found vo have 8 etna 2h mpc cand a porosity of 38%._ In a laboratory test that cinalee ant as waa found chat ot its densest stat, its void rio 8 Soest sone its vokd ratio is 40% Determine the relative of the ‘A052 coe B 08st B, 0472. ap, Fit deiry test on # compocted fill of sandy clay gives the folowing results: "Weight of mofst sol from the hole = 1038 grams ‘Ovetednied weight af the oil = 914 grams Volume of test hote = 0.0169 £0 tabortory aeisire deity test on his sll indicated « mazar a see ee cen mcstare content of 1, Which of the denaty of 120 Pe gies the escent compachon ofthe il ‘836 : cars Dae 6 B. O72% 1s 21. A strong oxidizing agent R aa TE improves the effectiveness of va ceo pee by by, elmer of Sa diene asa oso Ricaaeeen at oe 2 oa Same The dachirge ever retangnta wed we Te ectangalar suppressed weir my Zande 2d th bac of ts War ind he engin oft vest of the irri tebe SD om: Comsoe he velo of spac wl he A tsiee 163 eine Dim Water flows through an orifice atthe vertea! side of water tank under a total head of H. The jet strikes total head of. ‘The jet shes a point 25 m below the orifice and 25m sizoatally aay fom the vena contracts. What isthe vale of the head? B Osim D. 083 mip Mydrautics and Civil Engineering 16 May Geotechnieal Engineering. Reference Vol. 2 fr Solutions to May 2000 Examination Oe as 00826 LQ? _ 0.0826 f2t3037 m1 DF DS nahath sp where Is = Zpand D;= Ds 100 = 80 7 (6= 4)-7= 10 kN /m? a fi Qt (2/3) Oe PB ie mz ms eehcoal ofthe block. R-AlP ‘i A= 20) +4) x2 Heat paatt "i = 24 1AUdn2= 4.828 m AGH -20) = 2 : eee on) R= 3/ 4828 = 0.621 m a? hot Va pane Vp= Va Wi = 345,600 ry Vite meas) 8 AH -15)= SAH) Wag = 230400 leg O8H-12=SH 3 (2) Puyka eres = s000(00.5)21 + 1} Se eae geen P= 220.500! M=0at-12 Haan ree gs Ry = 576,000 kg Upstream from the leak: pstrear aD RM =345,600(8) + 230,400(14) HL= 0 um RM = 5,950,400 kg-m Y 981 5 ‘OM = 220,500 (OM = 1,513,500 kg-m one pe senate =i Qu= 0216 mys Ryx = RM_OMe Downstream from the leak: 4, 126, Bie BER = on a ee Qos * Qu Oo= 0216 ~0.205 = 0.011 m/s ‘Quin = H1 liters per second 576.0007 = 5,990,400 = 1,543,500; = = 772.m s08/2 F~02hm ~ (8) 526000, 6020 rae Ce) oe 39,700 ae pros 52220 ge 18 Gs me May 2000 ‘Geotechnical Engincering an 1 Ake re Arn An le 5 @P=H/2 H=3m Hs CA. fig =0.6(02" 25 cA. fig = 0.1088 _ (nf 2/2) _2_ (5 J) rer one 12559 see O-KARY asm / 3= KES \O7ERA k= 08075 For the trapezoidal section: ‘A= H4A+ 2/1732) AaS196 mt Red/2= 0866 0 (= OBN76(5.196}(0.866)2 O=3fla/s < Increase in flow = 0.81 m/s. protersiao: | Chingthentaimsti 29° = aans | ie «2 Oogles civit Reference Vol. 2 19 Gio i= 624m 8.0826(0.02)(1500)0)7 _ 55.4 i Qr= 1399s iyp=ao6 = 2082s(0025)(10001,* (0.45) r= 0886m"/s Qs 01+ Qs Qs = 1.399 0488 = 0.911 m/s, Pl To solve the elation of seserooirC: .0826(0.018)(900) (0.911)? = 05° 2 El, C= 8706-3554 = 835.06 m 354m For the pressure in be the soil be asm | ese | sav uniform, the resultant foot load! should coincide with the centroid of the footing. Locate the resultant load by taking moment about the 40-kip load: 90x 40(0) + 50(12): x= 6.67 feet From the gun: L/2=25+ 667; L~ 1834 feet Tosconypte the with of fot Eifective soil presiu 9, = 2000 = (62:4 x 24)(2) ~ (62.4 x 1.78)5 ~2) = 1367-264 pa Total load = 40+ 50'=90 kips = 90,000 Ibs Aang = 50.000. ene” 1367. 264 Lx W= 1834 x W~ 6582 W= 359 cot = 65.82 fe ycraulics and on 2A, {_1 2A 35.65 «1089 Bl ae ge 240,000) 1488 seconds = 24.6 mine xem | eH Fl ee Percent ompaction = Dryden et Sei, = ‘Maximum dy deny 28 sla cy dey 139 pot ae 7 Dry density, e= v= 200 ie $l he = 00016 FO (129(25) = A7ASS ce ' W914 / 478.55 191 gram/cc™ 119.18 pof as mae T1948 e Py cick, Percenteompaction= 1248. 00% = 993% = Wat Wecny* WeW, an Bie i First we neglect the velecity of approach to get nee Wg veo oe taf approach to get the = 1.8 CH? - —— 053 184 L, (03) L= 1.753 m MCC) (gem) Teme, nen = 088 8 025 m/ a = a A 175302) x ao 3 13138 2 _ 028 tf 18 13 ten 2 = 82 nonsi86 m ie er 19m 2g 21981) i 15596. Finally: Hs a 1a Gaia LG saya iar] 3 = 184 L [(03* 0.008185) (0 a e Sasa (0.008186) = ef 8/5 per foot wi | a ane cate per foot width as phe hs yesent 2 aor net = 20,7 cos? @ ones aaigay=o0e «8 2525 tan 0° POM2SY «502 m/s tan Ete ote nan = BH HN? Pree ce y= f2gH = 3502; = 0.605 m as | GMC~8¢ 276{8)= 085; c= 1299 ‘Structural Engincering: 22 May 2000 ‘and Construction SeatNos censure Examinat Senday, May 12 200 0:00 am - 01:00 pn JL ENGINEERING ANI ETH SErA INSTRUCTION; Select the correst answer for each of the following questions lark only one artswrer for cach item by shading the box corresponding tothe loter ‘of your choice on the. Feelin STRICTLY NOERASURES ALLOWED. Use pencil no,2 only. MULTIPLE CHOICE Situation 1 In the connection shown in Figure S23, load traits 200 KN Het acing at an eecenrlty of 200 ree For this problem a= b = 250 mm The ibet S tranmmitied te the colar by the plates and 8-22 un diameter tres The pc ae adage ore th lo he as Te Sraneton che analyzed by replacing the given load with on equtvle Tonding composed of a vertical force alone acting through the centroid of the frets da moment 1. Wve of the following most neatly gives the maximum shear sess inthe Hv in MogaPacals for evalest vertical force alone acting on the centroid. oi bm Ae 2. Which of the following, mast nearly gives Uhe maximum shear stress in the retin MegaPascas for the equivalent moment alone. a a tn be 3, Which of the following most nearly gives the maximum shear stress in the rivets in MegaPascals, A 10 m0 B, 160 D130 ‘Situation 2 The system shown in Figure ME-O8 consist af rigid bars AB, AC. MMBD, de CD hinged at paints, B, € and D. When P is zero, points B and C are concent 4.5, C&D ison astright ne onthe tension on AB "200 N. For the given arrangement in the figure: 44, Wich ofthe following most ely gives the tension on member AB in N. A. $60 C 980 5 1130 D. 1020 5. Which of the following most neatly gives the value of Pin N. a) C860 8 1020 D150 6 Which of the following: most nearly gives the spring constant K for system ABCD in N/m. ‘A. 3450 2000 B 170 ©. 3560 Tene ee te aa Civil Engineering Reference Vol. 2 23 Situation 3 The tes concurrent forces shown inthe figure are in sqallibrim: 7" Which ofthe folowing mast nearly gives the valve Tangle 8 A 52 ue Cc. a8 - a tich of the following most nearly gives the value of angle (i. A BSP a, CT vi B. B® D. 76" 9 AGREE Ss alee anh 6 Menbinbtt itl iee thal ey on ee oa Situation 4 - A prismatic beam 8 m long is fied at the left end and simply ‘Supported af the right end. The beam carries @ uniformly distributed Toad Of 400 KN /an throughout its len 410. Which ofthe following most nesry gives the deflection atthe right end due +0 uniform load when the simple tupport is removed ‘A. 237600) £1 . 184500/ Er 214700/E1 1D. 204800/ EF 11. Which ofthe following most nearly gives the deflection atthe right end due {2.8 un Tod one oti the right end, with the simple support removed, ‘A, 614/SEr ©. 457/387 2 wee 1. 537/581 12. Which of the following most nearly gives the reaction at the simple support. ALOK yee ado KN se B. 1500 Ky 1D. 1000 kN Situation 5 - For the structures shown in Figures AN-07, AN, & AN-03, identify whether the stricture is unstable, statically determinate, statically {indeterminate to the first degree, and statically indeterminate to the second 33. Thestructute shown in Figure ANAT is ‘A. unstable Bi, statically determinate statically indeterminate to the first degree D, statically indeterminate to Une second degree 14, The structure shown in Figure AN-08 i: ‘A. unstable B. stalically determinate ©. statically indeterminate to the first D: statically indeterminate t the second degree 15, The structure shown in Figure AN09 is: ‘A. unstable B. statically determinate © statically indeterminate to te frst degree 1D, statically mdeterminate to the second degree strvcturat Encincering 24 May 2000 ‘and Construction: Situation 6 Tor Uwe spiral cokumnshown in Figure RC-S2: 16. Which of the following most nearly gives the reinforcement ratio py in [peteoa, fhe ccventrichy 6 150 limeters. A 00325 € 0.0242 Whit he Telling most neal gives the ao ofthe conte to conta 17. Whid wring most nearly gives the ratio of the center to center sresing of reisfvrcemsmt to the cofamm dimension in the direction of Send A 045 C065 B 078 5. 055 ng the interaction Diagram, which ofthe following most nearly gives the naceanim downward lose that the cohumt can suppor, in KiloNewions ‘A, 2350 © 1730 B 2a50 D. 2230 18: Situation 7 - Section 5.11.6 of the NSCP states the following for the combined, Shear and torsion for non-prestressed meubers with rectangular or flanged Sections: (For the following discussions, x = shorter overall dimension of Tectangulay part of cross section, y ~ longer overall dinvension of rectangular 1 of ere section, h = overall thickness of member) S151 Torsion effects shall be sncluded with shear and! flexuxe where factored torsional moment Tx pee sty) Otherwise, otherwise torsional effects may be neglected ee S.1L.611 For meinbers with rectangular or flanged sections; the sum Ex'y shall be inken for the component rectangles of the scion, but dhe ‘overhanging flange with weed in design shall not exceed 3 times the flange thickness. S:1L.6 Sections located less than a distance d from face of suppext may ‘be designed for the same torsional moment Ty a4 that computed at a distance ‘A rectangular concrete beam 300 mm wide, 600 mm, and $m long deep is tied to tery the slab opstem shown in Figure RC-12.The lab is 200 mn thick and carries a total factored load of 8 KPa including its own wel Tesora enforcement i ob povided om the web oly. Assume /: = 2075 MPa 18. Which ofthe following most nearly gives the torsional moment on the beaan ue to the overhanging slab in KNEm. A C40 B 50 Biss 20, Which of the following most nearly gives the value of the sun Tay, due to the component rectangles of the section, in mv. ‘A of © 015 B01 D, 025 21, Whichof the following mast nearly gives the walve ofthe Kimiting torque T, ine. ‘A720 c15 BB D. 30 ‘Civil Engineering on Reference Vol. 2 Situation &- The implementing rules and regulations of FD. 1594 states that for lunit price contract, quantity overruns or underruns of not mote than fiftoen percent (15%) of the estimates per mojor pay item (ie. pay item which represents at least 20% of the total estimated cost ofthe contact) and twenty-five percent (25%) per minor pay item in the bill of quantities need not be covered by a Change Order provided that the same is authorized by the Approving Authority for the contract. The quantities and contract prices fer a project are as follows: ‘Quantity Unit Price (pesos) 100 1,000 Tem Item? 20 00 tem 500 00 Tema = 400 em Fr) 20 2. Which ofthe flowing most ely give the toa emma pce ofthe Pr parso00 © P5600 23, Which the fll era ; following is not considered ena alor pay item ofthe proj 2 item a a C item Pe 24, Whit the fl be covered by a¢ L. Which ofthe following may not be covered ty a Change Order: YO UN Decresse inquatry rom 20t0 Isp forem 2 B. Decrease in quantity from 250 to 160 for Item 4 'C. Increase in quantity from 150 te 200 for ler 5 1D, Increase in quantity from 500 to 600 for Ikers 3 Situation 9-A simply supported beam with span of 10 meters is subjected to a ‘uniform vertical downward load equal to 50 KiloNewtors/meter acting on the plane of the minor axis of the beam section, which includes the beam weight The beam is restrained against lateral buckling of the top and bottom flanges (or the entive span. The material is A36 steel with F, = 248 MegaPascals and modulus of elasticity = 200 GigaPascals. The allowable flexiral stress for laterally braced compct sections is 0.66 F, The allowable deflection is 1/360 of the span. ‘compact sections are being. considered, as follows, with their respective properties relevant of this ‘problem (length units are in meters): ia(fs) Beam depth (D) 0.654 Section Moment of In W25%66 O.oo12e W805 74 0.00146 or W24x 62 oor O18 (Note: The symbol x“y means raising the quantity or expression = to the Exponent Oa, Willen Gf oe bolo mic neat pire he aninianh eicion eucdioe (Ga) n meters, such thatthe masannus Hensal stress Will not be exreededs A ons ©. 0.00525 B cons76 D005, 26. Which of the following most nearly gives the minimam moment of Tnertia (G2), in meters“4,sach that maximum deflection will not be exceeded ‘A. 0.00128 © O.om108 B. 00098 «Boom? 26 May and Gonetrecton, 7, Assuming that shear wrest is not enucal, which of the sections being consideved is the most economic section that Is adequate for the given load: A. 20 74 W268 x68 B, W24 x62 D. Nene of the sections fare adequate Consent of exterior wall Pam ce [75-1 —Crstction a erie parion [sa esoneaeatsenial cares ——T aes "EO Pag ana ig — 28, Which of the following, gives the critical path of the project: A weenie © bd B. abd-g-ln Di echilgbnen 29. Which of the following gives the curation of the project in weeks. AO CR Be D3 30. Which ef the following gives the cartiost start af activity m in weeks: A 38 ca Bas Da FIGURE ST-23 Civil Reference Vol. 2 TDeERACTION OULGRAM 1 12 140 GP UANorh = hla At), Noe: 1 ka = 8.8028 2D 8 18 28 May 2000 rsa conection nemrense Vala 29 ——— ss Tr Solutions to May 2000 Examination 2 Situation 1 (1103) Analyze one side of the bracket: iret load on one rivet: Ro= P/A Ro= 25kN ret ‘Stress due to direct load: T= Pe~= 10000) ‘Tm 20,000 EN-sirh Eee + yA) = (125? + 125%) x &= 125,000 mm? Figure AN-07 Re fete? = (20)? + (20)? -23294kN Sree R= 28.284(1000) Figure anoo Bee te Stress" 744 MPa The most stressed rivets are rivets 8 and C: -- END“ R= (442 =49.244N 492411000) | Snax S2HUOW) 320.5 Pa $225 1D Situation 2 (406) | sin = 025/05 = 30° wo 1A ho cena GA 18 BA WB ce Ke mA BO mC zt bret WR MO MD BA = 014m BA DB BA WE ze | Fa kex =10.2(0138) = 1367 KN = 1367 N Struccural Ensineering 30 May ‘and Construction Brom the FAD shown Atjoint Ai F +200 = 2(7'cos6) 1367 + 400 = 27 cos 30° T=10202N £ 00N AtjoineB s Po 27 sin 0 = 2(10202) sin 30° P= 1020.2 Spring constant, kare P= kacx= kc (05) 10202 Kage 20404 Nf {Situation 3 (710 9) For the forces to bein equilibrium they must fora closed polygon (as shown) By cosine law: 728 = 622+ 6D - a=7574 2(62.2)(645) cos b 22 aS ysinelaw: £22. 45 _ Bysinelaw: ina” in a= 507%; y~S347" ain 7B $204 30° = 75747; B+y+ B= 180 45.75" +5347" +p = 180"; B= 8078" ‘Vertical component of 77.8 kN force =77 on 4s.78° Situation 4 (10 t012) Deflection at B when the simple ‘support is removed: = E008" oa og : SET ‘SED Deflection due to unit load at B RE IG sca Sef 3EF eu 3, 2H800/El ssa kn 1 Reg,” S1a/3eF Saxe tinted Civil Engineering Reference Vol. 2 @ Situation § (13t0 15) Figure AN-07, (PIN-ROLLER) Reactions, R= 2+1=3 juations, E = 3 MINATE Figure AN-08, (PIN-ROLLER) Although there are three unlmowns avi three equations, the structure ts UNSTABLE beonuse the reactions are concurrent. The alt meet at the pinned support Figure AN-08, (PIN-ROLLER) Reactions, R= 2+ 1=3 Equations, E=3+2=5 (there are two internal hinges) Since E > R, the structure is UNSTABLE Situation 6 (161018) Ay. $2278 | t= Sr os = 375/500= 0.75 eee ; «/h= 150/500 =03 76 WickucHowDuGRAM fendi cTene y= a t612 7M Scrat Engineering «Civil Engineering 32 May 2000 ‘and Construction Reference Vol. 2 From the interaction diagram | WiSitustion 9 @t02) $F. 1a hax 68908 = 8961 MPa | i | 0.75 P,= $ GOO} x 8.961; P, ~ 2346 kN © Situation 7 (19102) Tateioral ment cue 25108 Eee = 066(245)5, = 2610426 sum’ ~ 0.00082 0° Teas 18075) i & Hane a T2aSkNmn = Salt _ Sea) 'o0Hn" _ 10000 Eanes | Sau Sea] 342000001 ~~ 360 a2 | T= 1371875000 m= 0.01172 mt Devs ose | “The required sstion must have [= Q001172and 5, 0.00362 P | For 7256S = 00247 (4/2) = 05079 adequate a. For W'30 74, 5, = 0.00146 / (0772/3) = 000878 (net aeuatc tee cy = SIE a2 onan For 262 5, 000118) (De10/2)- DOU adequst Ty 19746964.78 N-mm = 1995 kN-m ES Thus, the adequate section is W 24 x 62 tation 8 (22 1029) eat 1D Situation 10 (25 to 30) ‘Oty Tem T [100] — 1000 Tem 2 [200] 6007 Tiem3 | 500 | 100. Tema] 250] 00. Tem | 150] 700) “Total Project Cost 3P- TL05% Bre, Total contract price of project = 475,000 Minor pay item = Item 3 Ikem 2 (Major pay item) from 200 to 150: Decrease = 50/200 x 100% 25% > 15% Needs Change order Item 4 (Ma } i | | From the diagram shown, the critical paths: acbed:efeomen ay sem) from 250 to 160 Decrease = 90/250 x 100% = 36% > 15% | Duration of project = 65 weeks Mezds Change order Earliest start of activity m=45 weeks Item 5 (Major pay iter) from 150 te 200: Increase = 50/200 = 25% > 15%. Needs Change Order ‘A Item 3 (Minor pay item) from 500 to 600: Increake = 100/500 x 100% = 20% <25% No need of Change Order mathematics, Surveying, and 34 November Transportation Engineering ‘Seat No: ___ CIVILENGINEER Licensure Examination Saturday, Novernber 18, 2000 6,00 am. -01:00 pm. Marit SURVEYING & TRANSPORTATION ENG" sera ION: ‘Select the correct answer for each of the following questions. in anf anes ov cheb cig te bo comengncig tle of your chotcean the answer sheet provided ICTILY NO ERASURES ALLOWED. Use pencil no. 2 only. ‘MULTIPLE CHOICE 1. Twenty-eight persons can do ab n 60 days. They all start completa, Five Ponane ated the fob atthe beginning of the 10h aye they ere Teinforced with 10 persons at the beginning of the 45 day. How many days was the job delayed? ‘A S78days VS ASR days B. 114 days 1B. 245 days 2 Point A is between points B and C The distances of 8 and C from point A ‘8 1000 m and 2000 m, respectively. Measured from point A, the angle of clevation of point B is 18° 30, while that of point C is 8° 15'. Find the difference in the elevations of B and C. Consider the effects of curvature and refraction, ‘A, 444m © am B. 326m D, 521m. 3. Pind the area of the curve xt + y+ 6x 12y49= 0, A. 135 eq. units C9259. units B. 113sq. units D, 13889, units 4 Bnd he dance between the fot ofthe curve sie 25y'= 18+ 100y- 115-0. A? : B 6 D.12 5. A right ilar hexagonal ym is inscribed in a right circular cylinder frtose eight Is 20 che. The difference between the circumference of the Sele and the perimeter of the hexagon is em Determine the volume of ee es ce C. 10857 B. 114752 ce D. 10367 cc 6. A.solid has a circular base of base radius 20 em, Find the volume of the solid if every plane section perpendicular to a. certain diameter is an isosceles right triangle with one leg in the plane of the base. ‘A. 21333 ce © 18667 cc B, 24155 co 1D. 20433 ee 7. What's the area bounded by the curves y= 4x and a*= 4y? A 60 © 6665 B, 7333 D. 5.333 t Civil Engineering Reference Vol. 2 a 8, The slope of the curve at any point is given as 6x ~2 and Ghe curve passes through (53). Determine the equation ofthe curve e ‘A Bxt-2e—y-62=0 © 22+ 9x -y-62=0 B. 2a-3e4'y4 52=0 D. 3s°+ 23 -y + 62-0 9. Evaluate the iniegral of x dc / (c+ 2) with limits from Oto A032 © 0.208 B. 0.108 D, 0,247 10. The ares bounded by the curve y ~ sin x from x = 0 to x = x is revolved about the Xoaxis. Whats the volume generated? ‘A. 2.145 cu. units C. 3.452.cu, units B. 4.995 cu units D, 5214.cu, units 1. A closed cylindrical tank having a volume of 71.57 m? is to be constructed. If Uhe surface area is to be a minimum, what isthe required diameter of the 35. tank? 4m C 5m B 53m D. 45m 12 A flagpole 3m high sands athe op of «pedestal 2m hgh load at one Side ofa pathneay. At the oppose side ofthe pathway directly facky the flagpole, the fagpotemubtents the same angle a the Pedestal. What the witha the pativay? Ata ©. 628m B 321m D. Bal m 15, A car travelling at 60 kph applies a brake and stopped at a distance of 30m. ‘The coelficient of friction between the tres and the road is 0.5. What is the grade of the road? A. 43% c 28% B. 43% D. 28% 14, PLO0,000 was invested to an account eaming 8% compounded ‘continuously. What the amount after 20 years? A, P4A5279692 ©. P5,365,147.25, B. Passaorza2 D. P5456.254.14 15, A man made « year-end payment of 100,000 to an account earning 8% annually for IO years. How much is inthe sccount alter 20 years? A. 751275408 P3.327,45236 B. P4075,45899 D. P3,247;111.25, ‘16, The cost of producing a certain commodity consist of P5.00 per unit for labor and material cost and P15.00 per unit for other variable cost. The fixed ‘cost per month amounts to P450,000, If the commodity is sold at P250.00 ‘each, what is the break-even quantity? A 2014 ©. 2509 8. 278 D. 2367 17. The number of accidents for 4 years recorded in a certain section of a hhighwvay is 3:768. If the accident rate is 4.220 per million entering vehicles, ‘whats the average daily trafic (ADT)? A. 588 412 B61 D, 527 36 Nowemlber 2000 18. From a paint A on a simple curve, the perpendicular distance to the at point Q is x The tangent passes through PC. Point A is al station 20+ 260 and PCis at station 20+ 180, If the radius of the carve is 600 m, find x A. 624m ©. 739m BL 247m D. 622m The ground mikes a uniform slope of 4.8% from STA 12 + 180 ta STA 12+ 250, AESTA 12 + 180, the center height of the roadway is 1.2 m fill. At the Other station, the centes height is 3.11'm cut. Bind the grade of the finished road. A. 42145 ©. 1478, B 2.149%. D. 1357% Aline in a map was drawn at « scale of 1.25000. An error of 0.02 mum in the drawing is equivalent to how many meters in actual? A Sm C, 005m B08: D, 50m parabolic curve AB 400 m long connects two tangent grades of +6.5% and. ‘3, Ifthe elevation of the summit ts 123.256 m, what is the elevation of point Pr A211 my ©. 121361 m B. 1194625 m D, 120542 m ‘The geometric mean of two numbers is 8 and their arithmetic mean is 17. What isthe first number? AS qa B. 6 Das Solve for 8 in the given partial fraction: The motion of a body moving vertically upwards is expres: where fis the height in feet and 1 is the time in seconds. What is the af the body when ¢= 2 seconds? ‘A, BLT Spe C24 fps: B. 287 fps D. 356 fps Given that tan A = 4/5, what is the value of SEBA= CORA. 3cosAvsinA A. 0579 ©. 0654 BL 0752 D, 0925 Given « regular hexagonal with consecutive comers ABCDEF, If the bearing, of side AB is N 25° E, what ts the bearing of side FA? ANS W © Nas Ww BL Nabe w D.NS'W The perimeter of a triangle is $8 cm and its area is 144 sq. cm, What is the radius ofthe inscribed circle? A. 497 em B, 965.em ©. 5520m D. 312em Civil Engineering 37 Reference Voi. 2 38, What i the equivalent wecomgular Goorin of 0 pou whase polar ‘coordinate is (7, 38°) ei 2 x 56 451) © 5249 m Sass) D431 355 29. The base ameter of cone fs 18cm and sax inclied Ow the base ihe els 2 crn long. what Une volume of thecona? ‘A. 1524 cc 1uS ec BL 1469 ce D, 1689 ce 30. What is the magnitude of the space vector i+ 4j + 9k? A. 18,078 Cc 154 BL 15125 DTS END“ an Pam CSA WO HD ae mC J2e 1D wa we Be ZA 2g fA Wc uA Be BE WE HO MD Me 5D 8 %A M8 BA m0 38 Nowember 2000 —"“ranponaton nnginecring Tr Solutions to November 2000 Examination a1 Required number of man-days to finish the job = 25(60) = 1680 man-days 28 persons worked for 15 days, 285 = 23 persons worked for {4 - 15 = 29 days, and 23.+ 10 33 persons worked for t days unlil completion, 28(15) + 25(29) + 33t~ 1680; t= 17.97 days ‘Number of days the job was completed = 15 +29 + 17.97 = 61,97 days Number of days delayed = 1.97 days » Q0e7 Ra = O067(1 = 0.067 ha = O067(2F-= 0.268 Innight In night tramngle AEC: y gle ADB: y, = 1000 tan 18.5% = 334595 m 2000 tan 8” 15 = 239.986m Difference in elevation = (ha + ys) = (ha Difference in elevation = (0.067'+ 334595) ~ (0.268 + 289.986) Difference in elevation = 44.41 m Reduce 21+ t+ 6x- 12y + 9=0 to standard form: ait Gr+9+y2- 12y+ 36 =-9+9+36=36 Ett Wo r=6 ‘Anea = nrF= = (6) 113.097 sq. units aa xt D5yt= 18+ 1ODy- 116 Ofellipse) Sie! — 2+ 1) + 2548 + Ay +4) = 116 + 9¢1) + 25(4) = 205 IR a ose - Fay, an5.p-3 be cen VRIe = 4 Distance hetyrsen foci = 2e™ units Civil Reference Vol. 2 as Poste ~ Preven = tem 2ar- Grad; r= 14135 em VeAxh Ay ¥4(14.125)(14.125) sin 60 x 6 Ay= 518 36cmt Y= 518.36 x 20~10367.13 em? The solid i 9 prismatoid with the figure shown, Ve a saat. Va (dit dwt Ai) As Arno ¥a(40)(40) = 800 cm? v= 2 fo+800)+ 0) Vn 21,3383 ce ‘Points of intersection: ys4x equare both side y= 162 y= 16 (ty) 64 y =~ 64y=0 ¥y -69)=0 y=0 andy=4 When y0,x=0(0,0) When y=4, x=4 (4,4) ‘Using the formula: ‘Ares ~ Aven ~Aiganant : Area = $(4)(4) - + (4)(4)~ 5.333 sq. units + siope Sm en-2 | aymede-2ut Ylaiees 86.3) 3=36"-26)+G C=-62 ye St 2e- 62; By —2x-y-62=0 Mathematics, Surveying, and ‘Transporation Engineering "ede [ya = [Pin +3) ie re iat *?) aV= nytdee m(sinay de Vs aint de "a 4a[1n (= In @) |= 02027 Vex fowse eft Vee be 5 a 4.955-cu. units mu For minimum surface area, D= V=AsxH=4D:D~7157 Deasm m2 Intrangle OAB: —_Intriangle OAC © tan = 2/z tan 20 = 5/3 une 5 tanzge One, 3 Teun'o’ = og Bi —————————— m3 = 0028 = 25% ow Pei 1,000,000 ae F F=Pé953,0sra2 Civil Engincering Reference Vol. 2. M16 mis m9 41 ——— mis Let Fbe the money in the account after 20 year, then: Present worth of F = Present worth of annuity for 10 years, F____100,000((1+0,08)"—1] Tag” Goo 005 Poanz7 cons To break-even, Cost Revenue Fixed cost + Variable cost = Revenue 450,000 + (45 + 15) N = 250 N N= 2368.42 say 2367 units No.of cident = Accident rata No of enlclng veces a Atty 0 024901 vehicle 1,000,000 ae _ 2am 185) s768= ADT 61157 RO |e a = Seo. 12+ ito sa12 4250 y7336-311- 025 m Grade of finishes road = 72=¥ 12-028 — qgis67 70 70 Grade of finithed rand = 1.357% pases se tering 42 November 2000 ‘framsportation Eapinecring im Actual eror= 0:2 x 25,000 500 mm = 05m oa Sn 0 oS en1zsI6m ays 4(136316)(003) nabs m pa 1m2s6- 1285 HB e121 361 m ma on in Gir eet ae end avin boule the mcan Geometric mean, as = 8 aoa a8, gan a: 64> (1) Arithmetic mean, a) = 17 17—a,= 2-17 atand; w~B-a +2) InEq. (1): a, (4-2) = 64 a?- 34a +640; = S2and2 os xt-15x) ~32 4 Gees Be sx) 922 - 12-14 = Ae -2Yla + e+ 2) + Bee + I)la+ SiGe +2) + Clr # Ife Bae 2, + (Dx Bifee 1x 2yx* 3) Setx = 2: G- 1S(2I*- 8202) 12(2) - 14 = 0+ BEBO} +O0+0 270" 908; B=-3 b= 1001-16. Veloc at = 100-322 (= 100- 32212) = 35.6 Fee at tan A=4/5,A= 38.66" 4sinA-cozA _ stn 38.66" cos 38.66" _ BeosAssin A Seos38.66"s sin 38.66" m6 ‘The sum of the interior angles of the lot is 180°(n = 2) 180"(6 = 2) = 720" a~ 720" 6~ 130° 0-3 O+a+ p= 180° b= 190° - 25° - 120" = 35° Bearing of FA = N35°W mar 4 = semi-perimeter = d= 129, r= 4.97em ma 2 Fora point whose polar coordinate is (7,8).x= r cos 8, y= rin x= 70538" = 552 y= 7sin38* = 431 Rectangular coordinate of the point = (5.52, 4.31) he 20in 60° = 17.32em Va pau =9 Hydrauties and 46 November 2000 Geotechnical Enginessing TS. Twopipes | and 2 having the following properties are connected in series Pipe I: Length = 1500, Diameter = #60 mum, f= 0.05 Pipe 2. Length = 1200 m, Diameter = €50 mi. f= 0.020 Itis required to replace these two pipes with a single pipe whose length is 2700:m. Assuming that the friction factor for the new pipe is 0.015, what is the regres pipe laren? ‘A ote © 72mm B s78mm , Simm 16, This occurs whenever the normal decrease of atmospheric temperature or moisture or both with height changes anomalously over a relatively short vertical interval. ‘A. temperature conversion C. temperature inversion none of these D. surface heat 17. A sewage pipe carries: pi ‘A storm water fatat materials B potable water D. sewage Figures Figure 03 wow n a tl ee ih Figure og ‘ Civil Engineering a7 — —— ——<_—_________ i Solutions to November 2000 Examination ai 0 = 60"; i= 180-20 = 60° evecd Yi y ce OO" = LIS y ‘A=siyilinp Aa Viht3e7 yp sine Aq 057% P=2y.=2300Ly ged oy P” 2a08ty gaa lgifgia =025y anasrra 1 (025yF/* 000 ao7= yn y=1.678m Qu Dynamic force, F= 22 y= ne ea p= Bgl ort? = 2¢H Dynamic force = © (A) 2gH Dynamic force= 2 Ait Summing-up pressure head from A to B in meters of water 2A + to(084) + @=) Y (oe t 0+ 08a 3-x-a+2=0 O84) =1 m= 119m 48 November 2000 oy Solving for the velocity of the jet ‘a dhe stummit (highest point, Ay Bro am [iat tig? - {+0 = Sms ‘Ss is os cca Qo=Qs Aven Aae (0.05) (10) = As (5); Ay = 0.008827 mz ms ‘Note: MLD is million liters per day a ® [| um ora Lit, Jday the _1m? es em 500.000 ob x SER Sea Gapuie ”0707 ms From the figure shown Qs+ Qh= Oc = 0057875 (1) Hila= His 0,0826(0.025)(500)93" _ 0.0826(0.035)(400)957 035° oa = 078790, 228 InEq. (07579 3+ Qs = 005787 et 24h ou era IO, 2, Me (Qs 2844288 Lit/day = 284 MLD me Froin the formal: peo (Ti Altay CA, 2g ) Hy am Hy f= 2min go 9 _2__ ig 345 060.10. 1 ae in= 432m 120s 7 | Givil Engineering ‘Reference Vol. 2 =e] From the formula Y" dy tage Ba0= HORA 11.28 ee From the figure shown: rrv=0 BF, + BF:= 35000 B= yet Vo BE; = (62.4408)(1212.9) BE, = 21,5654 Ths 21,565.44 + BF; = 35,000 BR, = 1343456 Ibs BEL= ue Vo = 624 ((12)(12) 0] = 13,494.56 h= 14098 ht Fim pA =120 4 (0.4)2= 1508 kN nAz= 120 4 (04)? = 1508 kN 2 A 0.15(8.81), Sa (OH 198) Re = 1526 KN 7 ™ ww) Bye Doyo z By 1505» EES 94-0) n= 152548 Re RoR? = R= 2158 kN (05.267 + (15.26) ee 50 November 2000 ‘Geotechnical Engineering io 2 ae = 26.5%10,000) [14 see 1.84(10) | Hy ll Final water surface elevation = 42 + H2~ 42.921 m_ A= 41.3) am 2m + V3.5) «2 ~ a3 10.708 m \/P = 105/10.708 R=096 m o- J2eH = J298NG2) The aoe. essen) oat Geis9+ 1896 18941898 THe 224228189 +289 = 045 2158 (es) Porosity, n= <= SE Ia 088 Porosity, = To: 5 =031 = 31% roy r Civil Engineering Reference Vol. 2 ——————— 1 ais For the sand above the water table Yow" Taste om 221 24) = 0838 = GrGMe _ 27is 271(02) Tease 8) Yount * 1104 pet Teo Tend T= weharw wre +(E Ya ate ater in = (Ha ~ 62.4125) + (128 - 62-4)(30) * 110 Ter lat = SLaMZ2) > (128-6240) + 110A) For the original pipe: Discharge = Q= Qi = Qz Head lost HL Mi + hf Hig = DE2AONRS|ISONO? _ aoeasKe.can}e1200}07 (085)° 55) Hla 24.066. Foribe 6 Discharge=Q Head lost = Flo .0826(0,015)(2700)0* a D= 0874 m=674 mm = 24.0669: sreuctural Engineering Civil Engineering 52 November 2000 ‘nd Conseraction Reference Vol. 2 53 Seat New Which of the following most nearly gives the distribution factor al Bon member BC, in percent Use the modifed F ioe CIVIL ENGINEER Licensure Bramination 58 C78 Sunday, November 19, 2000 0:00 am. 01:00 p.m. Bae ne 8 Which af he following mot nearly gives the moment at Bn KiloNewton- STRUCTURAL ENGINEERING AND Ct ON SEEA 413 ©36 B10 ied INSTRUCTION: Select the correct answer for each of the following quest Se t Le egeai . pas Boaatatie: Situation 3 - For the frame shown in 15, the force F acting upward at C ser for each item by shading the box ¢orresponding to the letter if e ee ‘causes a horizontal reaction of 100 kN at ‘Mark only o ‘of your choice on the answer sheet ICTLY NO ERASURES "ALLOWED. Use pencil no. 2 only 2. Which of the following most nearly gives the value of the force F in MULTIPLE CHOICE A100 . 135 B. 75 D120 Situation 1 - A project has been bid out by the Department of Public Works and 8. Which of the following most nearly gives the reaction at A in KiloNewten. Highways. The approved agency estimate (AAE) is 500 milion pesos. The ACLs ©. 158 restlls of responsive bids areas follows Bsa D. 17 Bulders Peso 2346198 % Which of the fatowing mat nearly gives the angle in degrees that the Bidder B - P610-345,763.12 reaction at A makes with the horizontal axis (positive counterclockwise): Bidder C - P454,218,557.98- A, 260" Cc. 1a Bidder D ~ P389,122,897 44 8B. 30" D. Bidder E - P234,755 420.58 me: “The implementing rules and regulations of P-D. 15% states that no award of A S.cg Mock resting on a smooth surface is pushed horizontal Contract shall be'made to a bidder whose bid price 4» higher than the ‘force Pe ahuen in Figure MECH, The gph of foee ? vers tine wey allowable government estimate (AGE) or the Approved Agency Estimate so, hot inthe figure (AB), whichever ts higher, er lower than 70% of the AGE. The allowable ) Which of the following: most nearly gives the acceleration of the block remment estimate (AGE) ts defined as one half the sum of the AAE and Ele average of all responsive bids. For the purposes of determining the faverage of responsive bids, bids higher than 140% of the AAE or lower than 0% of the AAE shall not be considered. "No negotiation will be allowed to bring down the bid to the level ofthe AAE/ AGE 1. Whithof the following gives the responsive kidders for the project. A. Bidders 4, C,D,and EonlyC. Bidders A, C, and D only. B Bidders 4/5, Cland Donly _D. Bidders Cand D only 2. Which of the Following most nearly gives the average of all the responsive bids pesos A. Pig 356200.0 . ass 678,100.00 B, PS05 78,600.00 . Piei525,200.00 3. Which of the following most nearly gives the value of the approved government estimate (AGE) in peses <. Ps76.456,200.00 ‘A. PAB2.262-700.00 BL P507,455,900.00 D. PA$6,382,100.00 Situation 2 ~ A continuous beam is as shown if Figure AN-20. Using the ‘moment distnbution method and assuming E and | to be constant: 4. Which of the following most nearly gives the fied end moment at A due to the loads on member AB. ‘A 10 15 BD Dw 2 curing the fist two seconds in meters persacond per secon ie fas tenet which ofthe rs the ich ofthe following most nearly gives the velocity ofthe block after five yet ofthe flowing most eur gives the vlc ofthe lak afer © 120 ‘A113 B 102 B19 Which of the following most nearly gives the total distance travelled by the block in five seconds in meters. A. 395 420 B. 380 B43 Situation 5 - A rectangular concrete beam has a width of 300 mm and an fective depth of S60 mm. ‘The beam is simply supported over a span of & sv and is used fo carry auiniform dead load of 25 ki/m and a uniform live Toad of 40 KN/rm. Assume f= 21 MPa and, = 312 MPa. Compression ssnforcement if ocesary shail be placed at « dept rum fom the Sulermost compression concrete Which ofthe following most neatly gives the maximum tension see! are. forsingl enforced condition ene ‘a abl0 Bam Bo structural Engineering 54 Novembe and Construction TE Which of the following most neatly gives the required tension steel area in square millimeter, ‘A 3900 © 3610 B 3750 D, 3860 15, Which of the following most nearly gives the required number of 25-mm tension bars AB ce B? Be Situation 6 - A propped beam is as shown in Figure AN-16. ‘The moment ‘applied at the simple supported end causes a unitrotation at that end. 16. Which of the following most nearly gives the value of the moment M KiloNewton-Me A. 670 C700 B. 680) D. 710 17. Which of the following most nearly gives the reaction at the simple support in KiloNewton A120 cm B, 140 ©. 155 18, Which of the following mast nearly gives the moment at the ficed end in KiloNewton- Meter. A, 300, B 330, 350 D, 380, Stew 7-8 pot hn bm bl out th Ds ighways The approved agency esta results of responsive bids are ox follows ‘Bidder A= P350234451.98 Bidder 8 ~PL0338,768 12 Bidder C ~ PAS\218.557 98 Bidder D — F569 122,97 44 Bidder E — P2B4 75842654 ‘The implementing rules and regulations of PD. 1594 sate that ro award of Contract shall be made to 4 bidder whose bid price io higher than the Sllowable government estimate (AGE) or the Approved Agency Estimate (AE), whichever is higher or ower than 70% of the AGE. The allowable inment estimate (IGE) is defined as one half the sum of the AE and EE average of all eoporsive buds, For the purposes of determining the average of responsive bids, bids higher than 12D%of the AE or fower than GOs al the AE shall not be consldered. No negetation wil be allowed to bring dowathe bid to the lvel ofthe AAE/ AGE 19, Which of the following most nearly pives the masimomn bid price forthe martment ef Public Works and JAE) is 500 milion pesos. The rojec in pesos em 508 0000 © Pss0.0000000 3B. Pé00;00,000, D. Pas0.00000 20. Which of the following most nearly gives the minimum bid price for the Pk Past 00 000 & F300.000000 8) Pas0,000,000, 1. Pawn 000 , Civil Engineering Reference Vol. 2 5s 21. Which ofthe following gives the bidder to which the award Gan be made. A Bidder D © Bidder A B. Bilder C D. Bidder E Situation 8 — A circular timber beam 250 mallimeters in diameter hs a simple ‘pan of 4m, The beam carries a uniformly distributed load of 1» (2N/mn) ineluding its own weight. ‘The allowable stresses are 18 MPa for bending tral? Mbs for seas pace t grain Allowable dfaction i 1/240 of ht Span length. = 6000 Mia, Which of the following most measly gives the value of w eo that the allowable bending strest sell nat be exteeded, Hink Convert the irculat Section to square tection having the samme ates, ‘AIG ci wt Ba Which of the following most nearly gives the value of w so that the oeabl: shecang dee wil ite eee au C10 fe) 5 iso Z. Which of the folowing most neariy gives the value of W so that the Allowable defection wif notbe exceeded ate cs re) Bia Sua 49 ~ A simply supparted! steel beam 6 m lor 82 kN/m and an axial compressive force of 320. ‘heel section iss follows: ‘Area, A= 14700 mi Section Modulus, §, = 1921 « 103 mm? Flange width, y = 280 rim lange thickness, y= 16mm Overall depth, d= 390mm Web thickness, = 19mm According to Section 46.1 of the NSCP, for members subject to axial sompression and bending, Ze + Li. compression ard bending, GlE= + 1 f= computed axial stress, MPa Fy vied strength of steel = 248 MPa {fim computed Bending stress, MPa F. allowable bending stress = 0.66F, 25. Which of tse following mest nearly gives the computed axial siress in the ‘beam dite to aia force alone acting on the beam, in MegaPascals. te be B77 gives the computed bending stress in 26. Which of the following most ne the beam due to the uniform load afone acting on the beam, in MegaPascals, C7 carries a uniform load of ‘The properties of the AL 64 B Bt ds 27. Which of the following most nearly gives the value the interaction equation, A. 05 vere. o7 = Bod B. 06 56 November 2000 Figure: L-5m = 200000 He 125s 10 ast Figure AN-16 ENDS ‘ANSWERS: tC 6A WC MA RA BO 2D 70 25 me BA m0 Sk AC BC mB BO 26 45 MB MB MO 5D WA GA MC BA Strucsural Enginesting ‘and Construction 20" oiasa s Figure ME-12 eu | 37 eS T Solutions to November 2000 Examination £2 Situation 1.4 to 3) ‘AAE = P500,000,000.00 120% of P500}000,000 = 00,000,000 60% of P500,000,000 = 300,000,000 ‘Therelore, only bidders A Cand D shal be considered as responsive bidders ‘Average of responsive bids: 550,204 ASL98 + 454,218,557.98 + 380,122.897.44 Averages SSO SSAGES IN SSS eee Average PS64,525,20247 AGE = “fAAE+ Average of responsive bids) ‘AGE = {500,000,000 + 464,525, 300.47) = Pas2.262,651.23 {Situation 2 (4 to 6) 20K ou Fixecl-end Moments: Fev = BE = OO" 2 4sten 2 a2 EMaa= nor = FEM OOF = rae FeMyc= 2h = 2008) - 54n-m 18 FEMcs= 5 km FEMcp = uae - soy =356kN-m e e FEMpc = -28 fe aca)? Nam Beam Stifness ‘Assume = 12 Kane lan/Lan= 12/6 =2 Modified Kas = 20/4) =15 Kec~ 12/4=3 Kep= 12/6 Modified Kes = 2(3/4) = 15 Sectnral Baginceriog Civil Engineering 58 ines Seetrenen Reference Vol. 2 ae Distribution factors | Ssitustion ¢ tote 23) DEwe=1 " DFa=15/(.5+5) rez0n [ DFac=3/ 0543) DFce= 3/1543) =2/3 Deo = 157 (15 +9) =°1/3 Foote dagram es a= fit anos Aeleatiectirn ‘ouren ‘Yeloaty tie aag'am, Moment at B =-12.87 KN-m , ‘The given graph is that of Force (P) versus time, But since acceleration (3) een) = Fotve / Mass, the acceleration-time diagram can easily be constructed ley as shown above. FaiakN The area under the acceleration-time curve between any two points -0 represents the change in velocity between the points. Since the block = starts from rest, the initial velocity is zero as shown in the Velocity-time ae | diagram, The area under the -time curve represents the distance EFy=0 traveled, Ryy= F=120kN ‘As shown in the diagrams: Ray | Acceleration dusting the first two seconcls = 40 mys? pee Velocity after five seconds = 120 7 Re \fioay + (20)? = 1562 kN | Ry . 10 PF _ Distance traveled tano= Rav = 129 ety eens 5 = ¥{2)(80) +80(2) + (2/3)(2)¢120 - 8a}+ 12041) B= 180" + 6= 23039° 5= 413.33 m + Serechirel Rnglacesting 0.85 since fis less than 30 MPa oO} 903199 312612 + 600) Passe = 0.75 py = 0.75(0.05199) = 0.02309 Aye Pasi = 0.02399(300)(550) = $959 min? S- = 5 Nm P= 0.9, (300) 0)? = 463.5 5 108 < pmax (singly reinforced only) 0.0227 (300)(550) = 9745.5 mum? cD Situation 6 16 to 18) * moment in KN: = reaction in kN 00,0005 10") 1000 = 6M - 18 3M - 9F = 500 0) aya = Ze(Areaas x9) =0 = M(G)(3) ~H(ONRNA) ~ 0 E Areas Civil Engineering Reference Vol. 2 s1 —— Eg.) 3(GR) = 9R = 500; = 186.67 EN M = (166.67) = 666.67 kN-m = 6R = 666.67 ~6(166.67) = -353.5KN-m Ma {© Situation 7 (19 to 21) ‘AAE = P500.000,000.00 120% sf 500,000,000 = P600,000,000 ‘Minimum bie price = 60% of 500,000,000 = 300,000,000 Therefore, only bidders A, C, and D shall be considered as responsive Bidders. Maximum bid price: Berespaclteeinlee $+ 89227 4 (AE + Average of responsive bids) (500,000,000 + 464 525,502.47) = Pas? 262651,29 70% pl AGE = P397,583,855.86 According to the rules and regulation, no award of conteact shall be male to a bidder whose bid price is higher than either the AAE or AGE, (300,000,000) or whose bid is lower than 70% of AGE (P337,583,859 86). Among the responsive bidders, the award cant be madé to bidder A. It may either be awarded to bidders € or D, and among the two, Bidder D is mote desirable, G2 Situation 3 (221024) Bending: Equivalent square section: (b= d= x) 22 $Q50);1= 221.6mm sa Mg i” mea) -M=22,646,041.086 N-mim = 32.646 kNsm wid _ wld 7 8 = 32.646; = 16.32 N/m oP AOD 62 Now VO. AY or circular section 2 = 294524.3.N = 294.924 KN * BaiaS0y V=Reaction= = = wo 473 KN {D Situation 9 @5 to 27) Axial stress: P Uendingstress: 326) Rawat Tate M= 14 kitem M lax 0" 5 eae fo= 74.96 MPa {nveraction equation al, Ag coy Traces)” O.86¢ ‘and Conscraction , Civil Engineering 63 ‘Reference Vol. 2 ‘Seat Nos CIVIL ENGINEER Licensure Examination Friday, May 11, 2004 8:00 a.m. - 01:00 p.m. MATHEMATICS, SURVEYING, AND TRANSPORTATION ENG'G, SETA INSTRUCTION: Select the correct answer for each of the following, questions. Mark any one answer for each item by shading the box conesponding to Whe leer af yourchoice on the answer sheet provided STRICTLY NOERASURES ALLOWED. Use pencil o. 2 ony, MULTIPLE CHOICE 1. Acar travelling al 80 kph ona 4% upgrade suddenly applies a brake. I the toetiiend of fiaion berets oe cr and te peed oO oe ana the car rave after applying the brake? 445m cam Bam D.éim 2. A truncated prism having a square base hata volume of 1000 cubic meters The height of the prin ok cach comer i respectively rn 7a lay and 10m What the area of the base? A. 117.65 me? c B.12521 lolograsn measure éf\ em and &3 cm and the shorter ermine’ the smallest interior angle af” the 34.32 mt parallelogram, A Bar © s945° B. 3027" D, 1265° 4 Water flows at the rate of 16 m#/min in conical tank 12 m diameter on top fund 24m deep, How fast is the water surface rising when the water is 12 m deep in the tank, A 0251 m/min © 0828 m/min B. 0712 m/min D. 0.566 m/min 5 What is the period of the graph y=sin x? At Con B. 3a/2 D. x/2 6 An openctop cylindrical tank is made of a metal sheet having. an area of 4882 square meter. Irthe diameter is 2/3 the height, what is the height of the lank? A32m 423m B 243m D. 523m 7 An equipment having a first cost of P450,000 has a life expectancy of 10 Yeurs with a final salvage value of P80,000, Using the double declining lane method, what isis book valucafter 6 years? A. PLIZ 958.80 . PI84.320.00 B, PI47,A5600 1B. Pxsaaoo.00 64 u 15, 16, 18. 1. ‘Mamemadics, Surveying, and May 2002 ‘Transportation Engineering Find the effective aie On nominal rae of 85% compounded continously crs © bare 3 8am 3. iz iow faris the line Sx -y +7 Ofrom the paint (2,8)? a © S21 Boiss D.SA7 ‘A man walks froai his house to the office. If he leave at 8:00 ofeock and tvallcat the rate of 2 kph, he will arrive 3 minutes esrier, but if he leave at 8:30 and walk at 3 ph, he will arrive 6 minutes late, What time should he arsiven the office ‘A. 2:06 0clock © 854 o'dock B. s320‘clock 1, 843 o'lock Find the area bounded by the curve y= 4sin x and the X-axis from n/3 tora ‘A. $ square units C. Sequare units vt zaguare ats i, paguare unis Which of the following is nat aprime number? A108 . C7 107 De The offset distance from PC to PT of a simple curve is8.m. ifthe angle of intersection of the curve is 2, what is the radius of the curve? A187 m 15398:m B. 15265 m D. 136m Determine the equal payment series future worth factor of an annuity of 15,000 per yeat for 25 years at 18% interest annually ‘A. 0786 C326 B 1389) B. 167.21 Determine the volume of a regular tetrahedron whose side is 3 m, ‘A, 3182 C. 5321 ms B 2.983 mi? D. Li19m ‘The area in the second and third quadrants of the curve x2 + yi ~ 9 = Oi revolved about the line x~3~0, Find the volume genecated. ‘A, 534.54 cubic units ©. 37958 cubic units B. 11297 cubic units D. 27434 cubic units ‘The chords of the ellipse 4x2 + 9y* = 14 having equal slopes of ¥% is bisected Ty its diameter. What is the equation of the diameter? A 18x -25y 20 © 2x +17y=0 B, tex +27y=0 D. 14x -31y =0 ‘With the transit at point A and line of sight horizontal, the stadia intercept at Bis 0.6 m. If the stadia interval factor is 99.96 and the stadia constant is 103, find the distance AB. ‘A, 54210, © 732m % 387m D. 60.28 m. A compound curve has the following properties: hae Length of longchrs rom PC to PCG," 288.98 m b= oe Length af long chord from PCC to PT, Ly= 178.23 Find the length of chord from PC to PT. A. 40215 m © 47635 m B 37654 m Di. 234.76 m Civil Engineering Reference Vol. 2 20. A circle of radius 9 cm triangle whose area is 4823 square cm. If one side of the triangle measure 18 cm, determine length o the shortest side of the tangle, Sa nes A 437m © 564m. hen B 234m in « hydrographic survey, a staff gage reading of 8.15 m was observed at the instant the depts ofthe sounding was 17.6.3. The ero mark ofthe salt ‘gage is at elevation 148.2 m. Find the elevation of the point where the reumseribed about x Sounding wr made, 38.25 m © 136.25m B. 13873 D175m, 22. The sum of the first m terms of a series is" 6. Find the fifth term of the ‘Aas © 1458 wo nee D. 1426 hwo aides ofa tranigle measure 14.cm each. What is the maximum possible area ofthe triangle? on Lag A. Item! © Stem _B. 76cmnt D. ene 24 alone the tof (2-8) / 6-2) as xapproaches2 Be ba 25. A body moves such that its acceleration as a function of time is a = 2+ 12, Where “is in minutes and “a” is in:m/mint. Its velocity after 1 minute is 11in/min. Find its velocity after 2 minutes. AL St m/min C sSm/min ve, Benin D. 18im/anin 26 A block weighing 200 N in placed 25 m from the center of a circular rotating platform having a radius of 4 m, If the platform is rotated at 15 ‘pm, what miniovum cosificient of friction between the Block and the platform to prevent the block from sliding? ‘A 0432 C0629 BoB D. 086s 27. A 45cm x 45cm square plate ABCD of uniform thickness is supported by tice verticl strings, The first string is located at B, the second is Iocated 15 {in fom A along side AD, and th thie is at the point midway of side CD What percent of hs lon cae by te ata cco sap? ht pe : peck B, 60% ce 5 20% Mathematics, surveying, and Ree Aa Civil Engineeris 66 May 2001 Transportation Engineering | eeterente WoL ae 67 28. A closed traverse has the following data Solutions to May 2001 Examination Distance, T Necoere 2520 SEIPE 35.00m a1 S720rW 795m eae 30° een GNSAY — 2GBIYOS+ HONG. O2 Find the length of side EA. A. 23m © 549m B 27m D. 316m ed 28. The longituclinal ground profile and the gradeline shows thatthe length of 1010 alia Sab whe the length of ils 740. The wid ofthe zoadbed i 10 za ; 1m for both cut and fill The profile arcas between the groundline and the y= 107.65 gradeline, which are parallel, are 4800 m? for cut and 5829 i? for fill. Pind the difference between the volume of fil and the volume of cut in cubic meters ifthe side slopes are 15:1 for cutand 2:1 for fll ms A 43 © 40,37 mo From the figure, the smallest Bs Di, 56.208-2e intenerangets. 30. Determine the area enclosed by the curve 32-4 y+ Sr dy- 610, A128 Cares By eaaais tie Baers eee! 42 © 68+ 432 -2(68)(89) e050 cos 6 = 0.36365 “END 6 ma | ANSWERS: is = rc aC cone m [aa en fhe & ; 58 88 ono m 75 8 A Mo BD [ke be gt Og é MY ZOOL From the graph shown, the curve completes ane eyelets = ~ Aaa Advent = $D8+ wD Hi 48.82 = $.QH/3)+ xQH/3)H: Ht FC = Paso,000.00: SV = P0,000.00 n= 10years = (am BY Mathematies, Surveying, and Transportation Engineering Reference Vol. 2 69 | a. Let tbe the schedoled time of arrival and 5 be the distance: rom his home’ to office. Leaving a ‘O0and Lking 2 kph, he will arrive 3 minutes earlier Arrival time = {- 3/60 fal time ~ departure time arrival tiene - departure time Travel time = (1 +6/60)-85~ 1-84 S=pi= 3-84) Papo 72,000:00 57.400 00 0.00 26,564.00 70 —— Seat us FR= e'=1- eth -1~0,08872~ 8.872% | avait S=3t-32 > Q) |_END of yeur PSG, SI | asso 2t- 16.1 ~ 34-252 wats sock ee on an [Ps ie ‘A prime number is 4 suinber that has no other factor other than ane (1) and itelf. “Among the choices 77 is not a prime number because itis factorable by 1.7, 11, and 77 re ints atiemaste, soeviotons bad 70 May 2001 Trenopertation Sapinecring ie ae aioe A R-8 05 20° ‘ Ro Rooe itt R= 1A265.m r we SERN ee re etl ermal leet peter ip AEDT rye pay EEL scat te pect ra beri: (L911 +010) =1 _ 5e5 605 ms A tetrahedron is a polyhedron with four faces From the tet pedron shown: Civil Engineering Reference Val. 2 ais The curvex?+ y= 9= 0is circle with center at (0, 0) and radius of 3 By the second proposition of Pappu Ve Ax 2k (3)? =4.50 R-3+0~3+ Rea2m aR Ve 45a = 2n(4.2782) = 37958 cubic units anes 9ye= 144 Disferentiate ryith respect tox Sr+18yy'=0 ety Bx + 18yG/4)= 0 16x42 y=0 S++9 D = (99.36)(0.6) + 03= 60.276m From the figure shown: = 180"= 16" -12 = 152 [= 23598 m 178.23 m Mathematics, Surveying, and 7 May 2002 “Transportation Engineering iba The diameter of the circle is 18 em, hence one side of the triangle is the diameter of the circle and by principle this triangles a RIGHT TRIANGLE 8 ng Nab 95646 (1) erea ie Besa 3.) Squacing both sides of equation (1) b= 93045316 (024-09 B= 90085316 be =324 b+ 99045316 =0 ? pew EYES ADIROESG) as 7) = 5:643.em = 17.092cm Thus, the shortest side is 5.64 em on Elevation = 1482 8.15 ~ 17.6 Elevation = 138.75 m an term = Sum of frst vers = Sum af first (terms 4 S,— Set is~ [36°2-6]-[ 34-46] = 1458 an A= =e anh be tens arbre | Mrtbex 14, os, oat Ste mero s-a-05e s-c~M4-05r sb 05x Ax fuerosnj@5n 0504-05) A= forsx (196-0257) « {0254196 025%) aa 0.25(392r—2) dx 2,f0.25(1964 —0 25%! 392, x= 19.7989 Another Solution: = 1A(14)(04) sin @ = 98 sin 6 = 98.c080=0 4 = [080 = 0,090" A= 98 sin 90"= 98 em? Another Solution: The figure below shows the possible triangles: Obviously, the area ofthe triangle is maximum when "hi" is maxis and is equal to 14, A= Stata) = 98 iy WADED = lim +2)=2+2~ “Another Solution: Try x= 1989 ind | 1.999657 r=? 19999=2 =4 = 2124 do= 2+ 129 at Pe fo |= [anee M1 = 2(2=1)+ 6(22 12), v= 31.mymmin = erat surveying, and Civil. 74 May ZOOL — Tine 4000 (Turbulent) as Q=Av B5~Ag) A= 75 (= 697m? Anhi+d~697 9 (i) pe dpeeg = 3ft/s= 0.915 m/s sis = —! pavwo.0002)"%;R =0.935 m dois B97, ay = 0556 b+ 22d = 720-28 > @) Substitute bin (2) (1) (728-2828) +s = 697 7294-1828 = 657 1928 -7.20d+697~0 d=159 m= 522 feet ay Q= Ave ALRYS12 a2 foet=061m aA ADA Dos feo 4 a 0.1525 m 1 ' = $061)? 1 (0.1525) (00025) = 0.321 m/s = 3 (srt 7 casas cnonasy i Q- 113245 Note The Manning's equation in English units fs: R952, where Ris feet HL = 375m fag = ¥ =10 1658-85021 =1TA Say atop Po igois.e9ibe ay 1 o058 TED otoss ioganke Ae ae anita an 25,133(8.302) ee 1698-04907 =1.5699 [2Me= 0} hor) 13,019,89(1.5633) = F(4) P= 50685 Tbe Plasticity Index, P The tiga limit of the soll the tmoistuve content comesponding © Bmmeone pertiraton Fromihe & i 5 s ‘0 = oS pusricenner Test [fens | weer] | Tetsz | cetem | ee | Se me | a | nam | maa | ae | mew =e | oe | om ms | ma | ae to ina | som = (2.08 + 21.06-+ 2099 + 21.08 + 19.93) / 5 =n01% PI =52% -21.01% = 30.99% an Mg = (12 slug)(@076 (7s) 3076 slug ft/s? 369.12 Ibs VA2AYS AVS) ~ 364. 5KN Moo} FG.) = 36427(24) 2302 KN Cheek for p: ip.) (1.295) = 2302 73 Pa sximum p= 65(1.2) = 78 kPa (OK) Fo) T+ Rh Tr 364.5 - 2302-14. 3KN From the diagram shown; 0) = tam! (4/1) = 7596" 0) = tami (5/1) = 78.65" {EF -0) F, sin'01= Fi sin Os y= 0.989 F, EFu= 0} F, cos 0s Frcos 02 F; cos 75.96° + 0.999 F, cos 78,69" = 1343, Fi, = 307.6N (compression) = MSA) = 210% =m May 2001. Geotechnical Engineering, Ou For theinclined ‘gage and with the rain falling vertically, the horizontal Projection of arta Satching the rain A= A, cos When 0 = 10" Am A,cos 10° A, = 0.985 Ay Ay= 98.5% A) (cage tne Thus the gage catches 1.55 less then when itis vertical (as given). When 0 = 20", A;= 0.94 A,= 94% A, ‘Thus, the atch is 98% of the vertical gage Seat No: —__ CIVIL ENGINEER Licensure Examination ‘Thursday, May 10, 2001 05:00 am, - 01:00 p.m, STRUCTURAL ENGINEERING AND CONSTRUCTION INSTRUCTION: Select the correct answer for each of the following questions ‘onky_ane answer for each item by shading the box corresponding tothe better Pereira teste does peice ICTLY NO ERASURES ALLOWED. Use pencil no, 2 oaly SETA MULTIPLE CHOICE Situation 1 = A contractor was awarded » four-year contract for truc services. He wishes to purchase pick-up truck worth P1,000,000 each, The truck is to be used for 20,000 km per year. The estimated fuel consumption per year is F125,000. The dealer af the truck agrees to handle al Hhirfonance cost for four years at an additional cost of P200,000,. Maney is worth 15% per year. Salvage value of the truck ater four years is P300,000, 1. Which of the following gives the annual cost of the truck not including the ihaintenance cost. ‘A. P345,872.23 ©. Pare 994 B PAS7 19845, D. Pa5.18575 2. Which of the following gives the annual cost of maintenance alone. ‘A. 70,053.07 ©. P62875.45 B Pod.349.23 D. P51.38265 3. Which ofthe following gives the total anral cost of the truck A. Pa32.98212 C. PBs 238.82 B, P512/893.42 D, 62367233 Situation 2 ~The beam shown in Figure AN-26 is 150 mm wide and 500 mm deep. The allowable bending stress on extreme fiber is 8 MPa. [fx 2m: 4. Which of the following gives the maximam moment that the beam can Tesist in KiloNewtonsmeter. ‘A. 1550, c 750 B 1800 D. 3400 5, Which of the following gives the maximum value of the uniform load w in KileNewian per meter. cB i Bis D. 215 6 Which of the fottowing gives maximum value of P, in KiloNewton. AL 125 Cus B 200 D180 Situation 3 A square tied column is to carry an axial dead load of 500 KN and fan axial live load of 750 KN. Assume f<= 21 MPa and fj ~ 273 MPa. Use 22- sun bars and reinforcement ratio of 2%, 88 May 2001 ‘and construction 7. Which of the following gives factored lowd P that the colunan wil carry, in KileNewtora, A. 1234 C1975 B 1845 D. 2002 8. Which of the following gives the required column dimension in mm. A 254 C 367 B. 392 D. az 9. Which of the following gives required number of 20-mm bays AS co B7 D109 Sitwation 4 - A reinforced conerete beam having a width of 280 mm and an effective depth of 520 mm is reinforced for tension only with 5-28 mm bars. Gonerte tenth f= 128 MPa f= 21 Mia, fo= 048 n=8. Use working surength design 10, Which ofthe following gives location of the meutval axis from the outermost compression concrete A237 cus B 198 D. 276 11, Which ofthe following statements is tre forthe given bear 4s The section complies with the code Hh The tension steel yields first . Theconcrete yields First DD. Thestee and concrete yields simultaneously 12 Which ‘of the following gives the moment capacity of the beam’ in KiloNewton-meter, A 1 138 B. 167, DIM Situation § ~ A certain equipment has a first cost of P900,000, life of & years, and ‘salvage value of 200,000, 18. Which of the follawing gives the book value of the equipment after 5 years using the straight-line method. ‘A> Pai 800.00 C. P34s,g00.00 B P462.500.00 . P#98,200,00 14. Which of the following gives the back vahie of the equipment after 5 years ‘using the sum-of-the-year’s-digit method, ‘A P389325.00 CP 267,329.00 B Panz345.00 D. P316,667.00 15. Which of the following gives the book value of the equipment after 5 years using the double declining balance method, ‘A. 325,982.00, C. Pas3,235.00 B. P245,389.00 D, Pan3is74.22 Situation 6 ~The magnitude ofa force is 1000 KN and passes through the origin and point («= y= 3.2" 4). 16. Which of the following gives the Xccomponent ofthe force in KiloNewton. Aaa Ete & ass5 D. 3242 Civil Engineering Reference Vol. 2 so 17, Which of the folowing gives the ¥-componant of the fore In KIbNeWton Ate ee 71 B, gn D627 18, Which ofthe following gves theZ-component ofthe force in KiloNewton A459 27 B78 D.sz4 ation 7 = For the truss shown in Figure AN-13, the product of the cross: sectional area and modulus of elasticity is 1,000,000 Newton. 19. Which of the following gives the ail forceon member BC due to the given loads, in Newton, A. 500 ¢. 400 B. 200 D: 600 20. Which of the following gives the axial force on member BC due to a vertical tanit oad at C Al © 125 B 075 D, Es 21. Which of the following gives the vertical deflection at Cin mm, A. 535 © 845 B 675 5. 325 Situation 8 = The angular section in Figure ST-17 is welded to a 12 mm gusset Plate. Both materials ore 436 steel with F = 248 Pa. The allowable tenes stress is O6F, The weld ts FSO Electrode with F, = 550 MPa. Allowable Shearing stress forthe weld is O3F,, Properties of L 180 x 90 12: ‘Ara=25tmm? | 89) mm y= Simm t=12mm H=150 mm, 22. Which of the following gives the value of 4 # bin millimeters A. 267 C ae B. 395 D, 351 23. Which of the following gives the value of bin millimeters. ‘A. 267 C17 B22 D. 296 24, Which of the following gives the value of Pin KiloNewton A 409 C. 567 C 436 D. 365 Situation 9 - For the beam losded as shown in Figure AN-36. 25. ‘Which of the follwing gives the resultant of loads in KloNewton A 26. Which of the following gives the location of the resultant load from the left ‘support in meters. A285 3.95 B15 D. 375 27. Which of the following gives the reaction at the left support in KiloNewton, 430 C25 Bas D. 35 90 May 2001 ‘and Constroction ‘Situation 10 — A simply supported beam has the cross-section shown in Figure ST.03, The section is 1721 147 A96 steel, compact section with F, = 248 MPa and reintorced with 12 mm x 360 mm A36 steel plate at the top and bottom. The beam is laterally supported over the entire span of 6 m and ‘arries 1 uniformly digtributed Foad of 360 kN /m including its own weight i allowable ending stress for Interally supported compact section is O.66F, and allowable shearing stress i Ot J Allowable deflection 1s 1/360 of the span ‘The properties of W721 « 47 areas folloms = 540 mm t= 18mm 1o=1 51 10¢mm* 28. Which of the following gives the section medlulus of the section in m?, ‘A, 00076 C, 0.0054 8, 0.0085 D, 0.0058 2. Which of the following gives maximum deflection of the bearn in mum. A qd © 5g c 137 Ds 50. Which of the following statements is true for the gi ‘A. The section is adequate for flexure only B. Thesection is adequate for shear and deflection only C. The section is adequate for deflection only D, The section is not adequate for flexure, shear, and deflection section Figure AN-13, Pid IDM SOK iS SiN Lm! Sms ism! Lm ‘Figure AN-36 Serueroraltacincering 92 May 2001 EE ________— ie Solutions to May 2001 Examination ‘© Situation 1 (1 to 3) ‘Ant cot ofthe rc excluding maintenance) Annual cost = Annual interest-on investment + OM + annual depreciation Annual interest on investment * 1,000,000(0.15) = P150,000 (OM = P125 00 (operation cost “gas") repreciaion = (FC=5¥)! _ (14000000 3000905) acres aay" (+015) =1 Annual depreciation = P140,185,75 ‘Annual cost = 150,000 + 125,000 + 140,185.75 = Pe15,185.75 Annual cost of raittenance: C ,000{0.15)(1 + 0.15)* Ace ape EOOROENA SOI = most ‘Total ans cost = 415,185.75 + 70,053.07 = PA85.238.82 Civit Engineering Reference Vol. 2 Azmust be -3650 that the moment at C becomes -18 kN-m- Ar™ s(3) =-36 g0-2KN; Thus, P= 6+12=18kN ID Situation 3 (709) Poa LA DL +17 LL 1.4600) +1.7(750) =1978 kN Py = 08 § [0.85 fe (Ay— Ani) + fy Andi Aw = 0.02 Ap 1975 » 1000 ~ G(0 7(085(21)(dy = 0.02 A,) + 275)(002.A,)] 3526785 7 = 72.995 Ay A, = 153,385 2mm! ‘Ags f= 153385 2 h= 391.6 mm An 0.024, = 0.02(153,385.2) = 5068 mm? $20) N= 3068; = 9.8 say 10 £2 Situation 4 (10 1013) 20m Am $5 A.= 3078.8 mm? nA,™ 27,709 mm Je 045(21) ‘f=9.45 MPa [Ey 0] 260(2(c72) = 27709(520~ 2) 140 2 + 27709 c- 14,408,680 = 0 = 236.8.mm cm kd KG520) = 236.8 = 04556; )=1-4/3 = 08482 Interms of concrete, the resisting moment is: Me Vapi jb di = ¥o(9-45)(0.8482)(0.4584)(280)(520)2 ‘M= 138184021 Nomi = 138.18 EN-m. Ine ana ‘ T= Aft ~ 9078 28) 95n2y00 «179817324 Nan Meweliinen eee Thus, the moment capacity is 138.18 KN-m From the results, the concrete yielded first and thus, it does not comply with the code. 94 May 2001 ‘GD Situation § 13 to 15) g= EC=SY _ 900/000 —200,000_ w e Dy = dx 5 = 87500(8) ~ P437,500.00 BV; = FC=D3 = 900,000 - 437,500 BY, = Pag2;500,00 87,500.00 Sum-of-the-year‘<-digit method: H(i m1) 3Sum Ds= (FCS) (14 8)= 36, oy onan -aminn S2O=5*1 eps BV = P316,666.67 Double declining balance me {n=8) “itp date= 2 BVatthe ping te your wane TaD “os BEGINNING of year EN the year T= FC = 900,000. P25 000.00. Por O00, Saas = 2 eee Tise0o8 | Pao sears as ae rassas | vines meee rousaues | PRA as S39 87S OORT 5 = Prana | Paras + [eee rasan | _ pono |= Ta a Fan [= Fano av om “hus the book valu after S years is PABA 765.625 Note Since the salvage value is 200,000, the minimum book value must bbe 200,000 and thus, the depreciation charge after 6 years must not ecard PI3.574.215 Civil Engineering Reference Vol. 2 95 ‘Using the formule: BVa~ re(s V4 =, 0(1 BVs™ P203,574.22 {Stanton 6161038) Force = 1000 kN through (0, 0, 0) and (2,3, 4) Le Venn) #s-y #2 be {O=9F Ga scot = Ei=Forex 2241 2 T wmv i F, = Force = 1000. TL Force x 2 = T 2 Situation 7 (19 10 21) Stress Atjoint C [Fv =0] Sec(4/5) = 400 Sr 500-N (compression) (oi- oF Sic 600 + 5003/5) Sac 900N (tension) Atjoint A (PFv= 0) Sm = 400 (iension) Stress due tounit load: Atjoint Ce [SFr 0) Usca/5) =1 Upc 1.25 (compression) [EFir= 0] Use™1.25(2/5) =0.75 (tension) {to actual loads: 96 Mey 2001 and Construction TAtypanea [Fy=0) Ug = (tension) al deflection at C a ge ee AE AE p + + 5 Tapocann Sav Uae ban + Sic Mac Lac * Sac Une Lae] 8 [aay MOOK Aoot + 50010753000» (500-1. 25)(5000)] 6=675mm 12 Situation § (20 24) Value af P. P=FixA=(OR)A P= (0.6(248)) 2751) P= 40518 8N Ped09.35kN Value of a+ P= 07071 LF, P=0,707(10) [0.30650] = 409,350 L=25imm=a+h (99) = SY); 0= O5151b 0% b= 351 O5151h + b= 351; b= 25.7 mm 22) Situation 9 (25 to 27) WKN ADKN 2OKN ISKN SKN ‘Resultant of loads: t P= 2)-10+30+15-5 A . F=50kN Location of resultant load leat isctisalusntisn oat rein ond Sal igmtismisnlis + 30(45) + 15(6)- 5075) xa375m Reaction at the left support: Ms =o) Ra(?-S) = 20(6)- 10(4.5) + 30(8) + 15(1.5); Ra ~ 25kN Civil Engineering Reference Vol. 2 (Situation 10 (28 to 30) Section madubus: To 1511 = 1os-92[360(12)9/12 + (360)(12)(286))] T= 2217,824,120 mmé = 1 2 2aramam C tost 5 =7596,278 mm! $= 0.007595 mt ‘Maximum deflection jm Se, __5(360)(6)' (1000)* ‘SSAET — 384(200,000)(2,217,821,120) 8= 137 mum Allowable deflection = 1/360 16.7 nui Allowable shearing stress = F, = 0.(248) = 99.2 MPa ‘Actual shearing stress: Vin Reaction = =) = 1080 kN 0801000) eNj8) ~ 107.14 MPa> F, (not adequate for shearing) p= tot.) 7,595,278 r= 2133 MPa> Fa (not adequate for bending) “Thay the selon is adequate fer deflection only 98 November 2001 —"tampaetennsputiang marcen cal a 99 Seat ‘Water flows fo a tank having the form of 8 fustum of a Hight realy cone. The tank isd m tall with upper radius of 13 mand the ever netics CIVIL ENGINEER Licensure Examination of m. When the water inthe tank i 12 m dcp, the aufoce tes stake Sstucday, November 17, 2001 08:00 a.m, - 01:00 pm fae of O12 m/s. Calculate he dichge of wate owing no the nh mys - foe © 00g MATHEMATICS, SURVEVING, AND TRANSPORTATIONENG'G, SETA 5 Bae Using a 254m tape, a square lot was measured and found to have an aren of WUCTION: Sclect the correct answer for each of the following, questions 1 hectare. Ifthe total error in areas 4.004 square meter showt, what & Bee (ates ie a ae tigate Es ie errr in sich tape lenge? of your choice on the answer sheet provided ‘A. 0.00 m too short C. 0.008m too tong SY NO ERASURES ALLOWED. Use pencil no. 2 only B, 0.008 m too short . 0.0 moo lone Triangle ABC has side b= 160 cm, C= 190 em, ond CS 190 cin, Paine D suLrirLe cHoice is along side AB and AD = 100 cm Point Eis alongside CA. Determine the length of AE ifthe atea of rangle ADE is /5 the areal tangle abe 1. Detsrmine the central angle of a 250'm simple curve ifthe nearest distance 1653 em © 1824cm ftom te curve to the point of intersection of te tangents is 18m & 1512 cm B1739cm Ao C36" ‘A house and lot costing P2 million was bought at a downpayment of oe B30" 500000 und PI mullion after one year. The remaining balance il be oa BA rereised curve has the following properties: at the end of the third year. I te Entorest vate is 245, compounded she Sof curve, Dy Soul, whats the requied payment” Central angle, i= 28 A. Phasz87600, © PLa7432600 Eek B. P768905.00 D, PIB721500 ea oo cee How many combinations consisting of {digi canbe made using the digits Staining of = 67 + 34523. from Oto Ditermine the stationing of PRC (point of reversed curvature), Use arc xz ca D420 A 67+ 905.23 r+ 4723 Evaluate the integra of xcos (4s) dr with lower imitof 0 and upper limit of ers 53023, D. 67s 680.23 ay B.Useate te center ofthe curve t+ yts Sut dy 61 =. A /8 < 1/16 paraes © 43) B ifs Ba/t6 a (43 D. 4,2) ‘An observer wishes to determine the height ofthe tower. He observed the 9 Datei the lat rectum ofthe following ell top of the tower from A and got an angle of elevation of 20" Tie then 4932 + Soy! + 56m - Togo = ‘walked 25 m cleser fo point B and observed the angle of elevation as 40" A163 3215 Pointe and 8 are atthe seme elevation, and oma dite line witk thetentor % 10285 D.oase Hos high isthe tower? 5. Artough tus an open top 030m by 6m and closed vertical ends which are Aaa ©4627 fuller! tangles 30 cra on each side. I filed with water to ball is B S732 D. 7ea2m nd the Volume ofthe water in su The ground makes a uniform slope of 5% from STA 12 + 180 wo STA 12+ nD © 07 240, rst 12 it te coer hah of the rondvray i614 m fil, AL the ts B08 ‘ ater station, the center heights 28m cat. Find the pradeof the finshed © Whatisthe present worth of a perpetuity of P10,000 annually is money is rou Hs compounded monthly? A ee ‘ano Pio9.000.00 B ex fox | Pa DPiossian2 The abserved interior angles ofa triangle and their corresponding number J Rkariateweling st » unilorm speed of 80. The driver saw a roadblock 3 of observations is as follows: fae th ot stepped onthe ake, canning the ca to decelerate ‘Comer “Angle No, of obeervation orm tn he dntane fom te roadblock oe point where - a 3 thecaesappedis 12m. Find xf the perception reaction time is 30, : aig e036 m CF 3 BSS m Bi. §7.65m Mathematics Surveying, and 100 November 2001 ‘Transportation engineerin Tietermine the mast probable value of angle C. (A. 75° 34/21" (C74? 48°56" B. 73° 54’ 52" D. 74°25 23" 17, The cost of producing a commodity consist of P00 ner unit for labor and inate cost and P2200 Perunit for other vacuble cos. The Fined cost per ‘month amounts to 700000. 1 the commodity is sold at P290.00 each, w! ‘ the break-even quantity? ‘A. 3200 < 3600 B 3500 D. 3400, 18 Determine the moment of inertia of the area bounded bythe curv the line x= 4 Gand the Kani, with respectto the Y-axis ‘A512 © 52 Ba D215 19, A car travels on a banked circular curve. The front wheels ofthe car are 15 ws apart and the cunve 1s banked such that the two wheels are 15 cm vertically apart. What should be the minimum radius of the curve so that there will be no lateral pressure on the car's wheels at speed of 62 kph A. 2675 m © 329m B, 3008 m D, 252m 20. The base radius of a certain solid is 20 cm. Find the volume of the solid in ce AC every culting plane perpendicular to a given diameter is an equilateral ay, mangle A 16543.29 © 1847521 B. 1945357 D. 1421398 At. Determine the aren bounded by the caren = 1/y,2r-y=d2= and the A. 2138 ©. 2324 B 2308 268 2 Arman walks from his house to the office. If he leave at 6:00 o'clock and walk at the rate of 2 kph, he will arrive 3 minutes earlier, but if he leave at 620 and walk at 3 kph, he wall arive 6 minutes late. Find thedistance from his house to his office A21km, © 24k B18 kn D. 14 ken 2 Salve for the sum A, B, and C frum the following equations: 2A". 2B + 3C = 4 ‘A+ BB = 2C = 45 gh + 4B 4 3C = 2 h2 C5 Ba Da DE. A so steel ball of radius 3 cm is immersed in a cylindrical nk containing ‘iter o adeplls of 10 crn, Uthe water inthe tank rises 2.25 mn, what is the lamete ofthe tank? ‘A bem © 9m B dem D. cm 25, A circular platform of radius 2.m is revolved about its center atthe rate of 8 Fm What is the normal acceleration atthe exige ofthe platform, in m/s? ‘A 1S roa Biz Di tte722 meeeeyase 103 —— 25, The three cdes of a tangle measure 86cm, 18.cm, andl 24 cm What is the length ofthe median drawn the longest side to opposite vertex. ® 12395 cm ©. 1013 em B 2s em 3045 cm B.A cele af ads 9m ccc about a angle whose are is 823 are cm. If one sie of the triangle measure 18-cm, determine length the other side a ae % ‘437m C som 5 1664m D. 1709 m 28. “The distances from the focus to the vertices of an ellipse are 4 and 6 units, Determine the second eccentricity of the eli aes A004 C0503 B. 0.305, D, 002, 29, ‘The sum of two numbers is 5. What is the minimum sum of their cubes? A 9/3 Cc s/2 B S/4 D. 5/5 30, Water flows at the rate of 16'm!/min in conical tank 12m diameter on top and 24 m deep. When the wator in the tank is h meters deep, the surface is rising at the mate af 0566 m/min. Find the value of h. 10m Clm Bld. Dem “END 102 Novenbar 2001 i Solutions to November 2001 Examination aa ws os Mathematics, Surveying, and ‘Transportation Enginesring ‘The nearest distance'from the curve to PI is the external distance, E. E=18m,R= 350m Em R sec (1/2) -R 18 ~ 350 sec (1/2) ~350 sec (1/2) = 1.051428 = 1/cos (1/2) 0s (1/2) = 0.95109 [= 35.99" Stationing of PRC = Stationing of P:C.+ Le. dg 20 = NAH 60m Stationing of PRC = 67,345.23 + 160= 67 + 505.25 eeptert4y-el=0 (24 Be + 16)+ (y+ dy + 4) 61+ 16-44 Grats (y+ 2) = a1 ‘Center (,-2) 100s Sots <9 -656=0 See tS Sex stg vice tera)? , y-3)? = 8 BITTE a2 | 20° sane “1 Civil Engineering Reference Vol. 2 103 ‘The water will assume the shape of a teangular prism. ‘The volume is V= yx L, where Ay is the base area with the shape of equilateral triangle of side 0.3m. V= [540.15)(0015) sin 60°] x 6 = 0.0584 m? pad ERaganty = ERytay (ret = (1+ 010/12)2- 1; 7= 020871 10,000. 995,499.07 P qaou7t From A to Bi (t=3 see) te" 80kph~ 22727 m/s 22.2220) 6.57, From tC: (a=-10 m/s%) vieagt2as OF= 22.2223 + 2(-10) 5, y= 24.69 m em S,+ 5:4 12 66.67 +24,694 12=103.96m, 2. 03 eae x= 0125h Reite + 0.1254 ya Bh Va Bie Ree eat ) va Mears Sa cansicyomy . v= foonseasne-+ 037sh +3] 3 v= aonseass arses a 105 gauso ou a2 mas Thus, the tape is 0.008 m foo long 3 Asor® = Act oem =A *A(LOOKAE) arb — 2 {16019 sip AB= 182.4cm Balance 2,000,000 - 509,000 Balance = P1,500,000, Py=7,000,000KP/F, m) u p= 2,000,000 fal Gray Py 797,198.88 Pe=nP/E an) 797,193.88 + 0.506631 x = 1,500,000 x= PI,367,21500 Number of combinations = C{10, 4) = foes from fod mae~ fia wex dumdsy — domcosr dy, ain = Mathennatics, Surveying, an 106 November 2002 “Transportation Enginecring 1 jin Ax ~ [Fens = [Mirai sxe Dncosax]|” : B mase [Basin ore thease | 5 mas 1 1 1 n140"+ cos tan] [1o)sind+ Aecoser 2 16 ow fo 16 | 8180" - 40° = 140" = 180° - 30° -140° resr In Triangle ABO. __3 in 140° ~ in 10° x=92542m Intriangle ACO: sin30°= = 92542 sin 20°= 4627 m mas eos), =am FEET yr3-28-02m ‘Grade of finistved road = Grade of finished road =-2% Using the formula: +H ee, Same Gt GA amc @- 5 Gn 097 O05 — Gna pena oe Gent = 0.02 2% (Civil Engineering ‘Reference Vol. 2 107 —— erry ‘The sum of interior angles ofa triangle is 180". Surm of intorior amples = 41" + 65" + 75° = 181° Error = 181° - 180"= 1° = 60° ‘The weight of error of each observation is K/N, where K can be any. constant and N is the number of observation, The ideal value of K i the LCM af all the number of observations, ie. K= 30, (6/2560) = 1385" (5/2560 y= Me (15/26)(60) = 34.67 ‘The error must be subtracted to the observed values since the sum of the observed values is more than 180°, Most probable value of angle C= 75*-34.61' = 74° 25" 23.4" on To break-even, Cost Revenue feaN= PN 700,000 + (65 + 25)N = 290N; N= 3500 4 fa dA (wu ~) dr eae e aa dA = (32/4 - 0) da 108 November 2001 Diy ‘Transportation ‘Such that there -will be no lateral pressure ‘on the wheel, tan 8 ri p= 62kph= 17222 m/s sin = 015/150-5737 — pis.m wat ogik tan 5739" = R=300.83m 120 i ved fay aa. ead gm 1] ArzAa=0) (40) (4) sin @28x8 ve 2+ -4(692.8208) + 0) Vo 18,475.21 cc Point ai intersection: Be-yntiy= 2 X= Wyorsy=1 = O77 dey = 1402 Paqoori, 14142) Ar= ¥a{0-7071)0.4142) = 0.5 (waar y AD civil Reference Vol. 2 Sao y= In(@)=In(07071) = 2158 A= Ai + An = 2.638 sq, units Let (be the scheduled time of arrival and S be the distance from his home to office, Leaving at 8.00 and walking 2 kph, he will arrive 3 minutes earlier. Se orm 2{(t~8)~ 3/60 S-21-16/10 (1) Leaving at 830 and wailing 3 kph, he will arrive 6 minutes Late, $=3]{t-85) + 6/60] Smt 125/5 >@ [s~5] 2=161/10= 3¢~ 126/5; = 91 In Eq, (2) $= 3(9:1)- 126/5= 21 km By Cramer's rule: 2-2 a}2— D=|1 3-2) 1 3|=(1g+12+134-(27-16-6) a4 als 4 “7 m4 -2 9f m -2 s 3-215 3| =2 4 al-2 4 Nig = [216-8 - 180 [18-192 +50) =148 Na. 146 aw Na HB Aare 2 me alo al Ne=|1 -15 -2] 1 -15] a -2 3/3 -2| [90-144 ~6] ~[-135 + 84-72) =-185 Mathematics, Surveying, and 110 November 2001 Transportation Enginecting z-2 la -2 Ne= |i 3 -15]1 | Ba 12+ 90+ 96) - [216 -120 #4] = 74 ~Aun ms 26 a Sum-A+B+C=4-5+2-1 Vigbee Vag $8 @) = HOP 225) D=Sem = (4/15)? Q)= 1404 nye? Solving for 0 by cosine Iaw 8 in triangle ABC: Daim 361+ 16% -2(36)(18) cos 036.336" BS 18 In triangle BCD: Ris 1 ¢ 16-2(18K18) con 96396 < Me + 1ii The diameter of the circle is 18 em, hence one side of the triangle is the diameter of the circle and by principle this triangle is a RIGHT TRIANGLE. Ap=thad B= tab b= 96.45, +0) ae meie @=34-b 9 (2) ‘Squaring both sides of equation (1) 2 = 9304 5316 G24 - 5 B= a3 916 1-324 i + 9004 5316~0 yo. Suz (3297 =ayesonsai6) 2g) Pa31BMs, b= 5643. cm a~17082cn “Thus, the shortest side is 8.64 em wm6r dees a-en4d S-cmdent @=pta ae Seb 1 b= 4.999 Second eccentricity, & = © ery = E 1 5558 és = 02041 = 7 OTHER ELEMENTS OF ELLIPSE Fret ecoiay = £ ripe Names, j= 2b ‘Angular cccentsicity, 112 Nowsmrlber 200i “tramporetion ingineering on Pats sdlyhe mar ects tee etyeSiyeS-x Semen pageliaae ASH) «501+ 36-263) =0 Bis spaandy=s/2 Sum = (8/2) + (5/2) = SA m30 V= turk 6 nie WV _ 54 16= 35 je (0566 ray him Civil Engineering Reference Vol. 2 113 SeatNoz CIVIL ENGINEER Licensure Examination Saturday, November 17, 2001 102-00 p.m, - 06.00 p.m. HYDRAULICS AND GEOTECHNICAL ENGINEERING SETA PNSERUCTION: Soest dhe correct answer for cach of the follwing questions fark only one newer for each item by shading the box corresponding t the letter Of your chien te anewer sheet provided. © * ICTLY NO ERASURES ALLOWED. Use pencil no.2only, MULTIPLE CHOICE 1, A trapezoidal canal with sides making an angle of 85° to the horizontal has abseil of 6m, The channel laid ona slape of 1082 with x= 018, the discharge of 75 m/s, what isthe normal de A1Sm emer B 3m dD. 25m 2 Water flows over rectangular wer (Cy = 1.56) under a head of 1.2m. Ifthe discharge is 1066 m/s, determine the length of the weir, A 6am C'63m 345m D. 52m 3. The section of a gravity dam isas shown in Figure 013, Assume hydrostatic uplift to vary uniformly. Determine the total reaction per at the base of the dam, Use sp. gr of concrete = 24. ‘A. 289,269 ths per foot 112320 ths per foot B. 131,040 Ibs per foot . 3421320 Ihs per foot 4. A rectangular gravity dam 4.2.m wide and 25 m Kigh has 204 m deep of ‘water at ils upstream side. What is the factor of safety against sliding, Asgume y= 06 and use unit weight of conerete ~ 2310 kg/m ‘A o6 COs Boz D. 09 5. The two reservoies shown in Figure G14 are connected with new 1200:m long of S00-mm pipe with roughness coefficient = Q0I2. After 10 years n= 0 the value of What is the percent reduction of flow in the pipe. ‘A. 50% = 75% B 25% D. 200% '6, Determine the value of yin the manometer shown in Figure O15. ‘0432 m, C0283m B.O234m D, 526m 7. A Lem wide rectangular canal cartes a flow of 24 m/s. What is the ‘aitical depth? A. O565m. © 452m B 067m D.0.378m A 460 cc sol sample taken from the ste weighs 8505 grams. After oven drying, it weighed 5844 grams. If the specific gravity of solids is 272, determine the void ratio ofthe soil. ‘A. 0976 © 1187 B. 1155 D. 1432 114 November 2001, Mydraulics ang Geotechnical Engineering o 0 nL 2 B 14 1. 16 v7, ‘A-4BD ce soil sample taken from the site weighs 850.5 grams Alter oven drying, weighed S844 grams. If the speciic gravity of solids ix 272, ee te tatinresed eestor ee ciel ae ‘A 0788 © 09 3 0978 Dosis ‘The section of a cofferdam is a8 shown in Figure 37. IF the coefficient of permeability of the soll is k= 5 x 103 m/s, determine the seepage into the ditches per meter length of the colferdam, A. 0086 m/s ©, 0104 m/s B. O72 m/s 1B, 0.058:m)/s Water floes through « 25 cm. diameter nozzle atthe rate of 10:L/. Calculate the dynamic force available at the nozzle ti ATBN can B. 276N D. 20N Water flows through a horizontal nozile located at the vertical side of » large thnk under a constant head of 3m, The nozdle is located 10 m above the ground. Determine the time for the water to reach the ground alter leaving the nozzle ‘A 102s 143s BO7Es D. 287s The flow of water through a Cipollett weir is 2.4 mis when the head is 0.6 sm. Determine the erest length in meters, ‘A215 355 8. 278 D, 185 A spillway controls a reservoir 5 hectares in ares. The permanent crest is at tlevation 73m. Water can be drawn from clevaiion 765 a to 735 min 42 ‘minutes. Pind the length of the spillway in m. Use Prancis formula A187 C108 OO px 11700, ny (Or, when the compression flange is solid and approximately rectangular in eross section and its area is not less than that of (45-60) the tension flange: 3.0006, ie ee as- B Gaya;y ern. In the foregsin 1 Gisance beeen cross section braced against twist or lateral displacement of the mression flange, mm. For cantilevers braced ay t twist only at hemuppat may conservatwely be taker a ve arta! length ry radius of of section comprising th compression Range plus 1/3 ff the compression web area, taken about an axis in the plane of the we, Ay area of the compression flange, mm® = 175. 1.05 (4/4) + 0.30 (4/4? but not more than 2:3, where i the ‘sznller and Ms the larger bending moment at the ends of the unbraced length, taken about the strong axis of the member, and where Mi/iMls the Fat of end esoments, is positive when M, and M; have the same sige (rowerse curvature bencling) and negative when they are of opposite signs Civil Reference Vol. 2. 129 2. (ingle curvature bending). When the bending moment an unbraced length is larger than that st both ende of this length, the value cf shall be taken as unity. When computing Fy and Fry to be used in formula (46-1a), Cp may be computed by the formula given above for frames subject © joint translation, and ie full be taken as tut fir frames braced agaist joint translation. Cay conservatively be abe as iy or ‘any point within W21 62 steel is used as a beam simply supported over « span of # im. The team is terally unsupported ever te cauieapan: f= 28 obo ‘The properties of the section areas follows: Depth, H/= 553 mm Flange width by 210:mm Flange thickness, = 15.6 mm ‘Web thickness, 1. = 102mm. Radius of gyration, y= $3.34 mm Section modulus, S;~ 2.077 «10% mm Which of the following gives the value of the ratio Lr. A. 130 C170 B. 150 B. 160 Which of the following gives the allowable benuling stress in MegaPascals in accordance with the provisions of the National Stuctora Cede of the Philippines ‘a. 150 S201 8 7321 D. 6377 ih ofthe folowing gives the sale uniformly dsttbuted lod thatthe beam can cary in KiloNewton per meter A 1938 1656 B 1521 B 1632 ‘Structural Engineering ‘and Construction Figure ME SL Figure co-37 -ENDe ‘Civil Engineering Reference Vol. 2 131 bm Solutions to November 2001 Examination 2 Situation 1 (1 to 3) Re 04D : ® Be Pace) From the force polygon in Figure () singor = EO, Ro ~ 3000 In Figure (i) tan d=, 0= 3687" In Figure (0): a= 0-30" = 687° Rw FEMs=0) Gsceos a)(10) = Ro(746H) Gaccos 6877)(10) = 2000(7.464) ‘The = 2255.39 N- (EFr=0] Ray+ Tecsind = Rpsin 30" Riy~ 2000 sin 30° - 2255.39 win 36.87" Ray 146.76 Seructsral Egtacering 132 November 2001 ‘and Construction Station 2010) Balanced pom BSL By 600 ° >” 7 (600+ f,) §| | 2am f= 0.85 = 0.008%". = 30) 3 =085- 0.008(245 ~30) = 016 3 85(34.5)(0.814)(600) = ex eee a boa Assume compression ste! yilds: am A's= (282) = 1231.5 mm? Av= $027) = 4825.5 mmi = A\-Ag= 48255-12315 =3594 mint [G=T] 085 f.2b= Aah 0.85(34.5) 4 (300) = 3598(345) = 1409 mn aie 003 M0.9= 08146 ¢=173 mm ‘s_eyagoeeo Check ifcompression steel yields: sal com From the strain diagram shown: f./ 200,000 0.003, 108; 13 {f= 3746 MPa> f, (yield) “assumption comect” ‘Thus, the depth of compression block, «= 140.9 mm ‘Nominal moment capacity: Ma=Ma+ Mar Mom Tild—0/2)+ Tad 2) Mam Aa fy (41/2) Aufl) ‘Mg = 3594(845)(450 - 140.9/2) + 1231.5(845)(385) My = 634189419 N-mm = 634.19 KN-m Civil Engineering Reference Vol. 2 133 (Gh Situation 8 (7109) 5_[_ Gunes [Toate | Pecenage ent | 30 0 ‘Sua a9 Thema [38 7 1300 | aa, kan 3 —| — 00 0 e500 78 Teme —1 10 a0 Ta is Teen’ —1— 3 2 1a a7 tee] i 0 ses ‘Toa | FPS Total contract price = P790,000 Minimum price to qualify as a major pay tem = 20%%(790.000) = F158,000 ‘Among the items given, Item 2 is the major pay item Situation 4 (10032) k= KeL=1Q)=3 m= 300mm ~ 5.56 MPa (rom Table 31) ‘E=731(1000)= 7310 MPa (From Table 3.1) xeoon JE miner [731000 o 155 E 956, ‘Since |,/d> K; the column is a Lang Column = O30E 0307310) Tat ae P= 4775 MPa P= PeA~4,775(140 » 140) ~ 93,595 P= 98585 kN 1 Situation (13 1015) Engineering 134 November 2001 maeeecues Reference Vol. 2 135 ‘Acceleration any tas Si i 4 g = kph = 16.87 m/s 7 4s Sa (166798 = 01+ 2a7)pa= 1.852 myst eo hae ea and = 8% = 008-0 =4.579° ‘G2 Situation? (19 to 21) Minimum coefficient of friction shen the tick is at rest: tan = 008 = Reaction at the fxed end: 2 (r= 9] ‘T+ 1000 = 1200+ 800; T= 1000 N-m_ ‘Normal resction ot th front whale, Resultant torque in each segment: mang DG Tee theo 800 = 30 Nan Ro()+ REFS) + Wsin (05) = W/cas 0013) Ten = -800 Nem Ru@)= Weos (1 5) -1Wsin 905) -REI Note: REF = Mea Rup) — 1900 BN See A574 OS an 4974] ABONL EEO) Masini sori soe Ra = BO106 N- Tyoax = Tas * 1000 Nom Tr, 16r 2 Situation (16 1018) Fee ae, ‘The concrete is @ normal weight concrete. Since the stot! le used is +16(1900 « 1000) not grade 415 MPa, the value shall be multiplied by (04+ f;/ 700), Ss ee mses © Forslab By: (Cantilever) s tans 04 + 5/70" AE ots 276/70 bane rk a n= 31820 iG 1G" For slab By: (Both end continuous) P= 3 Dt= 4 (A) = 405,000 mm! . L 8000, oe pon + 2 fan (04 + 5 / 700)= 5? 4» 276/700) 0 pees my 10001012000 + 2005201500) ‘on™ 227 mam - $00105@000)] Forslab Bx: (One end-continuows) 6 =-0.03882 radians x 180° a2 han (04 + f/ 700) = . Iron 199 ma . ; ") t ee _ > , ‘Structural Engineering. Civil Engineering 136 November 2002 snd Conrtruerlon Reference Vol. 137 DSiteationd @t024) Th Situation 10 (281030) Labor cost per hour = 50(2) + 40(1) +35()= P265.00/hour Fate oe ene ten Labor cost per cubic metor = F28.20/ hI 0553 seu. om aa ee 5 Oi hear G.= (for simply supported) Material cost per cubic meter =3(150) + 0.25(800) + 1(250) Material cost per cubic moter = P775.00/eu. m. Profit overhend, and miscellaneous = 30%(GH8S + 77) = P402.17 aney ean [55200000 _ 115, VAT = 10%(588 9) = P5889 Total cost per cubic meter PS88.9 + P77S + PA00.17 + P58.89 Total cost per cubic meter = PI831.96fcw. m. frSem — [ASOD — se 170.107, and CaF + then Fis the larger value of {2 Situation 9 (25 t0 27) +. = moment in kN-m i 83,000, R= reaction in kN Fay Penal more han 08, = Tradian i704 10°C, _ 1170410" = Aas= Arenas Fe a Se Oa El 1= Fie) - 4669) OaF 49.98)" 83.000 a ah pe oo Gaza fn 94" 25) = 276 ae 200,000(5% 10") Fi etree 63.77 | 1000 8M 24R es Ny Tanons33)/327] "SZ Ma M-42=125 >) ee abo 06 Fy = 0.6(250) = 150 MPa 2 = Baym Fp (Areaan 39)=0 Aves 34 = M(8)(4) - 4(8)(3R)(16/3) = OM=I6R/3 > 2) ee Inky. (i: 16R/3-4R = 125; R = 9375 KN Delis heey Infy.@) | M=1603:75)/3 pe MEM = 500 kN-m PS Ma=M-8R " M= Fy Se 63.77(2077 « 103) = 132,450,290. Ma=-250 KNm M= 18285 em : wit ae & Der) neb= =e . we" 16.56 kN/m 138 November 2001 Civil Engineering Reference Vol. 2 139 CIVIL EXGINGER Licensure Bearnination TICS, SU 7 : SEEA INSTRUCTION; Select the coret ansurr fr each ofthe following questions Mask nemo cal i by shang he ox cnrrpening tm he your hao = “i STRICTLY NS ERASURES ALLOWED, Use perc o. 2 only Mucriece cHoice 1 Evaluate tim | AG 3 D, infinity 2 Which of th lowing ae the gests flective ake? ‘A. 1231% compounded quarterly 8 1220% compounded month ©. 1239% compounded anni , 12.32% compounded semi-annually 3. InP. = 6nPs what is the value of? AD 1 BS D6 4. A circle of tadtius 8 cam is inscribed in's sector having a central angle of 80 What isthe area of the seclor? A. 195.63 cm? © 321.47 cmt B, 291.84 cm? D. 475.42 cme 3. Given a triangle ABC with sides AB = 30 cm, BC = 36 cm, and AC = 48 cm. Find the distance of the point of intersection of perpendicular biseclors to side BC. A 18.92em © 1236em B. 1821em D. 1a7em 6. A baseline measures 25 km at elevation 520 m. If the average radius of ‘curvature s 6400 km, compute the sea-level distance. ‘A. 24,986.24 m © U997.97 m B, 2495663 m D. 24995 24 m 7. Betermine the distance rom thecurvex** f= I6y* 16r* 64 =0 0 point 4 ey 752 B 528 D. 867 8. Two identical cones with vertical axis, one inverted and the other is upright has base radiss of 1.2 mand height of 48 m. Each cone contains equal olume of oll having apeetic gravity of 8, IF dh depth of oll nthe inverted cone is 24 m, what isthe depth of oil in theupright cone? ‘A. 063m Cistm Te B oa7m D.021m 140 May 2002, a 10 n. B uM 15. 16. 17. ‘Mathematics, Surveying, and ‘Transportation Engineering ane Ree ere ee eee Socios re eens Sogn Tees = a, ¢ Ben often Sa The impect factor of a road with radius 500 tn is 0.15. What maximum hie ee SES ole es ‘A. 97.65 kph © 63.25 kph B 12478 D. SLI iph A pate notes scoring othe parametnc equations ve z-P where x and y are displacements (in metirs) in x and y-direction, spectively and (is ume in seconds, Determine the aceeraton of the fafter !=3 seconds ‘A. 1285 m/s © ma m/s B18 m/s D. 521 m/st Determine the area bounded by the curves y= 4x and yi =8(x- 1). A. 5:24 94, units © 8799 unit B. 241 sq. units D, 47454. units From Station 12 + 180 with centor height of 14 min fill the ground makes a tuniform slope of 5% to Station 12 + 240 whose centor height is 28 m in cat How far isthe point of intersection of the ground and the road fom Station +180 ‘A25m 15m B 20m D. 30m ‘The length of the common tangent of a compound curve is $21 m. Di, = 25%, 1-367, 62°, Find the degree of the secand curve, Use arc basis, As cas BRP Da Amachine having a total first cost of P100,000 has a salvage cost of P0000 after 10 years. Determine the book value after five years using the double ‘dscns balance method. A. 32,768.00 C, PA0,960.00 3B. Pasaido , P34.is200 ‘Acylindsical tank open at the top is made of metal shect having a total area ‘of as square meters Ifthe height of the tank is 1-5 times its bose larner deternine be bose diametnt ofthe tank ‘asa C30 B asm B32m ‘The volume of a regular tetrahadron is 3.182 cubic meter. Find its total surface area. ‘Ac 1247 sq.m. ©, 16879. m. B, 15595q..m DB. 1042sq m. Civil Engineering Reference Vol. 2 161 8. A conical tank 12 fe high and 10 ft across the top is filled with a liquid that ‘weighs 624 pef. How much work is done in pumping, all the ligule at the top of the tank? ‘A, S811 fel © SPATS feb 69,421 feb D. 47453 feb Ww a. ‘A man depasits 76,000.00 every end of three months for his retirement If the interes rate is 1D% annually compounded quartedy, what imp Sum ‘ale cane expect after 2 year? ‘A P5280) © PL2i5.27832 B. 78550000 i ee AIT A composite area consist of a rectangle with vertices at coordinates 3, 0), (9, ah) and 3) and a tangle th verocen at coordinates @ 3) (& 9 ‘and (2, 5). How far from the Y-axis is the centroid of the composite area? ‘A. 6.083 62st . Thee guecig ouside Situs tee ot 97 cd the dist A line passing. throu, aa 3/2. Fin tance, ‘measured along the line, from t poe 3) to the Years, ‘a3 3 3 ssn D, 9018 ‘What is the distance between the vertices of the following ellipse? Sh + 2g es by 68 =. rch < 1021 B, 1254 D, $405 ‘\tiangulr piece of land fas one side ravuring 2 a The land ist be ivided into two equal areas by'a dividing line parallel to the given side ‘What is the length ofthe dividing lino? 6 © 76% B. 8.485 D.8 ‘Determine the tralfic low in a certain highway if the traffic density is 14 ‘eh pa Kilemete with 8 space men spend of 30 ph ‘A. 147.58 vehicles he “G B17 32 vehicles/he B. 127.21 vehicles /he DB, 198.17 vehicles/he ‘The length intercepted on the stadia rodiip 1.8.2. anc! the line of sight makes fan angle of 4° 30 with the horizontal. Find the horizontal distance, in m., from the center of the instrument to the rod. ifthe stadia constant 0:3: and the stadia interval factor is 100 ‘A 17243 © 187.36 m & 974m D. 196.87 m ‘Acspiral easement carve his @ length of 100 m with a central curve havin, radivis of 200 m. Determine the offset distance from the tangent to second-quarter pointof the spiral mn % 677m c199m D.356m “How many diagonals have an undecagon? aa” Re 6s Bs a aq ast Determine the shortest distance from point (2) te the parabola j= 8x, ‘A288 ‘ra Bas D. 632 142 May 2002 agit semen vol 143 2 ate artesian Sipeiare gps Se Solutions to May 2002 Examination B 512 Dz 1028 ee ee os “END Since both the numerator and denominator approach zero, we can apply ‘ Hospital's Rule: eg ee eee eee te ea) riage saat Again. both the numerator and denominator approach zero, Soec 2rfooo2x tan 23) + 2sin x ae = Again, both the mamerator and denominator approach zero, Tin ‘Mose 2alsec* 2x) + tan 2x(4 sees sec 2x tan 2x}]+ 2cosx Ba é = H2s0}e2 6 By Gillesania’s Rule: Set x = 0.001 radian (0.18/x degree) eee __ tanf2(0:18/ n)] -2sing0.48 / 2) =20059 (000m) ERS (14 r/myea ‘Choice AcER = (1 + 04231/4)+—1= 01289 = 12.89% Choice B: ER = (1 + 0.129/12)2- 1 = 0.1291 = 1251% Choice C: ER = 1235% Choice D: ER = (1 + 0.1232/2)2-1= 01127 = 12.7% Noten = To fit el 5) Gao But (n=3)! = (n= 3)(n= 4)(n = 3)! ems Besse Bese anone pmo: paps fn =3(n—4)(a~5)1 | (a3)! : (n= Mn 6 a 144 May 2002 reaping = Tar ase m-7n+ 6=0; (n- 1)ln-6)=0 neT(absurd); 1-6 = - snare A R-B= 12-4458 R=2n4458 cm aR6 360 = 3(20.4858)" (80°) a 350° Asmar ™ 291.84 ern? Arsoe * ete eee trig per sectors of any triangle is the center of the circumscribed circle (Giecamecenter), Ara (eae) sa (atbto)/2 > (96+ 48+ 90)/2 57 A= [GNG?— 306? 81720) Are 589.32 ABE _ 3688909) «9405 om 4A, (53932) From the right triangle showm (shaded triangle) atm R= US = 24032 ~ 182 x=1592em me By principle 2. R RGA Dy = horizontal distance at an aliuade off (above sea level) rn ce 145 Reference Vol. 2 D = Actual of Corrected distance at the surface of the earth ‘i= altitude of observation, im Rew radius of the earth, m 2400) meas Using another formula (approximate: fdston factor =1 4) Rew 1 -0.53/6400 Reduction factor = 0.99991875, ‘Sea-level clistance = 25,000 « 0:99991875 = 24,997.97 m De 4 - 16y* 16 64-0 (a2 Abst GA) + (VR— Tby + 6A) = 64 + 64 + 6b {k+ BF + (y= BE Cincle: Center (8.8) Radius = 8 = {16-8 + 4-8 i= 14.56 d= d\~8=656units pa 146 May 2002 Mathematics, Suzveying, and 10 ‘Transportation Engineering In Figure (a); vs 24y Mia (24) y= bVece esas) Va= Voom Vi= § Voom In Figure): e-(z) Vow (as fore Vi lie at of Neer 4) pres (aoa ae Art A= 180 2 2 @ +2) =180 ay 2, S1st— 1dr tx* 16 16 2a itty + 23040 xr Foxe dB be Bh =190 Difference in length = 48-24 = 24.¢m 7 Impact factor (centrifugal ratio) = tan = Fi 2 MS1(500) yar 125 m/s y= 9765 kph 015 = 147 : «= (Gta = (sa one Whenesseec an {26097 416 = 1844 myst mz teas Fr Bx-2) Points of intersection: ax=te—1) pea) ree etfs AwAl-As A=} Gv8)@)- § (248)0) Aw37T sq. units excnematics, Surveying, nd 148 May 2002 ‘Transportation Engineering 1s au Te Te=32Lm 1 ERS) 2 180° Ry = 458.37 m Ty= Ry tan (h/2) Ty = $58.37 tan (36°/2) T= 148.93 m T= 321-T T1707 m Tr= Ratan (h/2) 17207 = R; tan (62°/2) RS Re= 286.37 m x 2086.37) a 180° Dna ove Fe{1-2) 2sv 2 BV5=100.000(1 2y = 132.7680 Civil Engincering Reference Vol. 2 149 tie mur Aaa Pana ls Amance= $D2+nDH; where H=15D 4948 = 424 nD(SD), D=3m The volume of «regular tetrahedron of side ai vas sae 62 ey Adoa= 4x Aree toe Aacce= 4 VG) sin 6 ‘Amece = 15588 qm Work = force x distance a= dP xd #ayd¥=@QAn8 dh d= 1k In the figure shown: Et hae w= 5h/2 aP= 824x(5h/ 12h GF 34.004 bah EN'~ 54034 ie dex 12~ 1) a= 34004 (1258 =F) ae wees [oar iy weseenfer HT wei 2) W= 55,810.75 Alb 150 May 2002, 19 “Another Solution: Work = Force x Distance Force = Weight Force = 7 V = 624 4-nf5) (12) Force = 19,608.54 Lbs Distance = distance from centroid ‘of volume tn point of exit Distance = 44(12) =3 mt ‘Work = 19,603.54 () = 58;810.6 f-tb ‘This is an annuity problem with A = P6,00000 every three months or squarteriy; n= 10(8) =80; i =0.10/4= 0.025 AT=Antan BAF = 186) + (19/3) F-60883 Civil Engineering Reference Vol. 2 151 Oa Slope = tan 0" 3/2 0* 561° ‘From the shaded triangle shown: cos 0= Sd d=5/cas0 Distance between the vertices = 2x (txt + ye + 16x = 16y- 638 =0 peer pense ee Gis + x/4-+ 1/64) +2564" - 16y/25 + 64/625) ~ 648 + 64(1/64) + 25(64/625) Gils+ 1/8) +25(y- 8/25)? 651.56 divide by 651.56 Gesa/e? , y—8/2 | “T0806 * "350824 aim 260624 2=5105 Distance botween the vertices = 22 = 10.21 12 152 May 2002, on Mathematics, Surveying, and ‘Transportation Engineering ‘Since the small and big triangles are similar fas. «(Z) “Agg \ie. AS(2) a a Gs) = BA85 km Flow, ¢= density space nan speed Density = 14 vehicles / kan ‘Space mean speed = 30/miles ‘hr x 1.60975 km/ mile Space mean speed = 48.203 km/hr Flow, @= (4 vehicles/ km)(48.293 km/hr) Flow, ¢= 19847 vehicles/hour H= Deos D= fs+ye9 D=1001.8)+03 D=1803m H= 1803 cos 4° 30° H=17974.m. Offset distance to any point L from PC, x = Lg length of spiral = 100m, ‘R.= racius of simple curve = 300m Lem distance from 1S. to. point tn spiral L= 2B = 50m (econd quarter-point) RG * Se0j005, = 0.684 m An UNDECAGON is a polygon with 11 sides (m= 11) Number of diagonals, D= 5 (n-3) ‘Number of diagonals, D = 42 (11-9) =44 m2 B= @-9ps y-7P ze y/8 B~ (vfh- ape y-28 Eaten yp ayes = Ey ee L wayrm Differentiate de set dd dy to zero: dd _ ay? ui pest yo dands=2 @aQ-ge aap d= 2828 units -400 To get the sum of numerical coefficients in the expansion of a ‘polynomials, assign a value of one (1) to all the variables For(a- b), ‘Sum of numerical coefficients = (1 + 1}! =256 yarautics and civit 154 May 2002 coven ar Reterense Vol'2" 155 ‘Seat No.: ies of the following is closest to the value of the factor of safety agains CIVIL ENGINEER Licensure Examination A ‘Ess C175 Wednesday, | 4, 2002 02:00 p.m. - 06:00 p.m. B. i54 D192 Resend, Map 8. Which ofthe following is closest to the value of the factor of safety againet : overturn HYDRAULICS AND GEOTECHNICAL ENGINEERING az © 39 SEL B. 28 D. 33, i: Select the correct answer for each of the followi questions. 9. Which of the following is closest to the overturning. moment acting against i Eeieancest fr eh ey sing jue Bax Correepanacing te letter: the dam in EN-m. ae me of your coe onthe enw sheet provided ‘35318 km 28563 ICTLY NO ERASURES ALLOWED. Use pencil no. 2 only. B. 324.12kN-m D. 398.75 kNem MULTIPLE CHOICE Situation 2- Water ows from a tank through 160 feet of 4 tnchel ameter pi ‘and then discharges into air as shown tn Figure 20, The flow of water in 1, Caleulate the magnitude of the total hydeestatefonee acting on ofa side of vo Chez se, Aus n= 0018 and nl minor we. the 2 square ple ahown in Figure of the following gives the velocity of low in the pipe inf ‘SAN ©, G2UkN ‘A BSE ests eee B 5682kN D. Ssh BIDS f/s D, 1587/3 2. Two piezometers are installed on a tank as shown in Figure 17. Determine ‘IL. Whichof the following gives the total heed lost-in the pipe in fet. the value of in. A. 7850 feet C. 6140 feet a arom © 415m B. 8890 fect D. 6370 fest 3 dzm Das5M 12. Whichof the following gives the pressure atthe top ofthe talc in ps 2. Wate flows ate rate of 350 cfs n a trapezoidal canal having bse width of A 2927 pa 37 pa Teles and ide alope of tito TV. Determine tee! Soph Asse» 8 288 pat Do Las bat Bae feat . 2312 feet Situation 3 - For the shown in Fi A. 2564 feet a situation 3 - For i stem shown in Figure 17, 1 = 0.015 for all pipes and Pascetiee hal wih most icine geen the ene mae ee 4. A tiangalar channel with most efficient mn waierat of te following is closes tothe head lost in pipe tin feet. fate of | m/s Assuming n= 0018 and 3= O02) calculate the normal A amd cae tho flow in meters. 2 eae 5.539 763m oss2m 4. Which of the following is closest tothe total ond lost in terms ofthe B 0913 m D.Liatm discharge Q, where Q in ef. st he tts 5. An open cylindrical steel tank 24 m in diameter and 3 m high contains 2:1 ‘A Ontzd Qe osm: tm desp of water. Delecmane the stress in the walls of the tank near the B 03073 G 5. oagea o: fottom ifthe walthicknes is 8am. 1B, Whichof the following is cost to the fla ead lst in owt. 3.09 MPa ©. 3.57 MPa AL S374 C. 69.63 }. 247 MPa D. 274MPa B. 2541 D. 47.24 6 Arectangular 5 weir of length 1 m is constructed or cut atthe top. : ofa tall Tank having a horizontal section 20 m by 20m. If the Situation 4 ‘The soil shown in Figure 25 has a void ratio of 0.50 and G= 270. Iy {Rial heodl ower the weir ia 1m compte the time muir to discharge 72 =15me=oe Siem ofwater. Use Pancisformala 16, Which ofthe following ie closest to the effective unit weight of sand in A. 36.52 seconds C. 58.74 seconds N/m. B. 45:35 seconds D. 24.12 seconds = Ana SITUATIONAL 17, Which of the following is closest to the effective stress at point A in kPa 3835 ca Staion 1 = The scion of a concrete gravity cam shown in Figure 97, The an rh oe rn D. 6299 th of water atthe upstean sets mL Neglect hydrostatic uplift ich ofthe following is closet to elical Hydraulic gradient of se Siri erighe of somes = 250 AN/np-_ Comiced of fiction betveen ick condition). arate en ee the base of the dam and the foundation is 0.6 ‘Mydrauiics and 156 May Geotechnical Engineering “A085 © Lis 187 B54 Situation 5 ~ A vertical retaining wall retains 26-¢ deep of soil. The soil has a yur weight of 10 pl above water abe and 10 pe! low ater abe ‘water table ts 10 feet below the ground Surface. ‘The angle of inueal fctonef he eolln ss" 19, Which of the following is closest to the total ative pressure acting on the vain posi perfook width ©. 1752 B. 16,245, D. 14353, 20. Which of the following is closest to the location of the resultant pressure from the ground surface in feet. A. 68 c74 B. 192 D. 186 ‘A BL VH0452 fl D. 98,458 fie Situation 6 = A slid concrete reining wall shown i Figaro 67 The fl ‘behind the wall has a unit weight of L10 pef whose active soil pressure may be asoumed equivalent to a il prose of 30 pst per foot, The pasive equivalent to a fluid pressure of 200 psf per foot. Fre lve Lod exchange behod Gy all equivalent an alto of fect of fill Assume not weight of concrete = 150 po, Consider 1 length 2 Hi of he alowing ses he ala presage wal in ‘A. 4.500 7200 B. 8,300 B, 6000 BB Which he following gives the overturring mement about the toe in fb, A. 52000 ©. 47,000 B, 65,000 , 36,000 24. Which of the following gives the factor of safety agatnet overturning, A412 © 2a B32 Bi. 187 Situation 7 = A soil deposit is shown in Figure 32 ‘The grownd water table, pean “ter Sc wen the dyes a staan ef he and shoe afer Su of saaraton above 2, eine: the vertical effective the midhetght 3 wes pressure a cof the. ie qetiesfees lowering of the water table, in psf. iB be 26. Which ofthe flveing ives irs tha eon elo cee paste ee ilnght ‘of the clay Zea rational the wre Shree ‘ B 6) D. 5530 ae Civil Engineering Reference Vol. 2 157 27. Which of the following, gives the vertical efecive pressure when there sno ‘water in the sand layer, in pst A. 6710 © 5180 B 7450 D. 4660 Staion @ Io he sing of Figur 12 he dg dp — 20h nd = (0.002 Pas. The fove through the needle is 04 m/s, Neglect head loss 8 fergie the velo Binm/: chal the ves the velocity at point B in m/s. A732 S Coa B89 Daa 29, Which of the following gives the Reynolds number for the flow in the peed nea c. 1035 . D. 1878 20, hi ‘of the following gives the steady force F required to produce the BSN C 243N B 107N B384N rer le i= 10 0m 158 May 2002 160 May Geotechnical Engineering i Solutions to May 2002 Examination or 581(15+ Vin A522) ( ALG2KN, Summing-up pressure head from 8 fo © in meters of water, Fem 75) ° +225-he BA y= 425 m fe Atetitcal stage, 5 = O= 350 f0/s Ba ize ad Am 12d + Vald)(2d) x 2= 124 2 goo? _ iad 2st)? 322 Redd ee ‘By trial and error using the choices, d = 2.564 feet ‘Civil Engineering Neference Vol. 2 162 a4 Note: The proportion for most efficient triangular section is that of a $0- degree triangular V-notch. Aa Veda) =P penal )= 29088 ms onaleas: i a 9.00212 THe 25360 coon 1-128 20913 m as Tangerial stress, po pressure = y = 9.81(21) = 20.601 kPa D = internal diameter = 24 m= 2400 mm_ {= wall thickness = 8 mm 20 601(2800) o~ —— = 3090.15 kPa = 3.09 MPa 26) dle pe 24.) 11 Gt | Jar Ia ‘fone matics a any tame, Ay = 20(20) = 400 m? Langs oe oT tnetiead Hm ‘The drop of water level after discharging 72. ls Weir factor (Francis), Co = 1.84 a0) [_1_ 1] t= 20) [1 _1] 4535 seconds isi) [Jor i) 162 May DSituation 1 7 109) F=yeiiA = 981G)6 1) F=17658kN y= (0/3}6)=2m_ OM= Fx y=17658(2) OM = 35316 kN-m y= Wit Wem 376 + 188 = 56 EN au ce) Fae = reer Se RM = Wx + Wa x2 = 3763) + 188(1.353) = 1578.60 Nem rope BM. 1I7BSO 5 ooy OM ~ “35816 {Situation 2 (10 10 12) Q= 1240/s = 03401 m/s Dem 4" = 0333 f= 101.6 mm n=0013 L = 160 feet = 48.78 m or aun 6 | "Sa ee. ein oss) ; = 137785 fs pp = 029m LQ _ ro29co13)$ 48790-3400)" 2a (oasigy? ‘HE, = 1942.39 m= 6570.86 feet Civil Engineering Reference Vol. 2 163 Energy equation between A and C: ener Oe Paez ne 4 Bey men ae Brag ee 937785)" 9+ 100 230.2) one +10- 37086. B 6755.65 feet of water 7 Po ATLS528 psf = 2927.45 pst Using the English units for Manning's Formula: v= 8 enon vm 137-785 it/s R= D/4 = 0333/4 = 0.0833 ft S=HL/L=HL/ 160 =e my ye 137.785 = 7, (0.0833)2HL 160) HL = 6356 feot £0 Situation 3 (1310 15) Q=Qi= i= 12 (0/5 QnAe- ALE per sv2 English Version = Fp: 18 wyspocauyyse B= SAL pu = 46870035)" (1500)(12)? _ gy, Hla 7 (24/12) 59 feet 164 May 2002 (eemetaeiar meee "Total Head Lost, HL= Hla + Hla + HL Total Flow = Q)= Qu= 0 __ 4687 (0.085)? (1500) 0" Hy COMO a 0a3ss.Q: canny = £687 (0015) (2000) * _ 5 ge, ae any ae Qta-0 Fila = Hl. 4.637(0.015)" (4000)0,7 _ 4.637(0.015)? (S00) Q.* ag /12)"7 ays Qs = 030380, 2+ 0300:-0 @=076720 é = 4637 (0.015) (4000)(0.76720)" _ sas (8/12) cae: HL = 09388 @2+ 0.2826 Q2 + 0.051802 ‘HL = 03732. Q? ‘Total hoad lost = 0.3732(12)#= 53.74 feet Situation Effective unit weight = Submerged unit weight 1» cat. on Se = Fg 08 MLAS KN? Effective stress at ul = Yu I= 20.528) 6) = 62.784 KPa Po" Yothe = 9813 15) = 44.145 KPa Pee~ Pron ~ P= 62784- 44:145 = 18,639 kPa Critical hydrautic gradient: Gai 27-1 455 Tee "1405 Civil Engineering Reference Vol. 2 165 {Situation 5 (19 to 21) ese Fe 10 pt Aes oS Ye fat fe = 120-624 orem sadam “ee n= 576 pot _ Lesing 1FHine Considering 14oot length of wall: Pressure: = YK, Hy= 11Q0.271)(10) = 298.1 psf Ke Hs ~ 57.6(0-271)(16) ~ 249.75 pt pom teHs= 62.4(16) = 998.4 pst Total pressure: r= Vapa Hit = ¥4298.1)(10) = 14905 tbs Fam pe Hh = 298.1(16) = 476956 ths y= Vepa th = ¥A2H975)(16) = 1998 Ibs = Vepa ls = Vd998.4)(16) = 79872 Ibs Toulfore, FeAt tht Fm 14905 + 4769.6 + 1998 + 7087.2 P= 16,24533 Ibs Location of F y.= (@2/3)(40) = 20/3 A. yen 10-+16/2= 186, ‘yum yee 10% (2/8)(16) = 20.667 ft Taking moment of forces about the top: Fehr yt But Foye 16,2453 ¥ = 1490.5(00/3) + 769.6(18) + 1998(20.667) + 7987.2(20667) Yo 186 ft 26 26~18.6= 74 ft Overtuming moment = F z= 16,2453 (74) Overturming moment = 120,215.22 fll 166 May 2002, DSitwation 6 (22t0 24) = 2002) = 60 pot Fa2= 30120) = 600 pst ‘Active pressure: Fx = px(20) = (60)(20) = 1200 Ibs a= Yaps (20) = ¥2(600)(20) ~ 6000 Ibs Total active pressure, F=F\ + F=72001bs Overturning moment: yn 20/2= 108. yam 20/3 ft OM™ Fit Faye OM = 1200(10) +6000(20/3) = 52,000 #15 Righting moment moment: Wir Vim 150018 x 2 1) » S400 tbs Ve 150(12% 2 1) = 3600 Ibs TO(20 x 6x 1) = 19200 Ths (ia) = 6 2-4 6)=9 i RM = Wii + Wi.xa + 1.29 = S800(5) + 36006) + 13200(9) RM = 167,800 feb Civil Engineering Reference Vol. 2 167 Facts agit orn 2 = 7A Factor of safety against overturning = 3.22. © Situation 7 (25 to 27) Vertical effective pressure at A before lowering of the water table: pas Dyph = (120 ~624)(125) + (135 - 624)(50) a= 4,350 pst After lowering of the water table: ask aoe 7. ae Swett aed iso Re Ya" 116 + 0.2(135~ 116) =119.8 peh a= (120 - 62-4)(12.5) + (135 - 624)(25) + 119.8(25) Pa= 5530 pst Vertical effective pressure at A whien thore is no water in the sand layer: PA (120 -624)(125) +119.8160) Pa~ 6710p ydrautes and 1638 May 2002 (Situation § (28 to 30) Q._oaxi0* Larner A Foco0es) me=Sass7ays ie- 2 aetna ny =biare tame Oooh nergy Equation between A and 8: Bone Pag BAe. He 2s Bes Be one here ‘04 O (negligible) He-iyintie nestle ‘The head lost in the syringe is negligible ‘Since the flow is laminar: "f= 64 Re = 61/916.73 = 0.0638 py = 2080.98 (.02/0.4.10-)? 0.00025)" HL = 1689 m easy 20.81) 0+ FA +0-1859= +040 x a= 196,681 Pa Force, F= pa x Area of piston Force, F= 196,681 x 4 (0,01) = 1545 N Civil Engineering Reference Vol. 2 169 Seat Nor CIVIL ENGINEER Licensure Examination Thureday, May 22002 06.00 01:00 pam. UCTURAL ENGINEERING AND CONSTRU SERA INSTRUCTION: Sele the corect answer for each of the fllowing questions ark anizans anise rch in by aading We box coresponding Be oer of your choice on the anayrer shest STRICHEY NO ERASURIS ALLOWED, Use pene no. 2 only MULTIPLE CHOICE ation 1 ~The force P= 50 Nis hin Figure MIE-17. I Whe othe fllowing gives the cement of Pin Newton. 300 cs B, 350 2 a the following gives! the compose ‘of P in the Xaxis in Newton, ich 5 3 3. Which of the following gives the component of in the Y-axis in Newton. eee 8 Bl os B, 250 D360 Situation 2— Fo the cantina bom shown in gu ANS 4 Who he ollorang ve he moment uation for pois A and 2, ‘A Ma + ahs 32 = ©. 8M 2M 35 =0 weeMngaMs <0 DMs Nan 27m 5. Which ofthe following gives the moment equation for points A,B and C Sitar LNs aoe ©. Ma 1ede> 301 20 B. AM + 1a 81 =O D. Ma’ 13M 3810 6. Which of the following gives the moment et in Kilonwton meter ‘A 2724 ©3247 B52 D, 2865 Situalion 3A uyperi cable shown fn Figure ME76, Which of the following most neorly gives the tension in segment BC in KiloNewton. A883 C1854 8. 7023 D. 3147 Wyck of the foliowang mort nary gives the angle in degeses, oe D. S241 Sr WHE af ths foley sng mace cas ty res thes fal leigh of i cab meters ‘A 2948 C547 B 9225 D. 2817 Structural fagincering: 170 May 2002 ‘and Construction Situation 4A plate with width of $00 mllinwters and thickness of 12 millimeters ‘Sto be connected to a plate of the same width and thickness by 34 millimeters diameter bolts, as shown in Figure ST-13. The holes are 2 mm larger than the bolt diameter, The plate is A36 steel with yield strength fy = 248 MegaPascab, Allowable tensile stress i O60, Ik i mequired to determine the value of bso that the net width along bolts 1-234 is equal to the net wad along bolts 1-24 10, Which ofthe following most ives the value of b in millimeters. A 286 eee B. 374, D197 ‘IL. Which of the following most nearly. gives the value of the net area for tension in plates in square millimeters, 3634 © 3867 B. 3214 Di. 4178 12. Which of the following most nearly gives the value of P so that the allowable tensile stress of net area will not be excoeded, ‘ASSEN C59 KN BEN D.O4kN Sifuation5 - The shear diagram of a loaded beam is shown in Figure ME-€3. 15. Which of the following gives the largest concentrated. load (not inckacing reaction) acting on the beam in KiloNewton, ‘A. 10 C8 B. 40 B. 20 1M. Which of the following gives the maximum negative bending moment in KiloNewtor-meter, AB oes. B, 20, D. 10 15, Which ofthe following most nearly gves the location af «point of nection from the right end of the beam in meters. A, 287m 357m B 3am B375m. ‘Situation 6 — Section 5.11 of NSCF states the following for shear on beams subject { combined flexure and ardal load: SA132 Shoas strength Vi may, be computed by the more detaied ‘alculation of Se. $113.21 through $11.3. ‘5:11.321 For members subject to shear and flexure only, oe [rer ef) a] na 29) but not greater than 03,7), be d. Quantity Vs d/Mu shall not be taken ter than 1.0 in computing V, by Eq, (11-6), where My is factored! moment EEeurring simalianeously wath Vs at stction considered, Civil Engineering Reference Vol. 2 171i 5.11.3.2.2 For members subject to axial compression. Eq. (11-6) may be used 40 compute V; with Ms substituted for My andl Vs 4/2 not then limited to 10st Mee Meanedta! qu ‘However, Ve shall not be taken greater than vin03 Fy bed fo an) ‘Quantity 1, / A, shall be expressed in MP2. When M, €117) is negative, ¥, shall be computed by Eq. (11-8). In the forgoing: “Ay = gross area of section, mm? = web’ width, dr diameter of circular section, mm = overall thickness of member, mm Af bod = factored exdal load normal to cross section occurring simultaneously. ‘with Visto be taken as postive for compression, negative for tension, and to include effects of tension due te creep and shrinkage ‘Mu factored moment at section A prismatic beam having. 300 mm x 500 mm has an effective depth of 430 thm ant is simply, euppovied over a span of 5 m. The beam relorced for tension only with 5-25 mm bars having f= 275 MPa. ‘The beam curries a factored concentrated! toad of 450 KN at midspen and a factored axcal compressive force of 100 kN inclucling the effects of tension due to cree ‘nd ahninkage. Concrete weighs 235 RN/a, Concrete strengih f 16, Which of the following mest nearly ‘Ma fo be used at the critical section i 42 B 963 D865 17, Which of the follawing most nearly gives the value of the shear force V. at critical section. in KN Alm © 1561 18, Which ofthe fol aie a lowing most nearly gives the shear force to be carried shear reinforcement in EN. a erent ta A110 <1 B Wo i) Situation 7 ~ Three alternative pipe systems aro uncer consideration. Each has teen designed! to meet the project specifications. ‘The installation, annual ‘maintenance, and hourly operating costs are summarized in the following table. Each pipe system is to be used (or 500 hrs per year and has a project life oF 10 years, and then will become valueless. Use interest rate of 19%. 172 May 2002 ‘and Construction a Te 19. Which of the following most nearly ghves the capitalized cost of the 150-mm pipe. ‘A, P2856,45200 C. F3.380,683.00 B. P310752600 D. P3654.45200 20. Which of the following most nearly gives the annual cost of the 100-xm MK pargzas.00 © Pse9452.00 B. Pas2a78.00 . P535,402.00 21. Which of the following most nearly gives pipe systems is the most “100m © 250. ‘A, 100-mn mo pipe B stm pipe D, Not enough data Situation 8 - Section 57.7 of NSCP states the following provisions for concrete rotestion for reinforcement S271 Concinplace, Canereie(nonpretrssed). The follwing, minim concrete cover shall be provided for reinforcement: inimam cover, mm () Concrete cast against and ly exposed toearh......75 ()) Conerete exposte to earth or weathor 8 20 mim through 36mm Bars. oa 16mm bar, Wl or D3] wire, and smaller. 40 (0) Concrete not exposed to weathoror in contact with ground) Slabs. walls, joists ‘S2mmbar and smaller.. rimay envoroanent, es. straps spa. Shells, folded place members 20 mum ba and Larger Iisenan har, W31 oF DSI wire, and smaller ‘Given the details for slab, column. end footing in Figure CO-29., 22. Whichof the following gives the minimum cover for Detail 01 ‘A. 40mm. 20mm B isan D. 50mm 23. Whichof the fellowing gives the minimum cover for Detail 2: A 40mm C, 50mm B 15mm D, 75 min 24, Which ofthe following gives the minimum cover for Detail 03: A. 40mm 50mm B 20mm D. 75 mum Reference Vol. 2 173 Situation 9 - The activities, duration, and cost under normal and accelerated ‘conditions for a network diagram is shown in the accompanying table. 25, Which ofthe fellow ives the efi path ofthe network. ABCEF OChEF 25, Which of the fll fey pve econ of bm ce ofthe following most reary gives the costo Fitts sabe accelerated tof mam duration, See ‘A. PaLO0O ©. P5000 B. 724,100.00 D. P23,4000 2, Fora total cost of P22.100, which ofthe following gives the number of days tha jolrcan be accelerate for most economical oral ‘A I7days Teslays B idays Di. 19 days Situation 10 - For the riveted bracket shown in Figure ST-26, P= 250 AN. The Fivets have diameter of 23 mm. The conswction can be analyzed By iven load with an equivalent loading compased of a verte {otc alse acing tough the censbid ofthewsves ada moment 28. Which of the fo ‘most nearly gives the shearing stress in ech viv due to the vertical for eae . gate alone aeting tthe centroid. A, 156 MPa C1273 MPa B. 1128MPa D, 1864 MPa 29, Which of the following most nearly gives the shearing stress in rivet D in MegaPascals ABS C956 Be 1567 D. 134.7 30. Which of the following most nearly gives the stress in rivet B when the instantaneous center (IC) is 200 mm to the left of eg, in MegaPascals A7as 396 B 1875 Dag ) 6 Civil Engineering 174 May snd Conscraction Reference Vol. 2 175. Figure st-26 ‘Structural Engineering 176 May 2002, 2018 20m 2088 1 cil incering Soacaeyars 177 T Solutions to May 2002 Examination Situation (1 t93) X-component of P: Pr Peos 30° ‘Pie= 450.cos 90 = 359.71 kN component of Pr=P cos 6 Fre = 450 cas 60° =225 kN Y- Minimum duration 345+ 3+3.= 1 days ‘What if? If new critical paih is established (ay D-For BEF). try toaccelerate the activities along this new path, Part3 ‘Maximum available cost is P22,100- The normal cost (from Part 1) is P21,000. Since the maximum cost isto be 22-100, we may accelerate some activities with an additional cost of 1.10.00 Accelerate items along the critical path: Acti A (atin crt ine" 5-3 2) Crashcost Neraadion= Ard 1-100 35=3 Activity C: (Maximum crash time = at oemting = 2200-8008 Atty nimi ch ioe = 4-9 = a) Coto cmting = 1282 909 perday DAE Oded eh eS 3-2) 2800-1098 «50.00 pray Cost of crashing = Cost of crashing = Cost of crashing = 186 May 2002 ‘and Construction ‘The best activity or activites to crash first are those that have the least cost of crashing. Among these setivities, the best to crsh is Activity C while Activity Eis the worse For a total of P1,100, Activity C can be crashed for? days for a cost of 2 « 250 = PS00.00, and Activity A can be crashed for 2 days for a cost of 2 £300 = 00.00. For most economical project duration, the project may be crashed for a otal of 2+ 2= 4 days. ‘Thus, the job ean be accelerated to 21-4 =17 days 2 Situation 19 @8 © 30) Direct stress: Ramcr™ P/4= 250/4~ 625KN 625% 10? Stress = S29x10" $(25)7 =12732Mra Stress in rivet D: “T= P{25d) = 250 x 104250) T= 62,500,000 N-mm_ (e+ yf) = (50 + 2008) 4 Z(t +?) = 250/000 mme Rope TY = $2-50,000(200) Dersy) 2000 j Rox = 50,000 N = 50kN > Roy TE - = SOO BONIDD) tee) "250,000 Roy TSENY = 37500N ‘Total reaction at D: Ro= Yio +(975-625)° = 55.901 kN 5590110" ey Stress =11388 MPa Civil Engineering Reference Vol. 2 187 Fart'3 7 =250,900(250 + 200) T=112510N-mm, Eke + ¥) = 2608+ 2005) + 2,350: + 2005) EG? +!) = 410,000 mm? Ty me Betty") te. = 21254108 (200) ss 410,000 Ray ™ 54/878,05 N = 54.878 KN Te paass10' aso) Ra Ser aa EN Fay = 96937 KN Re= y(St878)!(@5+96097)" = 167.77 kN 183 November 2002 Ee Seat Noi CIVIL ENGINEER Licensure Examination Saturday, November, 23, 2002 08:00 a.m. - 01:00 p.m, MATHEMATICS, SURVEYING, AND-TRANSPORTATION ENG'G, SETA Select the correct answer for each of the following questions skank ane ane ech my tang ts bxcoerponding othe tr ot onthe answer sheet provided. STRICTLY NO ERASURES ALLOWED. Use pene no, Zon. MULTIPLE CHOICE 1, Find the value of x in (@°5)06) =34(25) ‘Note: The expression” means povie. ABS ©95 Bo D8 2 Whats dhe va of yin the harmontsproprssion 1, 1/5,1/9.1/ 2 Alo Bie DB 3, Xs 12% older that ¥ while Yis 11% elder than Z. By how much in percent Z. is X older than a B 2178 D. 23.66 i 4. P36,000i8 tobe divided among Artur, Bernardo, and Caloy such thet their shares nthe sae order, form an arithmetic progression, Beynards share Sie tna unr mane geo ihe 9 it ce le ABC, A = 116° 19, B= 55° 90, and C= 80° 37. —— a Z aoe. a ee 8 adnan outside the hexagon but iaide th cae 1524. AB 3q-c te 717 sq cm St eeewenntenpie eh tiaing, eer Tt ecramacengae eetiaginy, Beoe % Ce ae eet aac my onde layer base 2A ch 24 m oquare volume of the frustum in cubic meters. ote C57 33 Das ir ic em in diameter is. placed into a tall vertical ae oneness causing the water lvl tows by Oca. Wha 8 radius of the cylinder? ‘ia ings B Sos BL MS Reference Vol. 2 189 10. 1, Bb 4. 15. ¥6, vy, 18 1», ‘A conical vessel one meter diameter atthe top and 6llem high holds salt ata sepll of 36 cm fram the boitom, Haw many cc of salt does it contain? A g72t4 . 35,896 B 33,929 D. Bi57a ‘The area enclosed by the curve xt + y= 25 is revolved about the line x =0. Find the volume generated, A. 4908 C4788 3. 651 D. 4509 “A tangle have vertices at A(-3,-2), B(2 6), and C(@, 2): What is the ‘af the centroid of the triangle? abscissa A 3/4 C3/2 B 5/4 Bl IE he distance between points A(2 10,4) and B(, 52) e 9484, what tho ‘Ad 3 Be D5 ‘Determine the radius of the sphere whose equation is sobre eeg tino AS E B.3 D2 ‘What isthe derivative of y=9°@x)? ‘Note: The-expression * means porves. ‘A. 3*Gx +1) In @) © 3*Gr-3)In @) B. Gx) in) D.3*Gn in Gx) ‘Suppose that x years alter founding in 197, a erin erploy se association membership of fe) = 1002s) ~ 452+ 2643), at what tinwe between 1975 and 1989 was the memborship smallest? A, 1583 1984 A Siete lng steel pipe ass upper ee eaing er lang, steel pipe has its upper against a vertical wall fand lower end en alevel ground, ‘The lower end moves away at a constant rate of 2 cm/s. How fasts the upper end moving down, in cm/s, when the lower end is 2m from the wall? ‘A 181 cw B 1.66 D. 198 Determine the value of the integral of sin® x dx with limits from Oto-/6. ‘A O32 © 0275 Find eeengihof Fierce tonya ind the length of one arco the curve whose parsmetric equations are 1=2)-2sinOandy=2=2 cos 0, Saree A. 16 cu B18 D2 A body starting from point A is given an acceleration which uniformly increase from zero at A to 18 m/s? at B in 4 seconds. The initial velocity at ‘Ais3 m/s. Find the distance travelled at the end of 4 seconds. A 148m 162m B. 165m Bvim i190 November 2002 i} 2. Mathematics, Surveying, and ‘Transportation Engineering, ‘A comractor can buy trucks for PEDOO00 each, or rent them for FI,200 truck per day. The truck has a salvage value ef P100,000 ot the end of its Useful le of 5 years The annual maintenance coot is F20,000 por track. Using the anwaloat method and 14% interest rate, determine the number ofl per year hat each tack must be used to Warrant purchase, Use sig fi ‘method of depreciation. 4 C15 BT D. 199 What is the diftence between the sums an annuity due andl an ordinary nmi for the following data: eric payment rio Interest rate 10% compounded quarterly Paymentinterval > 3 months A C. POLES B Pa399, D. Pe7.042 At what interest rate, compounded quarterly, will an inivestment double in five years? A. 166% fosters B 151% D. 137% “The tangent of a simple curve from Pi. to P.C. his a bearing of N 65° Zand the other tangent from Pl. to PT, has a bearing of N'55° We At a point 150 mfrom PC. along the tangent through P-C, the right angle offset to point F 6on the curve is 62 m. Find the radius of the-curve. ‘A 16575m © 523m B 15472m D.1817.5m With the use of an engineer's level, the ‘on a rod 80 m away was found to ke 282 m. The bubble wax levelled om the level tube and the rod reading increased to 2684 m. What i the radius of curvature ofthe level tube if ane space on the fube s 0.6 mm long? 375m ©. 335m Bam D. 350m Given the following cross-section fotes of an earthwork on an S-m-wide roadvay: 6 0 475 +180 140 4150 [Determine the aes of the cross-section. ‘A. 12208 mt © 11428 me B. 11.897 mt D, 12615 me? Given the following areas bounded by the waterline of @ lake and the ‘contours 1,23, 4, and 5. Fach contour are at 2m interval “Ay = 6.150 mi Av 4160 me ‘Age 5010 m2 ‘Ap 3.150 me “Ass 4650 m2 Determine the volume of water in the lake ‘A. 35,500 cu. m, 57800 cu. m. B36 800cu.m. D. 36,900 cu. m. Civil Engineering Reference Vol. 2 191 WA reversed cave of equal rad connects wo parallel tangent 12 mapa ermine ‘The length of chord from PC. to P-T. is 140:m_ ‘etal length of thy revered curve ‘A, 14285 mi, 140.17 m B. 15am D. 167m 28, student recorded the fllonring mamber of pase flv aking a distance ‘of 50 m repeatedly as 715, 720, 70.0, and 69.5, He wardand to measure the distance between two points Cand D. He recorded t!ne following number of paces from Co B of back as 465,468, 463, and 460. What s the distance from Cte D? A. 328m, C. 462m B am D. am 2 Ap ‘of 120 "Pa and a stiffness Factor of 108, Determine the modulus of elasticity of the subgrade ‘A 16MPa ©. 15 afPa B17 MPa D. 1% MPa 20, Cracks in concreit»pavement due to temperature expansions can be ‘prevented or minimized by: 'A. insitring proper consistency and strength of concrete mixture B. proper preparation of subgrade €. Continous pouring of concrete 1D, providing expansion joints and sectional pouring of concrete “END Mathematics, Surveying, and ‘Transporaation Engineering 192 November 2002 bi Solutions to November 2002 Examination m1 ue ome BY. pre Bg=gU el? x=85 m2 Given the harmonic progression: 1, 1/5, 1/9, 1/* “Their reciprocals 1,5, 9, x form an arithmetic progression with common difference of By inapection, = 13 ms x=Lizy Eq.) Ye4lz 424.0) ‘Substitute Yin 2) to (0): Km 112(1.112Z) = 1.24322 Thus, X is 24.37% older than Z. me Let A, 8, and Che the shares of Arturo, Bernardo, and Caloy, respectively, | A+B+C=36,000 > Eq. (1) | Bm 2A 3G. ‘Since their shares form an A. P.: Common difference, d= B- A =C-B Wm AeC 24.0) Substitute Bin 2) to @): TOA)= ATC C=34 > Eq. (a) Substitute B= 24 and C= 3A to Bq. (1) "A-2A +34 = 36,000 A= 6,000 B= 2A = P12,000 (Bernardo’s share} as Given angles of spherical triangle: A = 116° 19%, B = 55° 30/, and C= 80°37" Civil Engineering Reference Vol. 2 193 ‘Using cosine law for angler tos A'=-cos Bcos @+ sin Bin C cose cos 116° 19" = -cos65° 30 cos 80°37 +-sinS5°30' sn 80°37 cosa os a= 1a91604 sss" ‘Ai Area of sector - Arma of triangle ete = 21609 a8 ar erin eo” pases ‘Aseuyon = 6 Area of triangle Kips ODEON Aveagon = 7LT sq.m. [ar+Aa+dAaa] Ar= 2.4? =5.76 m* Aan GQ 1ime b= 1. 0.989951 1.503 m v= 258 bres /ST| v=45om at 194 Nowedilbar 2002 as mio Volume of ball = (Volume rise) xt 4n(10)! =< (10) Re 1155cm Solving fo = x , 100 oO = 60cm Volume of salt = £[ (60P1G6) ‘Volume of salt = 33,9292 ce ‘The curve x2 + y= 25 fs a circle of radius 5 ‘with center at (0,0) From the second ay proposition of Pappus Vn Ax Bak V mnt 2x10) y=4.9348 ca units Rendered sold Civil Engineering Reference Vol. 2 195 ——— at Let (@,9) bethe coordinate of the centroid, Then, =3t2e4 Abscissa, ¢ = “A778 * 5c 3 Ordinate, 7 = Lat¥et¥e . -24642 : 3 mx ‘The distance between points Pir, yi. 23) and x(x ys, 2) is given by: f= tan}? ey) + AD (ASE = 6-2) +8 -10)+ (4 ‘ G-apee rode 2=6and2 as Reduce the equation to standard form, (z~h)?+ (y -k)? + (e~IP= 18 sityit ets Bra2yt1s0 itRre lee yt dye] += 16s) @Hpe y-ipeee Radius & Conter = (4,4, 0) mis ya 4) = at ina A Prom E(t) =a" Ina 4 40) ae NOTE get 3-3! ng) * Ina) x3 Je) = WOR ~45e + 2648) Differentiate the given function and equate to zero: Yea) = 1006 = Sx +264) =0 {e— 90x + 264 =0 s-15u+ 4d =0 (= 1)e-s)=0 xetandr=a When x= 11 tt) = 1oofacrt)>= 45011)*+ 2s4¢E1)] = 12.100 196 ay Mathematics, surveying, and ‘When x=4 fa) = 100,244) — 45(4) + 26449)] = 46,400 ‘Thus, the membership is smallest (relative miniswum) when x= 1 and ‘that is in 1986... ‘The membership is largest (relative maximum) when x = 4and that ss in 1979. ‘From the figure shown, a de By Pythagorean Theorean: y= B00 =a ‘=P ychen x= 2-m (200 cen) 4 a a79cm/s 4 179 cm) ‘Therefore, the upper end moves down atthe rate of 1.79 cm/s [inros Let O=33,x= 38 de~ $a Limits: when x= 0,0=0 whons= $= flrme= fore) from Fou = 4 2 1 4 fovea 4 8 ram Civil Reference Vol. 2 Gis 2-2sin®; y=2-Peosd Note: This eurve is a cyeloid where 2 is the radius of rolling circle and 0 is the angle of rotation ("= 360" for one arc) em 2-208) 0 dy=2sin0 20 . (226088) a0 oof ES RET oon iE | eae a 00 ret 2cas+ cos" 0 [pea aoe x= a(t sin) Yaa cs6) 8 = aus of alleg ore = angie of rotation S-2 [eer 8 = 22 frame but JE=coe8 = V2 sing m2 S= Areas 4(6.6)- FG) = 165m Anmualcostforthe purchase Ube operation an eal preening sheng 8) (fC-SVi AC=(EQIL OMY ey (qo 100200018) AC= (800,000,0.18) # 20000 + Ta aya AC=P237,89648 “Let N/ be the required number of days, per year, then ‘Cost of renting = 11200 N = 237,898.48 ‘N= 198.28-hrs say 199 hours Given: A= P14000.00 n= 16(d)= 6 f= 0,10/4= 0025 Annuity et som p= MBST=M MANOS T sai Civil Engineering Reference Vol. 2 199 ee es “Knmulty Due Sum = (Fay amen XL +1) Sum = 2159,685(1 + 0.025) P2.213,657 Difference = 2215,657 2,159.65 = ps3.902 (Or Difterece= ae, ference « ul0t{(-+0.005)¢¢ Duterence = AAA0002 1" —11oa5 «pas om.62 an ea epee F=2P n=5(@)= 20 ine P= wns) r= 0141 = 141% compounded quarterly From the nj Fong titre R= (R62)? 1502 Rim R2- 1248 +384 +2500 1BAR= 2253844 Rw 1817.62 m ie = 508) =3 nm o=am ‘Mathematics, Surveying, and 200 November 2002 eer ees oO 670 a 475 +180 aa aa m6 Aw Aye Ae Ast Ay ‘Ax Ve[A(.8) + 1446.7) + 1404.75) + 4(05)]= 12615 mt Volume by prismoidal formula: v= 4 [a+ 44m +24 eas + And) Ve 26350. $6,000 +4.180)+2(4 680)+ 3,150] V= 36,800 cu. m. om Civil Engineering Reference Vol. 2 : 201 ee eee In triangle (d) B : sind 22,9-a9171 2 =9,934"=1 ozs mn In triangle (4) sing = 2 x sinaigtri’ = 2; R= 40835 m Lg, AUSSI =2% aan ist 180" = 1407 m For the 50-m distance, 15 +72+70+695 4 = 07067 meter per pace Average pace = = 7075 paces 50. Pace factor = Se For points Cand 65468-1468 Average pace = SMB 1880 65 pce Distance = Pace factor x Average pace = 077 Distance = 327.91:m_ eae E, Seen SY tenis BEX | Eaten 15 MPa ‘Cracks in concrete pavement due to temperature expansions Eq. Q) daze “ . 4) q af Dynamic Force, Fp = =~ (75.54) = 60.31 Ibs iy H=5 : 2 oaser—11.9 322 H=54cm= 054m ‘ EMo-0) Fa) = Face ae > Fe (60.31)@2) = 120.62 Ibs ‘Consider 1 m length of dam: @ Situation 2 (10to 12) Rym= Rh -OM yxy = U8) = Fu 4) I y= Ry» 2500(4 (12324) y= 360,000 kg = WY Funyha Fe 1000(2 \20% 1) 4 Fig 200,000 kg = : £360,000 x = 360,00048) - 200,000 3) Leen =429m me 1-= 770m ‘Taking level 2as the datum: @ Situation 1 (7 to 9) nen 2B. 200,00 Energy, E: = 0+ 2 = P= _200,000_ 7 pe 920x981 Since E; > Ey the flow is from 2to1. r Energy equation between 1 ___ 2 O ancl A neglecting lose: =216m [Eom Ea} Toe 4 B24 255 Energy equation between? and 1: 2g Y oO ne Pea 16 y= 12 etsy peeomewmot anaes Hydraulic and 212 November ‘Assuming laminar flow (Re <2000) 128uLO apgD* 128(0.29)(50)2 ‘(920)(9.61) (0.06)' (Q= 0.00201 m/s Velocity of ow, »= y= 1016 = o.ooam omy? up _ 0711(0.06)(920) w 029 Reynolds Number, Re == 135 4-< 2000 (laminar flow, OK) =O711m/s Reynolds Number, Ry = Since our assumption is correct, then ‘Discharge, Q = 0.00201 m?/s x 3600 s/he = 7-24,mihe (fiom 2 10 1) Velocity of flow, » = O71 mys Reynolds Number = 1354 1D Situation 5 (1310 15) g=aasms = 10 pa 2 = 05 Vale oftlow, 9= = or 3.056 mys qn67La™® _ 10.67(100)(015)'* CHB BET Gay azo Frictional head lost, ly ~ 3.8m Frictional eel lost ly = Energy equation botween © and @ (Datum at El 0) E\-HL-HE =E; (040+ 197-3.89- HE =0+0+50 HE= 14311 m Power, P= Qy HE =0.15(0.81)(143.11) = 21059 kW Since the pump is above the water surface of the source tank, the pressure ‘at the inlet (at 8) is alitays negative (vacuum), Fi ‘As the discharge increases, the pressure at B drops, To avoid cavitatio the absolute pressure at B rust not fall low the given vapor pressure of 35kPa Energy equation between A and B: (using absolute pressure and datum at A) Ea-hyan™ Ep tt, Ph oz wee fay Ph ez A PZ, 3 Dae ayy Oe ag SMI) ea 8 gat °° Gas 2p” 25 * BT 1S =537m 0= B.692 ny/s (velocity of flow in pipe AB) ‘Theoretical discharge, Q = Av = § (045)? 0.692) = 059 m/s Velocity of flow in pipe CD; ‘Aco v0 Asn tas FOP r= $0457.6.602) op = 2.08 mys £2 Situation 5 (29 to 21) Givens W=S87Ib V=05% Wa dB7lby G=269 Density ofthe mest soil y= “= $37 —a134 byte 214 November 2002 We WAW, | 567-487 _ Molar conten, MC= Fe «TE = SSEEE a o6ts 2 269+ 2601642) tre SESME | rina BEEEAOIE 0.) = 0724 Iomc=sq 269(0.1643)= 5 (07234) ‘$= 0611 = 61.1% (degree of saturation) 2 _ or Porosity, n= <2 = 7B = ass {Situation 6 (22 to24) Intriangle 2: a= 90? }= 90°~ 9369" = 5631" 3 B R=atano = 7,571.66 tan 33.6% = 5,057.76 Ca set $7,571.66 see 33.69" =9,100 = A ‘amge2 Angle of failure plane. 0: ‘+20 = 160° 5631" + 20-180"; O68 Masimusn principal stress, 0, C4 R Maximum principal stress,» = 9100+ 5,047.76 Mona penpals 011416770 pt situation? (25 ta.27) = 20s (Fir =0} 4H = 70 H=Skips (hear force in each pie) Masimum moment Mp =H x10 ‘Mp=5(10) =50 kipstt Axial force at Ds T= 20(10) = 200 kip-ft r=IBft [a3 = (64189) x2» Pan te Py= 2 SO) 4 Po=Sskips Mydrauites and 216 November 2002, Geotechnical Engineering ‘To solve for the other axial force: ee i rox 200 » HMO» syervips 200 200(6) fy 2B. SAD ans = BO 2008) 8 Pa A aa AS Kips: Chick: y=0 Pat Pet Pe Po -200-0 45 + 48.33 +5167 + 55~ 200 =0 (OK) {2 Situation 8 (28 to 30) ‘Qverburcen pressure = pressure of the soil removed to place the footing ‘Gverburden pressure = Dy 120(8) = 960 1b /E2 x (Lton/ 2000 Ibs) ‘Overburdon pressure = 0.45 tons/ft? Beating Capacity from standard Penetration Test (SPT) Allowable bearing eapcity: 44-041 Nex Ps (HPO) Nar cv aet¥ NT ahdad penetration number (ihe number af ows Tere fo deve th sampler an adlitional 300 mm) ‘y= correction factor for overburden pressure ery! ov (=) +6482 (Lino and Whitman 1985) ay 0.7 logs (=) evS2,o'm>24 kPa (Peck, 1974) an effective ovesbutden pressure, KPa Gig conrection factor he groundwater level is within a depth B Seow the base of the footing 2D, +B) z= depth of the ground water table Dy depth of footing: B’= width of foo ‘w= 1if the depth of the groundwater level if boyond B from ifthe ptf th proundinaer eve fjord B om the ‘A. allowable settlement in sn For this problem: Nes or 062 ‘ ‘v= 1 (the depth of groundwater table was nots fd (i aA table was not specified) P.= 2 inches = S0me 7p 4 5041(11.16)(60.8) = 232-44 kPa x —Te2 PS 101325 KPa = 93.72 prix 1 = 4.85596 psf 4 = 4,855 36 pat « 2.000 = 2.43 tonaieg. Factor of safety: "3372 psi 6, =1D, ‘01 mazimum applied foundation stress = Po 218 November 2002 vows conmrenioe se ES Seater (CIVIL ENGINEER Licensure Examination ‘Sunday, ‘November 24, 2002_ (08:00 a.m. 01:00 p.m. STRUCTURAL ENGINEERING AND CONSTRUCTION, SETA INSTRUCTION: Select the correct answer for each of the folloyring questions ack anlzane anew foreach em by seine ‘the box corresponding to the letter Of your choice on the answer sheet provided. STRICTLY NO ERASURES ALLOWED. Use pencil na. 2only. MULTIPLE CHOICE Situation 1 - The 300-Newton block shown in Figure O2 is at rest on the rough ‘horizontal plane before the force P is applied at #=.0. Theforce P = 60t, ‘where P is in Newtons and t is in seconds. 1. Which of the following most nearly gives the time, in seconds, when the Black starts to slide? AL 24. 176 B25 D3 2. Which of the following most nearly gives the velocity of the block, in meters per second, after five seconds? A245 © 16 B. 121 D. 183 3. Which of the following most nearly gives the distance travelled by the ‘block, in meters, alter five seconds? A 166 C185 B 122 D108 Situation 2- A horizontal prismatic beam with span of 3 moter is fixed at the Teft fond andl ee at the right end (canblever), The beam is subjected to Frwnwrard vertical concentrated load of 10 kN acting at a point 2 meters to hetight of the fixed and a clockwise concentrated moment eq to 15 KN seacting at the free end, The beam material has a modulus of elaitity of Sho GPS and its cross-section has « moment of inertia equal to 100,000000 mum 4 Which of te losing most pert gives the fica at he fe ene in rmillineters, f the concentrated Lond were acting. alone? A. 338 C. 235 Bi ©. OM. 5. Which of the following most nearly gives the deflection at the free end in the moment were acting alone?” A238 © 1s B oss D. 38 6. Whidhol the following most nearly gives the masimum deflection, in mm? ‘A326 © 681 B57 D. 496 Civil Engineering Reference Vol. 2 219. ‘Situation 3- For the structuiea showrn in Figuye 0 7. Which of the (olloring best describes the classifica ture A: Staley eis the seca degre ret) B. Unstable . C. Statcally determinate a what lnetereinate th fet degree ch ofthe follawing, best describes the classification of airucture (A Scalyindettunate to he ist degee see) 1. Staticaly- indeterminate t the second degree ©. Unotable DB, Statically determinate 9. Which ofthe following bast desribed the lassi AC Stina ae eae Satay ine Statcally indeterminate tothe sacond degree D. Statieally determininge Situation 4 = The activities, duration and cost under = z mand cost under normal and accelerat conditions fora network diagram are shown inthe accombpansing tale Teds “aaety Tora Tees a teas] Bs FA | Cenng we Getting [10 [3 aso O28 | fecenert of prioournatease [| ammo | 4 | ameo © | Rowivay ad roroge exaveten 7_| 20 [so] aso 050 | ose ottase couse 3 | woe | 2 | taco 2a € | eat iy pp ane aca + [ee [3 | ao Ses [seecnceectioe ca 5 | 2m | 3] a0 10. Which of te following gives the a of aL eyernc om aS See Th. Whichof the fo mea |. Which ofthe following: the activity that is most cos effaiont i eho te elle the cnt tht s met cost effin convening AY Activity C © Acuity A B Actity F D. Activa E 12. If the maximum budget is P22,600.00, whith of the follow neath Bivethe reduction ime rumberof dys iy complee ee ee 3 a? ne Situation 5 ~ The continuous reinforced. conctete igure 05 fi subjected ta urorm service dead ond of 16a) sel sae tne oa of 32 KN/m, resulting in the bending moment diogram shown, Twent percent of the lve Tend will be sunined iv nature, we 80% wil be apple ood ntendy, The creme stnyih fc 2 MPa The elasticity of conc wen by the expression f= Sari) and the modulus of rapture f given by the exprestien ff =O 220 November 2002, B 14 Structural Engineering ‘and Censtrection Safa, The rlvan proves of he 12 National Sructurl Code of ie ilippines are: 26 Unless stifiness values arc obtained by @ more comprehensive nays iponedime debctton shall ‘be compaied wi te fective its of inertia as Follows, but not greater than I. “fey [fe fr ieesG eee Eq. 9-7 where: maximum moment at stage deflection is computed ii moment of inet of grots concrete section sboutcenvoial a reglectng reinforcement Jg= moment of inertia of cracked section transformed to concrete y= distance from centroidal axis. of gross. section, neglecting reinforcement, to extrenve fiber in tension id uous members, effective moment of inertia may be taken. Secu mae ema : ‘Unless values are obtained by # more comprehensive analysis, slikdcnal ong erm defection resulting fon ewe. and vinlage of flexural members shall be determined by multiplying the immediate deflection caused by the sustained load considered, By the factor «4 1 Aaa Eq. (9-10) where p= reinforcement ratio for nor-prestressed compression reinforcement fat midspan for simple and coninuous spans, and at support for ilevers time-dependen factor for sustained loads to be equal to Syears oF MARE or erwise2D 18 months. “TA, 6 months ele 3 months Th IWbich fhe folowing mort nearly gives th fective moment of inert at supports (maximun negative momen). a eee case B. 0.00686 m4 D, 0.00815 m4 - |Which ofthe foloveing most neasly gives the effective moment of inertia for he continuous member. ‘0.00586 m4 B. 0.00979 m4 0.00815 m4 D. 000753 m4 Civil Engineering Reference Vol. 221 TB. It the innaneancour deflection due jo the combed servi dead pra load is'3 mm which of the following moot nearly gives te adc deflection (in addition to Uhe initial deflection) after’S years, ander the Haine eating. aan 12m ‘A. 467 mam, B 840mm. D. 735 mm Situation 6 ~ As shown in the Figure 06, a secondary member inclined at an angle (= 30° i composed of two plates cach 250 mm wide and 50'mm thick. The secondary member is joined by bolts to the 100-mm thick main member. It an be assumed thatthe bolt capacity i the critical parameter. ‘The load P= AMEN, The thnber is 80% stress grade Apitong Gpecies group il). The allowable foods on the bolt (double shear) are given in Table OL The 1992 National Structural Code of the Philippines provides that the allowable load gpthe Belt in connection a! an angle ef nad othe gran beeen to 0° shall be computed from the Hankinson Formula a Pain" 0+ Gear 6 where P is the allowable load parallel to the grain and Q is the allowable Ira pe endicularto the grains ee B 16 Which of the following most nearly gives the minimum required capacity of ane bolt? ‘A. 73 KN C487 EN B. 820kN D, 615kN. 17. 1 failute vill occur in the secondary member, Which of the following most ‘nearly gives the minimuen diarseter of the bolls? ‘A Dram C 3mm B 16mm D. 20 mm 18. {failure will occur in the main member, which of the follawing most nearly sives the minimum diameter of the bolts? ‘A. 20 mm 5 2mm C 16mm D. 3mm Situation 7 — Two timber metnbers, exch 400-mm wide and 75 mm thick are joined fogether using 2S-anm diameter bolts as shown in Figure 7. The diameter of the bolt holes és 2 mm larger than the diameter of the bolts. The timber is 80% stress grade yakal. “Tho allowable strest for structural timber of Prlippine woods are given in Table G2. The allowable loads on on bot (Gouble shear) are given in Table O1. No adjustment is required for load uration, moisture, femperature or group action. 19. Which af the following most neatly gives the net area of the timber member, ‘A. 24,000 mn! © 18500 B. 46,900 mm? D. 37,400 mnt 3 Which of the allowing mast nearly gue he minan oadsuch thal the allowable tensile stress on the member is not exceeded. ‘A. O90 EN © 412EN B. 585 RN D.575KN 222 November ease cares —$—$—$ $$ _$_$£§____________—_ Zi. Which of the following most nearly gives the maximum load such that the allowable load en the bolis is not exceeded. ‘A. 284 KN KN B. 155 kN 1D. 390k ‘Situation 8 — A Bill of quantities is to be prepared for the square isolated reinforced ‘onesete columit and footing shown in Figure 08. The swell facta forthe backfill is 120%, 22 Which of the following most nearly gives the quantity of excavation, in ccubie meters: ‘A 0218 © 0465 B 038, D.os70 2A. Which of the following most nearly gives the quantity of concrete, in cubic meters: ‘A. 0568 0200 B, 0372 ‘D. 041s 24, Which of the following most nearly gives the quantity of formwork, in square meters ie, 415 B 256 D. 580 ‘Situation #- A rectangular column is shown in Figure (9, It is acted upon by the ‘allowing eae load due to dead load = 450 kN ‘Axial load cue to live load = 500 KN Bending moment due to dead load = 134 kN-m Bending moment cue to live load ~ 150 kN-m ‘The column isto be analyzed and designed using the strength method, ‘Use b= 400) mm, = 600 mim, and y= 075. 25, Which of the following, most nearly gives the eccentricity of the column in millimeters: ‘A. 300 cas B 288 D5 t nearly gives the area of the minianuan requires! stool reinforcement, in millimeters: ‘A. 5250 cc. 3820 ih of the fol vensly gives te of th ™ Which of the following most gives the minimum diameter of the reinforcement in millimeters, pt A GB B36 D. az 224 November 2002 wo ew Structural Engineering ‘and Constraction fenton tenes a CC oP oa =oh UGM O Figure 09, Figure or 5| ale cone sf. 305 10055 001 remoxee 226 November 2002, and Construction = 2 Ft i g - & : a : 3 5 : 3 : E 2 T= 0on7ism! Ta 0.00573 = poe ee : sraprons unas 2 muEBeS wuues. panes aT Mose seruccural 228 November 2002 eae Solutions to November 2002 Examination TABLEGE Werking Stresses for Visually Suse Graded ‘Unseasoned Sractural timber of ne Woods Sen ses ae (Di Situation 1 (1 to 3} Bancirg and and Cri Cargeession Tarra oe tag) arms) | “tran | “oan” |"boan |"? Belore the block starts to ‘slice, [b= jip= O14 a = : 5 B N= 300+ P sin 50° om oe a, as we = 300+ (80 4) sin 20" S| 2-8 | | wR rea a | & [oe | | fr OAN=120 + 160 ge | as | so | ip | ie fe | as | ue | Ge | de Go| ge | ee |e | & When the block stars fy ove, | ae | Be | | ie Pees (601) cos 30° = 120 + toe t= 2252 seconds When the block moves, END“ pew 02 {f= 02N = 0.2(200 + 401) f+ 6 Fx=0) REF + f= Pecos 30° 300. on (80%). cos 30° - (60+ Bt), ex 20tii-1sa~ 2; an nner) Integrate both side: = LOUIS 1.96284 C Whent=2.2528, 0 =0 0 = 1,00195(2.252)*~ 1.962(2252)+C C=-0.663 = LOO185 1.962 t= 0.663 When t =5 see = 1.00195(5} - 1.962(5) - 0,663 = 14.58 mys Dh aa stones taiae-nasse 8 vk (1.00195 2 ~ 1.962:¢- 0.663) a ‘Structural Engineering 230 ‘and Construction soxan-esere-o4 | $= 0:354(5)~ 2252) - 0.98106" - 2252 ~ 0.669(5 - 2252) S=1656m © Situation 2 (4 to 6) 20/6 ——_Contgate bean on concrete ld Deflection due to concentrated load, 5; = i’ 46.6kN=m? 8 = [¥a(2}20/ EN] x 233 = =a 46501000)" 2 saan (20,000) (100,000,000) Deflection due to moment, f= Ms! S25kN—m Sqm [5(15/ ED] x15 = 2 87501000)" "2" 0001100000000) Maximum deflection, = & + 8: 233 + 8375 = 8.705 mm = 3375 mm Civil Engineering Reference Vol. 2 23: Giron To i Sivations (7 to) Note A fixed endl has 3 reactions roller support has one reaction ‘hinge support hao two reactions ‘An internal hinge gives one equation (zero moment) Figure ) Number of reactions: 3+ 1+1.=5 Number of equation = 3 ‘Thus, the stricture is indeterminate to the (5-3 = 2) second degree. Figure (®) Number of reactions = 343 =6 ‘ Numberof equations = 3'¢ 2='5 ‘Thus the srushue ig indeterminate to the (6=5 = 1 first degree. igure (): Number of reactions = 2414255. Number of equation 3+ 25 ‘has, the structure ts statically determinate 2 Situation 4 0 to 12) Parti Network Diagram (Normal Conditions) =a ‘The paths from 0 to 4 are: Path D-P: Duration = 345 =8 days ‘A-C-E-F; Duration =5 +7444 5=21 days Path B-E-F. Duration =5 ¢4.+5= 1adays ‘Thus, the eritisl path is Path A-C-E-F Normal Cost = 1,000 + 3,000 + 12,000 + 3,000 + 1,000 + 1,000, ‘Normal Cost = P2i,000 Structural Engineering ‘and Construction The normal cost (from Part 1) is P21,000. Since the maximum cost is to be 22.600, we may accelerate some activitios with an adeltional cost of 1,600.00, Accolerate items along the critical pth ‘Activity A: (Masmum crash me = 5 ~3™ 2days) Crash cost — Normal co Cost Of emis ral drat 1600 —1,000 5-3 Activity C: (Maximum crash time =7-5 = 2 days) Ce eecniy = ee Activity E: (Maximum crash time = 4-3 = 1’day) = P500.00 per day Cost of crashing = = F250 00 per day (Cost of erashing = Activity F: (Maximum crash time= 5-3 =2 days) Con sfemshing= 220-209 « punon pe day Th mot con fete ct rth at ar hen thar he at euialcnehing. cng teased le weston elise each any cwile Awe tiers For a total of P600, Activity C can be erashed for 2 days for a cost of 2 250 = PS00.00, Activity A can be crashed for 2 clays fer a cost of 2 x 300 = 'P600.00, and Activity F can be crashed for I days for a cost of 1 = 400 = PGOOL0O, “Thus the total cost of erashing ss PI,500.00 (with savings of 100.00) [For most economical project duration, the project may be crashed for total of 2+ 24 1= 5 days. Thus, the reduction in the number of days to complete the project is Says. 12 Situation 5 (13 1015) ‘At supports: (ie) [- (2) ‘M,= 202 kN-m. f=07]fo =07 {72 =290 MPa 1, = 0.00715 m¢(as given) y=310 mm, __ 2903(0.00715 10008) 0 ) ons + HeS]] oms73) 1-=0.00878 m¢ Me = 66,956,290 Nemm = 66.956 kN-mm For the continuous memo (areamaaiel oscreasee Clouser “Jase somes (60.5527 ) Atmaximum negative moment (at support) = 000578 mxt ee Solving for [,at maximum positive moment (at midspan) 2.903401 t= (at) sn «[1-(2%)'] ose 1, 0.00793 ie See Syccal Additional Jong, term deflection = long term deflection x i = 0 (since there is no compression reinforcement at mids 2 (after years) fare aes Solving for the long term deflection under sustained loading ‘Instantaneous deflection =5 mm (given) instantaneous loading = 16 kN /m + 32KN/m = 48 kN/m structural Engineering civit 234 November 2002 ‘snd Consereetioa Reference Vol. 2 235 _——— ‘Sustained louding = 16 + 20%(32) = 224EN/m FQ Since deflection is directly proportional to the load Psin® 04Q cos 6 Seen 5. PxQ 27233 eee Pain®30"+ Qcas 30 ‘Additional long term deflection 2393 <2 = 4.67 mim Situation 6 (16 to 18) “Two S0-em Pamsin Minimum requited capacity of one bolt= $= = 7.33 kN For failurein the secondary member: Length of bolt in member, use 50 2= 100mm Load in each bolt (parallel to grain) = #4/0 = 7.33 kN Load perpendicular to grain = 0 ‘From the table, the minimum ciamoter is 16 mum ifn i For failare in the main member: ‘Length of bolt in member = 100 mm Load in each bolt at an angle to grain = 44/6= 733 KN. Angle to the grain, = 90° Try 13-mm bolt with P = $58 and Q~4.07 588x407 he a < (not BaBsin® sorsagreest ap ~STOOEN <733KN (not OK) ‘Try 16-mam bolt with P = 8.47 and Q=5.A1 =7A2.KN>7.38KN (OK) Bay 5a1 ee BAa7 sin? 30"+5.41 con? SOP Minimum diameter = 16am {D Situation 7 (19 to 24) Part Net area ~ Net width x Thickness Not area = (400 » 27:3)(75) = 23,925 mam? Part 2 P= Allow tension parallel to grain x Net area Allowable tension parallel to grain = 24.5 MPa (Table 02) P= 28 5(23,925) ~ $96,169 N ~ 536.165 KN 236 November 2002 ‘Part 3: (Yakal belongs to Species Group 1) Bolt diameter = 25 mm ‘and Construction This is.» two-member joint (single shear) of equal thickness. For thin condition, one-half the tabulated load for a piece twice the thickness of fone of the members shall apply Length of bolt= 2 75 =150 mm Allowable tension parallel to grain = %s46) = 173 P=173x9~155.7 KN (D Situation 8 (22 to 24) ‘Quantity of excavation: V=06(06)(0.1 + 0.25 + 0.6) ~ 0.342 me Quantity of concrete: Footing 0.6(0.6(0.25) = 0.09 m? ‘Column = 0.15(0.2)(0.6 + 3.05) = 0.1055 ms ‘Total quantity = 0.09 + 0.1095 = 0.1995 m? Quantity of formwork: Footing = none ‘Colm = (0.15x2-+ 0.232)(06-+ 3.05) = 256:mt ‘Total quantity = 256 m? {1 Situation 9 (25 to 27) Pa = 1.4(450) + 1.7(600) = 1480 kN P, | 1460 p= Fe = MO A 1428 kn 4 OF My= LA(134) +1.7(050) = 442.6 RN-m. 426 M26 6 078 kN OF Be Beas eionase me Reference Vol. 2 237 20 95 i #2, | 07(2114.28%1000) Ay *=00(800) (632.2850 [g600)(4100))(600), = 617 MPa 3.07 MPa ad ie Ee oe a eo ao 8 a8 WE ae (Pita «eta wr ‘From the interaction diagram, py = 0.017 Aw Fen Ave™ 0.017(600 « 400) = 4080 max? ‘The minimum number of bars for a tied column is 4 Solving for the bar diameter with four bars: Aus 02x 4 = 4080 D-%mm ‘Civil Engineering Reference Vol. 2 239 ‘Seat Now CIVIL ENGINEER Licensure Examination Wednesday, May 14, 2003 06:00 asm. 01:00 pm. MATHEMATICS, SURVEYING, & TRANS. ENG’ SETA INSTRUCTION: Select the correct answer for each of the following questions Mark only ene answer for each item by shading the box corresponding, to the letter ‘of yur choice an the answer sheet provided. STRICTLY NO ERASURES ALLOWED, Use pencil no.2 only. ‘MULTIPLE CHOICE 4 1. log, 122.262, find the value of log, 3. A075 c "e.05 5025 D. 10 2 What is the sum of the numerical coofficient of the expansion of (+ E)* ‘A. 6 cn B ss D. 74 3. gand bare positive numbers. ¢,~4,b... form a geometric progression. 14,2, i... form anarithmetic progression. Find the value ofa Ae C38 ecu vas alae i) (2) =7-4 4. Find the value of xin the equation (r+ yi) (1-2) =7 -a i sxyuation (+ y) +29 B D2 1 3 5. The sides of a triangle are 18 em, 24 cm and 3 cm, respectively. Find the lengths of the median ta the 24-cm side, in em. 4 4 24 CBs B29 D204 6. Acirede having en area of 224 sq.m. is inscribed in an octagon, Find the area of the octagon. A. B865q.m © 235359.m Bo 28 20¢ D, M6? s,m 7, Bind the radius of the spherical wedge whose velume is 12 cu. m. with a central angle of 18 radians. 236m ©. 252m B 273m D. 215m 8. How far from a vertex is the opposite face of a tetrahedron if an edge is 50 emlong? ‘A. 38618 cm B, 40825 em, D, 41.214 cm. 9, A truncated prism has a horirontal triangular base ABC, AB = 10 em, BC = A2cm and CA = B cm. The vertical edges through A, 8, and C are 20 cm, 12 cm, and 18 cm long respectively. Determine the volume of the pris, in A. 661 ©. 68 B, 558 D. S74 240 May 2003, 10, n 2 B. 17. 8 a Mathematics, Surveying, and ‘Transportation Engineering Uy wsing Pappas Theorem, determine the vole generated by revolving the area in the first and second guaclrants bouneled by the ellipse 4x? + 25y 100 and the X-axis, abou the Keaxis. ©. 9535, D. 8378 Find the ratio of the length of the minor axis to the length af the major axis af the ellipse 9x2 Toye 1d 0. AG. = © ous B. 062 D, 058 What is the radius of the circle 2+ y2-+ 4 10y-92 = 07 Az cn B.S D. 10 Points CG, 7.2)and D (4, 1,6) are7.28cmapart. Find the value of ‘A 3am © om B dem D. tem ‘What isthe total length ofthe curve r= 4 sing? A 8s Cm Bex Dit What is the derivative with respect tnx of 2cos (2+ 23)7 A. 6rtsin(2+ x) C Bante +3 B -6stsin 2 +2) D. 2sin (2+ 2%) What is the curvature of the curve y2 = 162 at the point (4, 8)? A 0.0K C -0.065 B. 0.088 D, 0.033 Using limits Oto 2/4, find the integral of sin 2x dx. A037 C 0322 B 0432 D. 0.245 Cost of machine = 140,000 pesos useful life = 8 years; salvage value = 10, 000 pesos. Determine the 4 year deprecistion using the deuble-dechining balance method, in pesos RARE © page BI7312 5. 15634 Terms of sales: P60, 000 payable in 60 days or P57, 500 payable in 15 day ind the equivalent annual rate of simple intrest f paid in60 days A 258 C u47% B 348% 329% Find ie pregent value in pest, ofan anny of 2300 poses payable smuaally foc 8 year, with te iret payment athe tnd of 10 years, if mony A P79354 C Pasa25 B Pras D. PBka2d around an unbanked circular curve ‘A car weighing 1000 kg rune at 60 Ky fiction on the tires should there be to wvith a rads G€ 100 me What Force ent the cor from sliding? MS 29 N © 278N B. 2671N D. 2568 N ‘A compound curve has a common tangent 520 m long. The first curve passing through the P.C. is a Curve with a central angle of 50°. Find the radius of the second curve i its central angle is 35° A, 1.0843 C1137 B 1268.2 D. 12083 civil Reference Vol. 2 241 3. A structure employed to direct the flow of current in rivers witha view LSS oan noes caren and erosion and the carrying away of ver banks nt © Preventscour sae cm ow a a f 2:1, od of 10 tn ne opel 2 a wate acs a SrFope i ee na eer ama een: eee eat of oe A car driver travelling at a speed of 65 mph approached a hazard Sari 9s Sn ee ce gh ae «Mt i Gece etl, Dc ne Baa D.27 Lot ABCDEFA is a closed traverse in the form of a regular nw exch side equal to 100 m. The bearing of AB wN25°E What te bearer, of CD? A. 535° E, ©. Sane D. $40°E B S4s-E The following interior angles of a trangular the same precision. What is the most probable val ie ae race on ae iene 5 B 7 6 ic 63 2 A. 4035 4057 B. 40.16 ‘D407 What is the central angle in degrees of the curve whose raclius is 200 m and the distance ofthe midpoint ofr curve tothe 1 1d 20m? ‘a a8 ca 1 Beaman Dat wericontal axis ofa transit wae inclined at vith the horizontal de to oneadjustiment Tho fst sight had a verteal angle of 30 the son Determine the errorin the measured heizontal eagle. tS AT 46" eo ease B 79a Dwar “END as Jog, 12= 2.252 x2 ay 203 log. 3 =lop3=1 “To.get the sum of the coefficients, set all vasiables to one (1) Sum of coofficients of (a+ HY = (+1) 64 Mfa,-4,b form aGPz Common ratio= + a a=16 0 Eq. (1) bem-1d Eg. (2) Substitute bin Eq. (2) to Eq, (1): a(2e- 18) = 16 21d 16 st 7a= 80 (Bett) =0 amanda +8 ete Two comple: nambers ate equal their comsponding rel and econ parame equal te-at ire c# difa~ cand b= (e+ y)@-2)=7- ai xo Bet yi 2y a7 = i yo 2xi + w-2y(l) =7-4i (92) Qr-yin di Then; x+2y=7 > Eq.(l) wa-y=4 9 EG) Note: =-1 ‘Add (1) + (22 x t2y= 7 Solving for 8 in triangle ABC 18 = 38 + 282 2G4)(24) cos.O o-3037" Solving for m in triangle ADB: nfm 342412 = 2(84(12) cos 2037" m= 281 cm. é ® ‘Area of circle =r? = 224 mt rm Bt 0 ~260/8~ 45° ' cos (0/2) =1/R os (452/2) © B.AMM/R R= 91397 Aocngee # 8X Astrg Acctagon ® 8 = Vif9.13977 sin 45° Accigon = 236.27 mi? ‘The volume of a spherical wedge of central angle @ is direct proportional to the volume of sphere, which is a apherical wedge of central angle 2x. Aor? 2 3 Neste 3 Be oe 8 2 ae a ara bm qa? (af2h = 8/2 h~ JE e | > Formala = [F G)=a0825em 244 May ZOOS as m0 ou ‘Mathematics, Surveying, and ‘Teaneportation Engineering Votume asx 22 Solving for A, (tlamgle) sing, Heros Formula As= {sG=0)6-BE-8 a arbre. 12+8+10 ae 2 sn15 2 As= {(15)G5= TAVIS BHO =} z " ‘Ay = 38,686 cm? Volume = 39.686: B9eT2 18 -epratec ipse: 4:2 + 25j2= 100 ear. a oor a=Sandb=2 ‘The solid formed is 4 prolate spheroid fy where em b= 2 4 4 jame= sate = 4 ate Vokumnem Snake = 5 Vokeme~ 4 =(\2)= 8377¢eubie writs [No mced to use Pappus Theorem as suggested inthe problem Sut Loys- 148 =0 Gxt 16yt= 14 ‘Divide both sides by 144 2s 2+i-1 9 a 16a=4; Majoraxis = 20-8 Bead be: Minor axis = 2h = 6 Falivof minor te mires © = 075 245 ——— ov att yd d= 10y-92—0 Bedeedey (a+ 22+ (y-5)= 121 Radius of the circle = 11 units o13 a ‘The distance d between Pax. yy, 24) and Pe(a, ys 23) is given as: ean Gewese ‘Then, 726m (a=5p4 (1-74 (6-27 (6-315 ‘ =i ra624=10and2 ou yA tam The curves r= asin @ and r= «cos arecircles with diameter a For the given curve, «= 4= diameter, Total length (perimeter) = nD = w(4) = 4x mee fu Note: 2 cog u = ine At a = 5 A peo @> 292i bain a+ (909 eg ir eta Mathematics, Surveying, and 246 May 2003, ‘Transportation Engineering ie The curvature of the function y = /{x) atany point is given by: ae N, “Ten? i = 16x . awaits yet @aay~Ser x ae 8 vous --D aon 0.125, arvature,k = 2125 — - gone ary ov ss fore Let 2x0 de Yad when x = 0; 0= 0. when x = 6/40 = 1/2 [Feta - fiatonga =%4 feintous = 4x 80 = Cores 0245 aw y= BVy-BV, av,=#c(1-2)" sv ‘ avy=tagons-2)" «tasers : ov1am{1-2) F050 <4 = PS9,062.50 - P$4,296.875 = 14, 765.625 Civil Engineering Reference-Vol. 2 247 ee ae ; = From the cash flow + diagram shown: F= r+ ri) 57500 000 57,50011 + ras/360) — L_t=sen F 4 r= 03478 = 34.78% 20 ‘This is a deferred annuity consisting of 8 annual payment of P20,000 with, ‘the first payment to start after 10 years. ote te eae es ' ao att aot aot 207 a ot ' cee a Ss ' a8 at aan The present worth for this condition can be expressed as p= MGxi" 11, 20.000{04005)*—1y = : = P83928.89 aay (12005)"" (005) the Friction force must he iprester then or qual tothe Fore ‘To provent the car from sliding | entrifugal force = 60 kph = 16667 m/s (46.667)? 00) Friction = CF= 2778 N CF = 1000! =278N Mathematics, Surveying, and Engineering 248 May 2005 ‘Transportation! oz Di-3" =50" Re3se EP= 520m, Th aRD O° ae Ri). po. 20 = SD Rin 381.97 m Ti Ri tan (h/2) Ty" 381.97 tan (60°/2) T,= 816m EP=T)+Te= 520m Ty=520 178.116 = 341.884. T= Ratan (o/B + 341.864 = Ratan (5/2) Ry=108.32m rey hor barséer built out into a body of water to shel Jety «wall or other barrier Bult out into & water to shalier a " hatbor, protect a shoreline Frm erosion, of redizect water carrents Levee 1 a Ratural embankment slongside’a river, formed by sediment during times offloading 2 Ok artifical embankment alongside a river, built to prevent iooding ofthe surrounding land Mote ~ construction tunneling machine: a tachine designed for boring ‘through hard materials such as rock ‘Areas Alt Art Ast A 31.7 = VASY(ZA) + Vala) + 48) + Vax) (5 + 24) + ¥595)(1.2) x= 264m Civil Engineering Reference Vol. 2 249 — Qa Distance trv dung the PIE tne, $= = 65 mi/hex (5280 8/ mi) x (1 mj x nm Raat See (1/328 9) (1 kin / 1000 m) S=722 5. na imo, t= Sm FOZ a. see is (29,065 enna of AB =N2°E Azimuth of AB = 205° c= 360"/6 = 60° Azimuth of CD: = M5" +20, 35 Bearing of CD f= 300"-325° f= 35" A-S35°E. z (a2 Sum of interior angles = 41° + 7° +63°= 181° Error ~ 181° - 180° = 1° = 607 ees A Ee x60 = 34.0158 Total = 60 (OK) MEV of angle A =-41° - 13.8461" = 40.765" Radi, R= 200m The distance from the midpoint of the curve to Pi. is the extomal distance, It may also be refered to as the shorts tence from Uh cure fo 250 May 2003 Mathematics, Surveying, and Extemal distance B= 142m E= R{sec(/2) ~1) 14.20 = 200fsee (1/2) ~1] sec (I/2) = 1.071 ces (1/2) = 1/1071 =0.9857 1/2 s2ns70e I=4196" Error in horizontal angle when the axis of the telescope ts not horizontal, Bror,£=etan® Where: ¢ = angle of inelinatian of the telescope axis, usu seconds (= observed vertical angle of the object E=crrorinhorzontal angle in minutes or seconds. iy in miniites or When two observations are made, the total erar may be expressed as: Error =e (tan - tan 62) Where: 1; = first vertical angle = 50° (th second vertical angle = 20° Error = 4.[tan 50" tan (.30)] = 7.076415" Error 7458" Civil Reference Vol. 2 —— EE os Distance: ree dene ey time, S= ot a Masry early dala a et (Lk / 1000.:m) 22 $ m2 2 Bs time, =248 seconds Bearing of AB = N 25° Azimuth of AB ~ 205° a= 360"/6 = 60° Azimuth of CD: =205+2a =325° Bearing of CD p= 360° 325° pa3s BS35°E ma ‘Sum of interior angles = 41°+779 + 63°~ 181° Error = 181° «180° = 1° = 60° fe [aT Teron l= |e ieee S| | Sum =25/20 | Towi=6r OK | MPV of angle A= 41° - 13.8461’ = 40.769" Radius, R= 200m 2s The distance from the midpoint of the curve to Pi. is the external distance. it may also be referred to a tho shore ditance from the curve to © 250 May 2003, External distance, E= 14.20% E= Rsec(t/2)~1) 14.20 = 200see (1/2) - 1] sec (i/2)~ 107 cos (72) =1/1071 = 0.9997 1/2=2n5798" In an96 Error in horizontal angle when the axis ofthe telescope is not horizontal. Error. B= etand Where: '¢= angle of inclination of the telescope axis, usually in minutes or seconds '8= observed vertical angle of the object E= error in horizontal angle in minutes or stones ‘When two observations are made, the total error may be expressed as: Error = (tan 0; —tan 03) Where: 18 = first vertical angle = 50° ‘82 = second vertical angle =-20" Error =4 [ tan 50° - tan (30°}) = 7.076415" Error = 74.58" Civil Engineering Reference Vol. 2 251 —— ‘SeatNoz CIVIL ENGINEER Licensure Framination We 7 May 14,2003. 02.00 p.m. - 06:00 p.m. RAULICS AND. ICAL EERING SETA INSTRUCTION: Select the correct answer for each of the following questions _ Mark paly one answer for sach fem by. shading. the bax corresponding io the leer df your choice on te anayer sheet proved STRICTLY NO ERASURES ALLOWED. Use pencil no. 2 only SITUATIONAL ‘Situation 1 A block of wood 0.60.m «0.60 m x fi meters in dimension was thrown ino the water ae eats wil 16 m procting above the water surface. ‘The same block was thrown into a container of a liquid having a specific Fetely of 0120 and it flats with 0.14 m projecting above the surtace, 1. Betermine the value of h ‘A. 067m. © 054m B O76 mew o8™ 2 Determine the specific pravity of the blo Bineed gee © 0867 B oss D. 0751 3. Determine the weight ofthe Block, ‘A. 122 kN azz 81509 kN Di 0851 KN Situation 2~ A trapezoidal channel has a Bottom width of 6 m and side slopes of 2 horizontal to 1 vertical. If the depth of flow is 1.2 m and the flow 1s 240 m/sec, 4. Compute the specific energy. A. 1am c1mm B 258m D. 102m 5. Compute the slope of the channel if n= 0014. ‘A. 0.00023 C. 0.00078 B. a.000i7 . 90094 6. Compute the average shearing stress atthe Boun: Sone Sn B 55Pa B.s2ra Situation 3 ~ From Figure O01, itis shown that tho gate is 1.0 mh wide and is hinged at the bottom ofthe gate, 7. Compute th hydrostatic force th KN acting on the pate Ae cian” 8 1590 eh net 8. Compute the location ofthe center of pressure of the gate from the ings, 06m Fc osm 3 8 078m D033 civil Myaraulics and Engineering 285 252 May 2003, Geotechnical Engineering | Reference Vol. 2 3. Determine the minimum volume of concreie (unit weight = 236 KN/m) ‘A. GIN (well graced gravel) GP (poorly graded gravel feed i heop the gate nlosed poston, " % SP (poory|pradodana), SW ed aasah ‘A 062m ©. 0266 me 8 0380 me D. 03M me Sieve No Diameter amy [Percent dee ee meee aes eae : See Genet pipelines 2 and 3, where it meng agin at font Ca form & + as sgl piping upto point D. Pipsinos 1 anal a in snes cnewcton > ae —— ‘whereas pipelines 2 ard 3 are parallel to each other. Ifthe rate of flaw from +t oe = A 0 82 1 Mers/sec and assuming f= D2 fr all ppc, compute te 3 aS s erie: [—s 025 i a Pipelines | Length (mi) | Dismeter (am) 30} ot T 00 200) See —* ST a ee tuation 7 ~ The following data was obtained from laboratory tests for a 3.200 2007 Cohesive specimen: mokiture content, , was 225%, Gy = 2.60; and to = 2300 ‘0s {determine the approximate uri weight, a sample having a mass of 204. twas placed in a 800 cr container with 382 a2 of water required to il he 10,_ Rate of flow in pipe 3 in ters sec container ‘A288, cin 29, What is the nearest value to the Total Unit Weight of the soil sample in B 231 D. 378 Rm ae 11, Rats of flow in pipe 2 Inters/sec 20; 7 tea) c. 556 B 2158 D.wa 8 943 Dees 20 What is the nearest value to the Dry Unit Weight of the soil sample in. 12 Total head loss from A to D. kN/m? ‘A168 m © 212m Aus cue Bo15m D, 288m B 161 D. 152 21. Whatis the void ratioe? Situation 5 - A tank of carbon tetrachloride (CCl) has a mass of 500 kg and a A. 0678 C. 0866 volume of 0.315 a, S82. D. 0784 13, What s the weight of carbon tetrachloride in KN? perry C580 Situation § - The ground water Jovel in a thick, wery fine cand deposit is Iocaled B 450 5. 300 20 mbolow the ground surface. Above the free grocind wale lie the sant 14 Whats he mass density in kg/m? i catty capi ston. The uit Weight of the sara soe i293 AL © ass N/m B 1626 D. 1587 22 Whats the otal ses in kPa an @ horizontal plane ocied5 m below 15, What is the unit weight im BNY/am*? ground surface? 1855 © 1629 A 7562 © 8236 Biss? D. 1482 B a373 D. 9135 2B. What isthe pore water presoure in kPa at this plane? tuation 6 - The table below shows the laboratory results of the sieve analysis A 3333 C16. ‘ofa sample. Plot the grain size curve of the sol inthe atached Graph 11-1 B 2453 D. ata 16, Determine the nearest value to the effective size 24 Whats the effective vertical sress in kPe'in plane A? 4. O45 C080 A 7212 C. 6683 B 030 D. 01s B 5463 D. 8796, 17, Determine the nearest value to the coefficient, of uniformity, C rien Co on 9 = A dense silt layer has the following propartics: void ratio - 040, BO DS effective diameter diy = 10 pm, capillary constant C = 0.20 cm?. Free 18. Classify the soil according to the Unified Classification System, Use the water level is 80 m below the ground surface. ae ‘tached sheet UCS 11-2. Mydrautics and 254 May 2003 Geotechnical Engineering 25. Find the height of capillary rise the silt, Capillary dseis given ashi C/ (ex ay). ‘A Sm 3m Bam Dizm 26, Find the vertical effective stress in kPa at 5m dey solids = 265 KN/a? and that the soil above ‘ground surface & partially saturated at 50%. A176 C. ise BBM D. 27, Find the vertical effective stress at 10 m depth. Assume: unit weight of solids = 265 KN/m and that the soil above the capillary action rise and ‘ground surface is partially saturated at 50%, ‘A, 106 Cis B 378 DB. 2a Situation 10 - The following, data were obtained from the Atierberg Limits test for Ascume unit woight of igen ee 28, Whatis the plasticity index of the soil? A. 215% © 19905 B 552% D. 054% 2, lsh in tt moisture content oft sll 20%, wha the igi index A084 c 045 B. 060 90, What could be the navure ofthe soil? A dense © liquid B, byte solid D. plastic Situation 11 - A footing 6 m square carries total load, including ils own wei er TdT aie basccl tes foeing kha dept neler Gee pot Surface. The sail strata atthe site consist ofa layer of stiff saturated clay 27.8 im thick overlying dense sand. The average bulk density of the clay is 1.920 kg/m? and its average shear strength «termined from undrained triaxial ‘st is 130 N/m. Given is Terzaghts ultimate bearing capacity for square footings: a= 13 6 N.# rD/Ny+ OAD YB Ny SI, Determine the gross foundation pressure in KPa, ‘A. 162 455 B 393 Dw 52. Determine the net foundation pressure, ‘A, 221 © 499 . 957 Di. 183 33. Calculate the factor of safety of the foundation agsinst complete shear failure under the undrained condition (both gross ard net). Side cohesion ‘on the foundation may be neglected. A. ES 82 ©. Spm 63 Sm" 103 Fan 75 B. Fp 53 D. Fie 78 FSm™ 8 Fez 98 ivi’ Reference Vol. 2 Situation 12 ~ Consider the straied sll depostshown in Figure O02 255 Tr yrdraulic gmdient sequal =) p= i 2 quantity of flo in each layers added to make the total flow g Note: Darcy's equation 9 = ki and q = of fhe. io, coefficients of permeability of ‘the rection. Consider 1 unit with. bby, are the: individual layers in the horizontal SA. Derive the expresion for the equivalent coeficient of permeability in the Selman tery (Dy Kiog = Hi [itn Ha + hege = Ha + 5. + dane x Hy | © fect sine Te ilaes me deta pe oan a Permeability in the horizontal divecion, ns eetuvalent coefficient of A. 6236 105 © 7257x108 B. 8005 x 104 D. 5061 « 108 36. If=0.70, determine the total low g tn cm/sec. A. O57 C0674 B osI9 D. 0758 Graph 11-4: Grain Size Curve oy Particle Diameter, mm 256 May 2003 coctaieslnoeioeine MCtcren’ her a® 257 UNIFIED CLASSIFICATION SYSTEM (USCS-11-2) meme aeons iy er (dBl> 4 G> Bafeban Babee 102 He ne meg tm emer Ct Ha See eee 5 i comes [eae pp eee eoiae is paeneas f a tt cemeneiie** ieee. 4a aminpeny [mec i > * a ii pelt a) a g> | * le i Lt HL a i i a i ae a I obs ay Figure 002 END + 258 May 2005 Tr Solutions to May. 2003 Examination situation (1103) InWeter ca > Drast= Sissi. i n=0.18 h-018= Samet y Swoait=h= 018. 3 Bq. (1) In another liquid Soot 09h = 0.126. > Eq. (2) [Sons = Sona hi hh=0.18» 0.9h 0.126 haosem, “> height of the block In arotter tgs (¢ = 0.9) Substitute ft to Eq. (1) ee) Soma = 0667 018 > Specific gravity of wood Weight of block = Yee Vit Weightof block = (0.81 « 0.667)[(0.6 x 0.6(054)] Weight of block = 1.272 kN G2 Situation 2 (406) Q=204 m/s om A= 6(1.2) + Yo(2.4)(1.2) <2 A= 10.08 m? 2.00397 ‘Specific et VH= +, + ft : 20981) 12 Specific energy, H =1.409 m. Part: Sipe oe bpigt a“ 1008 Re? epee) Om 1 20m = ar sya Sara 8868)? Su S=0.00098 > Slopes Past Ls Shearing, stress 5,» YRS = 981(0.8868) 0.00091) S,= 0.00818 KPa= 818 Pa Situation 3 (7109) Parka =982%1) —O5m = 1962 ym 4 (2) = 0.667 m faa) FxyeTx25 sweulnsey=25T am T= 5.202 kN From the FED of the ‘concrete block: EF, =0) T+BF=W BE = YeVomc= 981 Voc We Yom Vege ® 23.5 Vere 5.232 + 981 Veen ® 23.6 View Vien = 0.3796 m0) 260 May 2003 GD Situation (1ovo12) __.0826(0102)¢3000)(0.01)2 ha > #1549 02 2 gn SOREON) REINO —ayp554 02 We oszi(0.02)(200)(03)° =16.52003 02° 2 y= SOKO AION? 452m oe he= We Msgs = 16500 OF Q:=33230, > By) [2:+0)-o.01) 3325 Qs +@)- 001 Q;= 0.00281 m'/s Qy=2m Ys Substitute Qs to Eq. (1) (Qs 3.325(0.00231) = 0.007687 m/s (r= 7.687 Ys [HL = in + ha + ha) HL = 1549 + 1495,69(0.0076877 + 00852 HL= 1,683 m Civil Engineering Reference Vol. 2 261 ————— CD Situation § 13 t0 15) Mass. i= 500 kg Volume, 7=0.315m0 Weight Weight, W= ig 50019381) Weight, W/= 4,905 EN = 4.905 kN p= 1,587.3 ka/en! "i ait weight Y= Pe = 1.5873081) = 15571 N/m yr 1557 KY? {0 Situation 64610 14) wreNo =m Pasi ae for Fine 7 ca € cs 1 3 ba B w, z a 15 to 30 4 Part: Elfective Size ‘The Effective Size, Dia, 10% of the soil is finer. D, through soils and ean signi of soils. is the diameter of the particles of which i an important valve in ogulating flow fcantly influence the mechani Behavior For this problem, Dip = 0.149 mm The Avirage Grn Size diameter of the sll is Dig Hydraulics and Engineering 262, May 2003 Part: Coefficient of uniformity: 2 w= 22 nas Oo oie ope o l | il ® ll | x” : iat ee eee eo Lab ie | | cranes = i | Fo 5 ee | a | i «LL oy pm a oe a ae z oe : bari Det hi q= Pe se Dip 7 Dw = 0.149 a From the grain size curve shown, Dig =22:mm 1 y 5 iL a = a From the graph, Dsp= 0.63 mm = 121 (Between 1&3) = 0.BY 0.149(23) Mydeaulles and 264 May 2003 cecntnoenane Since G, > 6, ©. ls between 143, and more than 50% pass No.4 Sieve, the 's SW (Well graded sand) tion 7 (19 to 21) MC=25% G=20 M 2A M4 = 1888 g/cc bey P™ Sa0— 382 ws = Gan 12% = 1.208 (=P y an hen texGal Ya 981% 1896 = 18,62 kN? Geeme 2.64 2610225) = “l Se oat fra SESME | agen 282280 at = 0678 tue = 28 981-152 kN Tee Toss {D Situation § (2210 24) Total stress in A: prem 20325)*2032) pr= 91.35 kPa Pore water pressure in 4: pom 98125) Po 24525 kPa Elective stress in A: rape Pel Pea pr Pe pe= 9135 24.535 pe= 66825 kPa @ Situation 9 (251027) Note: ‘elowy the water table the soil is saturated and the water i called foe ‘walet, The water pressure is positive (compression) anil increases’ ‘with depth below the watertable. f ‘Above the water table up to the height of capillary rise is the | ‘cone water is called capillary water. The woter fan atmospheric) i the capil zo, i | Pete C= 0.20 emt hg = 10 m= 1010 m = 10x 10 cen en 04 020 Oagiox 10 500 m= 5m +n Pore witer sss dein Solving for yu 1 First ne solve G - 265= 981% G:G= 2701 5 2 2IDL+O4 yy : oa EEE OA 29.81 = 21.729 N/a 2701+ (05)(0 OUND gr = a 2033 kN/m’ ‘Vertical effective stress at A: ‘Total stress at A: pr 21.729(2) + 20.53(8) = 101448 kPa Pore water stress a A: Pom -981@)=-2943 kPa Tero pet pe] 1OLMME = pe» (29.43) pom 138.86 ka soars 266 May eotecnniea Engineering Varical cece teas atr pe ™ (21.729 ~ 9.81)(2) + 21:729(5) + 20.33(3), tes fossure ©. Situation 10 28 to 39) Plasticity Indes PIsLL~PL=4b=21.1 = 19.9% Liquidity Index: o.sa7 DESCRIPTION OF SOIL STRENGTET BASED ON LIQUIDITY INDEX. Uo” > Semlsolid stan ~ hiph sength, title (sudden) D Plastic state inermadiite strength, soi deforms cy Meeaplatc rte ut istate = low strength, soil deforms tike a ‘uid Since LI = 0.447, Le. 0- Gross foundation pressure, 4, =277.8 kPa civil Reference Vol. 2 267 — Patil Net foundation pressure, q. = Dy Y= pg = 19206.51) 16,8352 N/m? y= 18.895 Lyf Net foundation pressure, x= 2773 = (18.835)(3) Net foundation pressure, = 221.3 kPa Part 11: weet cos (2549/2) cos? women ( 2, ew eftt/4-4/2)un4 b= 30= 2/6 en Oren aT 5 55 sa517 | 2cas*(45 430° /2) 3.51? Ae 2eos* (45+ 30° /2) sesaar Ky=3tant [rt] =3tant (oe) ey =5208 fk eee oe + ) ean ar ( ae Neo 19745 a= SEN. +7 D/Ny+ OADYB Ny 4u 0+ 18.835(3)(22-46) ~ 04(18.895)(6)19.745) qe 2161.66 kPa Factor of Safety, FS po Factor of Safety, FSaq = D Situation’ 268 May 2003 2 (341030) qeavnkia Considering 1 unit width perpendicular to the figure: 47 1S Fhe = (Ha) + CL Hpbi + Haat y= Has Hy = Hes 3m han = 2x 10%em/sec hyn = Vx 108em/sec Jag. = 2x 104 cm/sec faa = ¥x 109 cm/sec Feou™= 3p adh + Hie He Han H+ +B Fl) eay= gphgy [2 « 10? G00) +1 108 G00) + 2104 (300), +1 109 G00)] coq = 0:0008025 cm/s = 8.025 10+ cays Part! 7=070 ga hem iA ‘Considering em width: H= 4m x4= 12m = 1200 cm Aw Hel = (12001) A= 1,200 mz = (81025 x 104 )(0.70)(1.200) S674 cm/s per cm width 9 Mydrautics and AL FM) Civil Engincering Reference Vol. 2 269 Seat Nor CIVIL ENGINEER Licensure Examination ‘Thursday, May 15,2005, 0800 am. -01:00 pm SINE TIO. SETA INSTILUCTIONS Ste the core anawer for wach ofthe following questions. Mark only one anawer for each tem by shading the box corresponding to the let De eae eee y 3 ICTLY NO ERASURES ALLOWED. Use pencil no. 2only NOTE: A times B is represented by Ax B or A‘B. A raised to the power wis ‘expressed as A’. All problems are 3.33 points each. SITUATIONAL . Situation 1~ The concentrated load shown in Figure ME-98 acts at point A, A. If the concentrated load acting at A is replaced by an equivalent force- couple acting at point &, which of the following most neatly gives the ‘magnihide of the equivalent couple, in kileNewtor-meters AW C16 B18 DB. 2 211 the concentratd Lond acting a replace by two horizontal frees, one acting at point B and the other acting at point C, which of the following most nearly gives the magnitude of the force at B, in kdoNewtons ‘A. 450 S60 2. Whichof the elon fy gies ke mopeds of ta ic following mast nearly gives the magnitude of the force at, in kiloNewions: ‘gs ‘A. 350, <. 600 B. 500 D450 Situation 2 A pline tras sande as shown in Figure MEI, 4. Whichof the following most nearly gives the reaction a, in Newtons: 760 A gO B. 645 D, 939 5. Which the following most nearly gives the bar force AH, in Newtons A810 on yee es 8 1900 D. 2078 6. Which ofthe following most nearly gives the bat force D, in Newtons A760 ©. 640 B. a10 B90 Situation 3.~ beam has a T-section built up by two plates as follows: a 200 millimeter « 40 millimeter plate as flange and 2 100 millimeter x 20 pilot plate a8 web, The maxim shear force acting on the beam is 60 leNewions Serucrural Engineering. Sind Conseraction 270 May 2003 7. Which of the following most nearly gives the moment of inertia of the section about the neutral axis, in millimeterst, 3 onan 8 Which the following most nearly gives the shear stress at the neutral acs, in MogaPascals: A. des C198 B32 D271 9, Which of the following most nearly gives the shear stress on the web at the junction of the lange and the web, in MegaPascals A196 C318 B 432 DB. 22 Situation 4= The beam by a uniform load as shown. re AN-24 is acted “The product El = 1,000,000 where the modulus of elasticity of the material {@) win Pascale and the moment of inertia of the erosssection (0) i In meters < 10, Which of the following most nearly gives the vertical reaction at the roller, in Newtons: A528 c ea B. 460 Bay 11. Which of the following most nearly gives the moment reaction at the first end, in Newton-moters! "A. 336 cm 3 468 D, 220 12 Which ofthe fellowing mot arty gives the slope at the roller suppor in ‘A. 0.000124 C. ogooss1 3 aoon26s , oon Situation 5 - A square timber column with dimension = 150 millimeters has height of 30 meters. The column fe fred at both ends (effective length factor = 0.67), The timber $s 80% stress grade Apitong, The properties of structural timber of Philippine woods are given in Figure TM-12. The provisions of the 1992. National Structural Cade af the Philippines relevant to this problem are given in Figure TMc13. | 13. Which ofthe following gives the classification ofthis cofimn: | A. Long, C. Intermediate B, Short 'D, Not allowed by 1992 NSCP 1s Which ofthe folowing mort neatly gives the allowable compresive sss, in MegaPascals: pas) © 646 B 776 D922 15, Which of the following most nearly gives the maximum compressive load, in kiloNewtons: A. 265, Cas B 362 Ds Referente VoL 271 —————__________ Situation 6 — A threzapen conkinagua bean. shown in Figure RC-6 Tho provisions 2 Netional Structural Code of dhe Philippines relevant to this problem are also given in Figure = RC-6, m 16. Which of the following mast nearly gives the ultimate (Actored) uniform load. in kilohewtons per meter ‘A 5.65 © 698 ich of he fo sty ghee wine (fs 17, Which of the following most nearly gives the ultimate (factored) bendi moment at G in kibNewtoremeters, Baal ‘A. 267 a7 8 Which of theft a 18. Which of the following; most nearly gives the ultimate (factored) bendi moment at, in iloNton-meters: Saree A366 cus B71 DBs Situation 7 An exterior column With service dead load = 760 kiloNewtons andl service live load = 580 kiloNewtons, and an interior column with service dead load = 1100 kiloNewtors and service live load = 890 hiloNewtons are to be supported ona combined rectangular footing whose outer end cannot rotrude beyond the outer face of the exterior colurnn, as shown in Figure: iC-21. The allowable bearing pressure of the sail is 290 hiloPascals. The bottom of the footing is 1.80 meters below grade and a surcharge of 4:3 JbloPascals is specified on the surface, The Footing thickness is 20 meters. The unit weight of concrete is 24 kiloNewtons per cubic metor and the unit weight of soil is 18 kiloNewtons per cubic meter. The footing is to be such that the resulting pressure under service loads js wniformn. 19. Which of the following most nearly give the effective allowable bei tire, in hiloPascals (allowable ‘Pressure minus the weights o Ecnerete so and suchatgey Se a AL 1d C90 BAS D. 127 20. Which of the following most nearly gives the distance, in meters, of the point of application of the resultant service load of the two columns, ‘measured from the outside face ofthe exterior column: ‘A298 © 350 B 185 Dz 467 21. Which of the following most nearly gives the minimum dimensions, in meters, of the combined footing: A, 200 7.0 B 350% 7.00 © 2505700 D. 3007.00 Situation &— A simply supported beam with span of 8 meters is subjected to a sounter clockwise moment at the left support and a clockwise moment at ‘the right equal fo 25% of the moment atthe left support both acting in the plans of ths: minor axis of the beamn.‘The Beam if nat restrained against lteral buckling, ‘The beam is A36 stool with vield strength Fy = 248 Fascals. The provisions af the 1992 National Structural Code at the ippines relevant to this problem are given in Figure ST-4, The beam is 21 62 steel shape, whose relevant propertics for this problem are: 272 May 2003, ‘and Construction Ry = 0.053 meters = 0.016 meters d'=033smeters = 0002077 meters ‘y= 0.210 meters Which fhe following most any geste slendemess ato, above which the bea is long with respect to lateral buckling: grrr Tee © B21 S D. 37 Which ofthe following most neasly give the allowable lexural stress in the compression flange, in MegaPascal NAS 85 B 619 DSi Which of the following most nearly gives the maximum value of the ‘moment at the let suppor in iloNewton meters: A 8 8. 193 D. 578 Situation 9 ~ For government projects, bidding is governed! by. Presidential . Decree No.1584 which states that for the purposes of cetirmining the average of Wve bids, bids higher than. 120° of the AAE or lower than (07 of the AAE shall not be considered ‘The approved timate (AE) of the government projet is 280, efrsals of te bidding ae asfollowss Liege 000. The res = P321,000,000 = 337,000,000 = 374,000,000 Bidder D = P 300,000,000 Bidder - P 294,000,000 Which ofthe losing most neat gives dhe aimum bd in millions of ‘pesos, thatcan be considered a responsive bi ‘A 370 C283 B 336 D. 235 Which of the felloveing most nearly gives the minimum bid x millions of sos tata be considered responsive bi ‘ © 80 B. 302 D, 168 Which of tle following gives the complete list of the responsive bids: © Bidders 4.81 A. Bidders 4, CD D, Bidders AD,E B. Bidders BCE ation 10 ~ The following are the activities for the construction of a residential house, giving the nodes of the Network Dizgzam/CPM, time duration and cast (in pesos) for a noemal and an accelerated program: aay [eet ¢ Concrete Wark 10 | 000 | i Tange We a ser ae =] san] 3] or Ea ie Sse] — ar eee Capea 8 | 10.000 6 15,000, [oan aor s-}—tae—[-$ ce Wats 5] — amo} a Ean Wr | ao | — ine feo oe ee E1K_| Pubs Fors ineoe =| — asp —| 4} eo 31a Pr a a 2B. Which ofthe following gives the cxtical ores ABGKE o pheli hg ws teSE (fh te llowing actress the mat csi lective in converting to ‘A Activity A th of the project: it Blown) 30. If the budget for the projet is 7103700, which ofthe following gives the reduction in the number of days to complete the projet comrere ta tee oe | program: ts = aes = 5 a E : E 8 E Figure ME-98 274 May 2003 $enom=0m Figure ME-17 Figure TM.12 ' ‘The effective length I, shall be used in design formuias given in this section, Actual length shall be multiplied by the appropriate efertve lngih factor to determine the effective length L. Allowable unit stresses of cross-sectional area of square or rectangular solid columns shall be determined by the fo formulas, but such tit stresses ‘hall not exceed the values for Compression parallel tothe grain F.. Short columns (Lx/D of 11 or les}: Pek Intermediate columns (L,/D greater than 11 but less than at where K = 0.671677; Fon k Long columns (Le D of K ar greater but less than $0): 0.30E (L/D 1,/D shall nat exceed 50, Civit RemeecsevoL ae 275 roo um Figure RC-6 ‘vm 1 An (uefectored) v= 25k munca) Section 52.3.3 of NSCP states that in lieu of frame analysis, the following ‘approximate moment and sheers are permitted for design of continuous beams nd one-way slabs (slabs. reinforced to resist flexural stresses in only one Gitection), provided: 276 May 2003 (8) There are two of more Spans, (0) Spans are approximately equel, with the larger of two adjecent spans not ‘greater than the shorter by more than 20 percent, (6) Loads are uniformly distrbuled, (3) Uni live does not exceed three times unit dead load, and (¢) Members are prsimate Positive moment End spans ‘ Discontinuous end unrest cove Mule 114 Discontinuous end integral with support... ca La 14 VRt@HO€ SANE eo nennnmnnone wi Let 16 Negative moment at exer foe ft nee supp us Two spans cong wile) 9 More than two spani i oncogene La 10. Negative moment at other faces of interior supports. wale 44 Negative moment at face ofall supports fot: Slabs with spans nat exceeding 3 m; and beams where ratio of sum of column stifiness to beam hs sifiness exceeds eight at each end of the span... muted 12 Negative moment at interior face of exterior support for members built integrally with supports = ‘Where suppor isa spandrel beam... wule 124 When support is @ eOIUMn one nnena Wa g 118 ‘Shear in end members at face of first interior SupPOR sone 116 Ws bal 2 Shear at face ofall other supports... wll? Where Le = clear span for pasitive moment or shear and average of edjacent clear spans for negative moment. ‘Table 4-1: Working Stresses for Visually Stress-Graded Unseasoned Structural Timber of Philippine Woods. 277 i T High Strongth Grou ‘ooho mM | am | usm | sor | 2eey tsa 200 | 935 1560 | 430 26h Malabayabas mio | am 1580 | 870 302 Nanogschepul a | as 1600 | 603 218 Noiave mo | es | suo | 634 288 ang ao | 8x | 1m | 48 261 Sasait uo | 7 | 760 | tom | -age Yaka aso} 918 | sao | 627 2ae TMedseataly High Sirength Group slips ‘6 | 535 | 100 | 300 2% Biogas reso | 657 | ti | 327 224 Bebe yero | gas | tig | aut 218 Deo sea | 543 gaa 2m i Gaia mw | ca | am | ae 235 Guo mo | e7 | aa | 42 240 Kamagang mo | 7 | nm | 43 247 Kamatog 4300 | 755 | ma | 395 235 Katmen’ sooo | gaz | to | 4a 2 pene Ae 4850 | 46 | tos | gaa an Malatavayan wo | ee | sa | 292 24 Nara 1300) 5m | tao | G01 ig Pehuian i660 | _653_| 100 | 250 205 Ti, Mediu Stent Group ‘ping fos | 731 so | 2m 173 Bagiian 1660 | Gas 89 23 162 Dengan 10 | 6 | 229 248 138 sau wo | 53 B16 198 190 Lanuton-bagys 1500 | 605 | 805, 202 1a Loum noo | 583 ate Azz 148 Maeaneneng 1ag0 | Bat 8st 195 19 Malasaging 1680 | 5a 9st 202 185 agai isa0 | 630 233 307 207 Palosaps saad | 588 838 273 188 Pe 470 | 686 82 $98 158 Salakin 1570 | 567 283 294 188 Struztural Engineering 278 May ‘and Constevetlon "ial inutan ‘so | 56 | 6m | 265 738 Figure ST-64 Section 451.45 of the 1992 NSCP gives the allowable stresses, in ‘MogaPascals, for beams not restrained against lateral buckling a follows! 1. Tension: - h=06F, : 2. Compression: ‘The larger value computed by Formal (45-64 o (45-65) and G27) atapplcnle butnotinore Gan Wath 703,000, & sea a ‘ 2 es (45-60) 3 10.55% 10°C, When E , S=zOO0OCS ® as.) E/RP i, when the compression flange is solid and approximately e crossacction and area is mot less than that of the 57) Civil Engineering Reference Vol. 2 279 Where Gh = 175 + 1.05(M@i/i) + 020(M/¥Ma)? But not mote than 23, Where Mi is the smaller and iss the larger bending moment atthe ends of the unbraced lengeh, taken about the strong axis of the member, and where 2/4 the ratio of end moments is Positive when My and Mz have the same sign (reverse curvature Bending) and nepative when they are of opposite signs (single curvatuize bending). When the bending moment at any point within an unbraced length & larger than that at both ends ofthis length, thve value of Cs shall be taken as unity Go may conservatively be taken as unity for cantilever beams Ry = radius of gyration of a section comprising the compression flange plus 1/3 of the compression web afta, taken about an aniein the plane ofthe wre. {B= depth of bears y= width of flange y= thickness of flange “ANSWERS: $8 6C uD HA mA md 20 74 Ua wo BA TD 30 88 hE wo mG Be SA GC WS hE AS BA SD WE HD AC Be we 280 May 2003 ‘and Construction Lr Solutions to May 2003 Examination Situation 1 (L403) are om ‘ : E 2 _ aa ; +t mm a « * c = c M=Momentat 8 doe o BOE fone M=150x S20 M=18kN- Parts 2and 3: wom, i a E 2 ss ten dom 8 ane e = € re Moment at C due to 150 kN = Moment at C due to Fy 150 AB = Fax ay Fa= 600 KN Moment at B due to 150 KN = Moment at B due to Fe 150% BR = Fex fy Fe= 450 kN Civil Engineering Reference Vol. 2 281 te EE ee es DSituation2(¢t0 6) see | fan x: a rs 60 1507 =0m [Ma 0) -480(21) + 4B0(42) + 96298(91.5) = 658-29(14.65) + R(63 Reaun ate ae [Fy=0) Ay +811 =480+ 480+ 98298 Ay =1132N (SFr = 0} ‘Au= 688.291N Isotate joint A. ; (EFy=0} ie 113 Fy sin 25° Aas Fu = 2678 N (Comp) an ie ero} 1688.29 + Fag = 267800 25° Fas = 1,738 N (tension) By inspection: Eu= Eu ~ 2678 N (compression) Fac = Feo ™ Fun = 1739. {tension} eq 480 N (tension) Isolate joint In right triangle DCI (See the truss) me omar = 180-65" 47" = 68° 282 May 2003 and Construction BF = 0) ¥ o Fozsin a = 480 sin 65° " Fo= 468.19 N Be Mc =O} Fp(31.9) = Forcos 8 (31.5) + a80(21) Fp (315) = 469.19 cos.47° (81-5) + 480(21) Foy= 640 N) 7 Situations (7 109) Solving for and Inas Bom Av 200(40) = 8,000 mm ‘Az= 100(20) = 2,000 mat A= Ai + Ar 8000 + 2000 = 10,000 mn? [Ay = za) 0.00097 = 8,000(20) + 2.000490) 731mm = M-20= 1m = 90-34% 55mm Civil Engineering Reference Vol. 2 283 Tian = 2(l, + Aah] = many z+ 204100)" : tas SEE + 000s: + SOPH + zonnrsey ux = 10,575,385 mm > moment of inertia Shearing, stress at the neutral axis: 200 mm Ay = 200(34)(17) 115,600 mun® 10573353 mmt 2200 man Vm kN V=60,000N (given) __60,900(115,600) 10.573, 383(200) fo= 328 MPa fe Shearing stress on the web at the junction of flange and web ve it Q=ay=100;20)56) Pe (= 112,000 inant T= 10.573,583 mnt (= 20mm V=a0kN V-= 60,000 (given) 60,000(112,000) _ s79 10,573, 355(20) " 284 May 2003, oe (DSituation 4 (10 to 12) team o i = lesa J (rea)ac Fo Om Fe PAS)SR) % (10/3) ~(1/5)9)A50) (17/8) (125/)Re- 19125 = 0 Re=45.9N > Vertical reaction at roller support [Ma = 5Re~ 350) Ma 5(159) - 450 Ma= 2205 N-m > Moment reaction at the first end 1 (Gsc= J (arenacd Bac [aS)S45.9) ~ (1/9)()(A50)] = 12375 Ne) X= 1,000,000 N-m? (owen) 123757 7,000,000 0.000124 radian > Rotation at roller support Reference Vol. 2 oe Ti Situation 5 03 wo 15) Part 1, Classification of column: I= 3.6m=3,600 mm Tonk. Ly = 0.67(3600) [= 2412 mm D=150mm K-sri ER E=7.31%10°MPa (from Figure TM-13) Fo=956MPa_ (from Figure TM-13) ‘ 751310" 9156 k= oor Ke 1855 ie D & = 16.08 (greater than 11 but less than K) The column is an INTERMEDIATE COLUMN, Part 2, Allowable Compressive Stress: Bor aft zea) . 31855, Fetches = 7-76(150 x 150) 7" 174,600.N 286 May 2003 Situation 6 (16t0 18) Part 1- Factored uniform load: < ey Tene + 1Fen.= ACD) + 1.7025) = 5.65 kN Part 2- Moment at C: F Point (negative moment is at exterior face of fist interior support with more than two spans. Wy gt oS Cy) ye $2975 wegen 2 S687)" 67 um Part 3- Moment at Hi: Point Hf (positive moment is at end span with discontinuous ond integral wrth support meee L= Ge (lec mpeat te voting mesa) 5.65(6.5) Mu= SSE 7 nm @ Situation 7 (19 to 21) REFER TO THE FIGURE NEXT PAGE Part 1, Effective soil bearing pressure: = $4~ Yet a Yn: He -Sureharge o 250 ~ 18(0:6) ~ 24(1.2)- 4.8 = 205.6 kPa Sivil Engineering Reference Vol. 2. 287, Patt 2; Value of a: Solve for X by taking: moment of force aout 2: y= 760+ 580 = 1,340 kN Pr 1100+ 890=1,990 kN Pe Py+P,=1340 + 1,990= 3390 kN [PX = Pit Pea} 3830 X =1,340(0) + 1,990(5 5) X=3317m X+0205 9" 3317 + 0225~3.582m =i 288 May 2003 Part 3, Footing Dimension Such that the resulting pressure under the service load is uniform, the resultant service load P must coincide with the centroid of the footing. ‘and Construction: Thus, a= L/2 S5a2=1/2 L= 7.094 m ‘Area of footing Ay= = 13559 mt 6 % (ante T35967.064xW; W=1Sim ‘Therefore, the footing dimension is 2m x 7m 1G) Situations (220.24) inane pe 2175 +105 My/My) + 03000/64)2 » My = 0.25. M MoM Mi/ Me (0.25047) ‘Mi/ia=-0.25 — (negative because they are of opposite sign) .75 + 1.05(-0.25) + 030(-0.25)° = 1aoas 23300 Fos pooG, __[70s,000K1-30525) { ht 2s pscomoncy _ [SER OOCTSORDS) 4.» Ta 28 Civil Engineering Reference Vol. 2 287 ea ee Part 2, Value of az Solve for X by taking moment of force about P\: Py = 760+ 580 = 1340kN P= 1,100 +850 = 1,950 kN P= Py + Pr 1,340 + 1.990 ~ 3,500 KN [PX= Pine Pa 3350 X=1340(0) +1,9996.5) X=3317 mm a=X+0205 2" 3317 +0.225—-9542m aa 288 May 2003 Part 3, Footing Dimension ‘Such that the resulting pressure under the service load is uniform, the resultant service load P must coincide with the centroid ofthe footing Thus, a= Lf; My=Lxm 13559 =7.06x iW; W=191m Therefore, the Footing dimension is 2m xm CA Situation 3 (22 t0.24) My ~025M Ma=M Mig = (0.25/21) Mi/Ma=-025 (negative because they are of opposite sign) 5 + 1.05(-0.25) + 0.30(-0.25" G= 150625 < 23 (OK) FOSOOOC, , [FOBOOO SIG) _ 5 4 y 28 fysaooO0e; _ [SSAODO0(LSOSE) _ 5 Er) y Reference Vol. 2 289 Part i: Sleteerness ratio, above which the beam i long with respect 1 Inara buckling: etm (EE yes 5 Pact 2: L [3,520,00C, Since 1 5 [BSADOOES ne cotapesnhn lange s sold ane , _ ‘approximately rectangular in cross-section and its area is not lees than that ofthe tenston flange, Fyis the larger value (45-68) and (45-7) but not sreater than O.6F), Fg. (45-68) r 1170/0000, _ 1,170,000(1.50605) _ ~ (LARP asi? ee ‘Eq, (#5-7) 83,000C, _ 83,000(1.50625) Be Tr "= assy 985 MPa tty (@21)(0.016) 0.6F, = 0.6(248) O.6F, = 145.8 MPa, Therefore, Fy= 98.5 MPa Part 3, maximum moment at left supports Allowable bending stress: In tension: Fy 06F,= 1488 In compression: e985 MPa Use Fe=98.5MPa M Ge) e5- 0.002077 x 1000° ‘M = 204,564,500 Nemm = 208.58 kN-m ‘Structural Engineering. Civil Engineering 290 May ‘end onstruction | peference Vol. 2 291 Peterence Vok 2 (@ Situation 9 (25 to 27) ‘The normal cost (Cu) of the project is: AAE = P280,000,000 ‘Cy = 6,000 + 22,000 + 8,000 + 9,600 + 6,000 + 10,000-+ 10,000 1% of AAE = P336,000,000 “Pnoag + 200 + 4.80n + 153900 + 2000 60% of AE = P168,000,000 ee Pact id Parts 2: ‘Maxinuan bid that an be considered a responsive bid is S86 000,000 Rene aa srin ra Part2 Masdinum cost avalable to accelerate the program ~ P103,700-- P9800 Mininhin bid that can be considered a responsive bia fe F169,000,000 ‘Maximum cost available to accelerate the program = P3,900,00 Past 3 ‘The most cost effective activity to convert to accelerated program is the ‘one along the critical path with the least cost per day to acctlevate ‘The cost per day to accelerate is as follows: ‘The responsive bidders are those with bid greater than P15S/000 ,000and lower than P336 000,000. ‘Thus, the responsive bidders are, D, and E ‘Activity A: (1 day) aera : Cate $5208 yon pe dy si dap) = 24,200— 22-000 _ pi 100 per day Cost: 70-8 P1,100 per day 3 hake h oir 3 Con = 8011000 — asp pray a Activity H: (L day) 2-66 | Doors and Windows & 10,000, Cost = (4,900 ~ 4,000) / (6 - 4) = P900 per day Ser ena ae ee 471 _| Gelling Works: A 2,400 7237” esting Works [os 0 Activity (@ days) Pan [ae SS vt = 1800-4800 a0 per day Sere a recente =r Activity L: None Thins, the most cast effective activity to accelerate is Activity A. Part: For acost of 3,900.00 ‘Accelerate A by Uday;1(800) =P 90000 ‘Accelerate B by 2days:2(1,100) =P 2,200.00 ‘Accelerate Hby Lday:1(@00) =P 900.00 Total = P 3,900.00 Thus, the reduction in the number of daysis1+2+1=4 days Part: From the network diagram shown, the cetical path is path A-B-F-H-I-J-L 292, November Mathematies, Surveying, and ‘Transportation Engineering, Beal New CIVIL ENGINEER Licensure Examination Saturday, November 22,2003 08:00 a.m, = 01:00 pan. \TICS, SURVEY TRANS. ENG'G. SETA (ON: Select the correct answer for each of the following questions. Mark onl swt for each item by shading the box corresponding to the letter rar choice on t of answer sheet provides. STRICTLY NO ERASURES ALLOWED. Use pencil no.2 only, MULTIPLE CHOICE 1, The magnitude of the vector F = 2i+6)- 8k is: ao Tors < B, 136 D102 2. Round off 108.288086 to four significant figures ‘A. 108 2889) TEN ee 8. 1083 , 108.2888 3. When the expression xt + ax) + Su? + bx + 6 is divided by (= - 2), the remainder is 16. When it is divided by (x+ 1) the remainder is 10, What is ‘the value of the constant? AS Ct oe Ds 4. IEversed sin theta =0:148, what isthe value of theta? A. 3157 C 178° S843" DL 1128" ‘5. What is the equivalent of 720 in the centesimnal system? ‘A 600 grads C. 800 grads B. 750 grads 1D. 700 grads 6 spherical triangle, angle B ~ 81°.50° and angle C= 94° 9, Iside ¢ = what is the value of angle A? A562 © or 28 BSP az D. 518 7. Given the aides of a triangle ABC: a= 363 cm, b= 25.9 cm, c= 7. The angle ‘opposite side ais 1027". Compute the value of sce cin ceniumetres, 152 C 2578 B 2257 D. 2045 8, Determine the area of a regular hexagon inscribed in. circle having. an area of 70 square centimeters A168 ©. 1482 Bo 12d D140 9. Determine the volume of a spherical wedge of radius 2 m and a central angle of 1.25 mdians. Rasy © 985m B 8.64 m D. 574m 10. The lateral area of a right circular cone is 386 square meters. If its diameter is one-half its altitude, determine its altitude in meters, 293 cue i. The regular tetraeclan 17 . The surface area ofa regular tetrahedron is 173.2 square: Bie surface ven of reg square centimeters, What & fem 72 12, Irthe cave Ast By? + P= Opacesthaareh (3) and curve As2+ By? + F= , 3) and (4,0) the curve is: Aree ol ee ney aaa B. anellipse D. ahyperbota 18, The distance between pins (2.3.0) and @ 10 eyietant units. Cake vali of a5 cs BF D6 é nce between points A(2, 5, 4) and B(x, 3, 6) is AB = 5.745 m, 14. Ifthe dis determine the value of x. ad C3 B6 enb.7, 15, Evaluate the following limit of (x =4)/(x2- x~12) as x approaches 4 AT . Coe eae ee B. undefined D.1/6 15. The sum of two mumshers is C The product of one by the cube ofthe other is tobeamaximum, Determine one ofthe rumbers. A ac) © 3c/2 B3c/8 B. 3/7 17, A triangular ot ABC has AB = $25 m, BC ='9.61 m, and CA = 862m. A rectanglar lot is inscribed init such that the shorter sie i on the 425 mh Shdeof the triangle. Determine the manimum area ofthe rectangular 10k ‘A232 rarered abou erste cle moves according to mete equations x= P and y = 27. What th elec ofthe parce ent = 2 = . 1 © 1a 8 tai Diese 19, Whatis the perimeter ofthe curve r= 41 ~ sn)? Asner ‘ca B bi D, ans 20. Find the surface arca generated by rotating the first quadrant portion of the gave =16-tystontihe Yeus eat ‘S341 az B. 6425 D. 6638 21, What isthe area bounded by the curve y ='6 cos rane the X-asis from = m6tox=9/2? ra G4 a3 Dil 22. A nominal annual rate of intevest of 7%, compounded continuously, has an effective annual interest rate of es i 725% © 747% A. What ete aittronce between the suns of te a ‘ste difference between the sums of an annuity due and an shruiy forthe following daa: pir els 294 November ee ee eodkeproat = P100,000 24. Payment infervals~ year Interest rate = 12% compounded annual A. P965,732 C. PR64.E29 B. P6573 D, P7358 ‘An equipment és bought at P420,000 with an economic life of 6 years and a salvage value of P50,000. The first year depreciation is P105,714, The cost of ‘money is 12% per year. What method of depreciation was used? A. straight-line method C SOYD method B id method 1D, declining balance method ‘A 1-Kg block impends io meve along a horizontal surface when a pull of 40 Nis applied on it at an angle of 50° with the horizontal. Determine the coefficient of friction between the biock and the surface. ‘A. 0.25 C018 B 034 Don A horizontal platform with radius of 25 m rotates about ies/¢enter at a constant angular speed of 10 rad/min, The normal actcleration at the edge of the platform ix A. 0.0854 m/s? © 0085 m/s? B 0.0s2m/s D. 0.049 m/st ‘A horizontal force of 250 N pushes a flat-bottomed 600-N block on a ‘horizontal surface from rest to a velocity of 22 m/s after travelling 120 m, ‘Compute the value of the coefficient of friction between the block and the horizontal surface, “A 028 com B Ok The cee ockered ao ola pil wie HES 1D tb Se Lhceare nei ‘The crosresectional area is as follows: SB pra 7a totes Determine the value of 432m cum ‘a Sse elangilar tavéme has tha llowing da a verse has the following Line Bearing Distance AB Nor E 5000 m Be Duse South Hes cA Neorw ‘An area of 200000 square meter i cut-off slaRing from corner A to point F ‘online BC. What the length of line AF? A #7835 m ©. 86814 m B am25m D. 91475 m Bris Wa WA He mA ue me wa we Be we a6 ms wo we Re MA A me BS Civil Engineering Reference Vol. 2 295 = i Solutions to November 2003 Examination fay +6? +-3) =10198 Number = 108.285886 Round one il gue = 1D Rounded af © fve significant figures Rounded to tice significant gues = 108 Rounded-off to fou significant figures = 1083 Rounded.ofto ve sigteant gues = 10829 pit roles has one significant figure 100 Ras one significant figures 100.00 has five significant figures ‘8.000289 has three significant figures ffayext +084 524+ 6 f2)=- 21+ a) +500 +10) + 6-16 +2hed6 daebel3 3 (1) me Cults attH tt parle 10 neem b-2-0 32) ‘Substitute b= 2a from Eq. (2) to Eq: (1 a+ (2- a) =-13 ay Sa+2=-13; 3e--15 o=5 b=2-(5)=7 Versed sin 0= 1=cos 0= 0148, 08.0 = 0852:0= 3157 Note: Versed cos = 1 = sin @ ‘The units of angle in: entesimal system is grad, where 90° = 100 grads, zap, Ogre oF = 800 grads B= 61 5 Using Napier’s analogies: sinJle-1) | tan}HC-8) sing(e+t) " ~ cotga sin}(90- 83178) _ tan }(94°90-81°50) in} (90 03:178) corp cot $A = 186213 tan $A = 0537018; A = $6" 28" on Given: a= 363, b= 23.9, A= 1027 Solving for angle B by sine law: aos fe ThA in” aint B = 39.96° and 140.037" Since a plane triangle can't have two angle more than 90°, use B = 39.96? Angle C = 180° - 102.7" -39,96° = 97.4° mn Solving for side c by sine law: € ea Te ‘sin 102.7" * ai ‘Area of circ r= 7.356 art=170 Atecagon = 937,356)? sin 60° x 6 ‘Axcngen = 140.6.crn2 Mace. $0 oi Yona _ $202)" 2 Ventge = 6.67 cum. = : 6.67 Lateral area, Ar= url, = 386 w= 1287 3.1) D=2=\h; hear 2a Bates De Qe ete L >a Substitute L of Eq, (3) to Eq. (1) eA) = 122871 = 5.458 ‘Then, L~ 5459 V17 = 2251m Surface area = a J3 = 1732 a=10cm Altitude, k= «JF = 10 /$ =816cm The curve Ax! + By!+ F= 01s conic section, Dividing both sides by a: erdys id FE) ALGO} PeROPHE-0 16 AL@,3) 4 4 GP 16-0; $-¥ In Eq. dt: 2+ By-16=0 95+ 16y9-144=0 (an llipse) Another salutio F The curve Ax? + Bye + F = ds «conic with center at (0, 0), IE can either be a circle, an 0) ellipse, ora hyperbola Since the eurve passes through (0,3) and (4 0), sbviowsly, the curve must be an ellipse, as x ese a3 296 November 2003 —“trenportaon sagiastring as The distanced between two points Pils Yu fi) and Pal Yat) it = (=m) Gn- yl # (ea? 9498 (g- 2 + (B- 10? «(6-4 2-35 Fe2e36 y= Band 4 au The distance d between two points Pc. yi) and Pas go) B= (=) (h- MF + =F S74 (x= 2/2 + 3-5 + (6-4P 2 aus By Gillesania’s Rule: setx = 3.999: a6 Let xand y be the number: styeCoryaC-r Product. P= xy Pa e(C-3)= C8 x8 F a2c-40 0 2@C- 4x); x= 0}x~ 0 (abound) $C-4em Em aC/a Another Solution: For maximum product of y= where x + y= C Mathematics, Surveying, and ‘Transportation Engineering 300 November 2003 aw r=d(1-sin6) —cardiod s= low cain if e-sfeec | ot 5 Bil(n/2) + cos (0")] = [(-n/2) + cos (-90°))) S= x= 25083 Note: For the following eardivids: r= a(t -sin€) r= a(l = cos 8) r=a(l +sin®) = a(1 + 6088) ‘Area= 1.5na%; Perimeter = 2n 6- By ¥ = my-2) dy 2 (4) ae i (4) yoade de rs as= t+ Cas) dx as itt ee dAy=2ax feat ax Aveo [fame al) ee asotin 2 [(tenet }"] = Seftor) 1-077 ] AS Ba(pe-n=6.27 9. units civit Reference Vol. 2 ma A [oni cle is A = 6(cin 90° sin 30") Aw3aq.units my ERS e-1= 00-1 =0.0725~ 7.25% Gas For the ordinary annuity: sum, p= AUG+O" =1] _ 100,000 +01: t Teme io Oar Sum, F= P7,205,200.24 For the annuity due: Ala i +0.12)— Sum, Fe mo Hey = MOON O12 ga Sum, F= PS,069,87355 Difference = 78,069 873.55 - P7,205,244.24 = P864,629.31 Or on FC= Pi20,000; n=, SV = P5000, m= 1 By stmight-line method: Dywa= FC=SV._ 420,000-50/000 = os = P61,666.67 inca airs al 302 Noveaniber 2008 —_“temsporntion enginciring By sinking fund method: te EO=SH (420,000 ~ 50,000)(0.12) Gaara ~P455951 Dra d= PA5,593.51 By SOYD method: nape ey MRD Da = {FC- SV). 2x SUM 6 SUM = 5 (i+n)= 5 (1+ 6)=21 312(6)=171] 2) ‘Thus, the SOYD method was used. ‘ ‘Dy = (420,000 - 50,000). = 105,714.3(019, For impending motien: Q (116.506) = 40 cos 50° = 0.2207 nore © 10 mad/min = 1 min/ 60 sec =1/6 rad//s0c sn = (1/8)? 25) = 0.0654 ns? Civit Reference Vol. 2 WorkEnergy equation from A to B: KE, £ Work £Wh= KEp 0+ 250(120) - 600)(120) + 0= ¥2 wot ‘ AmA\+ Art Ast Ay BBA = GNM) + GOS) + HOY7.4) + ¥4(9)0.2) yo 92a02 ‘ace = Yt1000)( sin 60° = 280,000 t= 646.63, In triangle ACE 22 = 1000 + 636.68 -2(1000) (646.63) cos 6° x= 87835 m Civil Engincering 304 November 2003 Geotechnical Engineering Reference Vol. 2 305 ee Seat Now Siteation 3 — A 20m thick rubmerged saturated cy Inyer has water Content ol 57%, Theapesific gravity ofthe sol particles 284. on CIVILENGINEER Licensure Beamination 2. Determine te density of the clay in kg. Sunday, November 23, 2003, £2.00 pm. -0600 p.m ‘A. 17083 C1614 1682 Disa Deserinine the total vertical stress atthe bottom of the clay Iyer, in KPa SETA A 365 cw Bas D. 396 Select the correct answer for each of the folowing questions. 9. Determine the effective vertical sres atthe bottom ofthe clay layer in kPa Mari or each iem by shading the box corresponding, tothe eter A. 20 cm of yt cos ote anne shea roe 8. 185 B38 STRICTLY NO ERASURES ALLOWED. Use pend no 2enly. E Situation 4 ~ A fuily saturated clay sample has a mass of 1525 prams Afr oven- enone : diving ts uss was reel 0 148 gre. Tha Specie gril icles ts 27, Situation 1 - A borehole log profile in a construction project is shown in Ty 10, Escalate the natura water conten of the sap in percent Sir56, “the Coroerpeicn el mreeapeet eran 9 226 A 52 = C6 * ‘Use unit weight of water equal to 9.81 KN/m*. Bs, Bao 1. Determine the nearest value to the buayané unit weight of the clay in 11. Calcilate the void ratio in percent KN/mt ‘A855 © 186 A843 € 957 B bm B 763 B. 1012 12. Calculate the porosity in percent. 2 Determine the nearest value to the effective vertical stross, im KPa, at the ae aes D9 isheghte sca Rae m4 a wes Bee er A ate Situation 5 The srain-size curves for sll an B sshown in Figute S52. is What the average settlement of the normally conbolkated elay layer ‘eauired to classify the sos according to the Unified Sl Classification centimeters. Compression index C. = 0.009(LL - 10} tem. Use the atached USCSS4. ‘AHL C34 B the walup of the coefficient of uniformity of soil A. Bod D ma ‘Aa C32 B73 D. ia Situation 2 ~ A retaining wall 8 m high supports a cohesionless soil having a dry 14, Which of the following classifies soil A? ens of 1600 bs/ m0 ane o Ren reas Ose fly oo A. GP SM 0.68. ‘surface of the soil is horizontal and level with the top of the wall. B. SP D.GC ‘Neglect wail friction and use Rankine's formula for active pressure of a ‘15. Which of the following classifies soil B? cohesionless soil. \. SP Cc. GF 44 Determine the nearest value tothe total earth thrust onthe wall in RN per B Gw DSW Lineal meter ifthe soils dry. A. 12 2 Cc 138 B. 186 ‘D. 157 5. Determine the nearest value tothe thrust on the wall in KN per lineal meter \fowing to nadequate drainage tis weeloged fo level ES m below the ace A 20 © 180 B. 250 DB. 210 6, Determine the nearest value to the height above the base of tho wall where the thrust acts during the waterlogged condition, 433m © 15m Mydrautics and Geotechnical Engineering 306 November 2003 Grain Size Curve TT tii | | i Ie [pea | esse 3 Percent Passing, % sss eto oot ‘oar a ’ 0 Partele Diameter, mm Situation 6 ~ A testis setup as shown inSM-23. A cylindrical mold 4" in diameter is filled with silt to height Fh = 0.2 ft, whose coefficient of permeability ki = 3.6% 104 ft/min. ‘A second coaxial mold is placed on top of the first mold whose inside diameter is d = 15° and whose height is Hs ~ 0.0 ft Its thickness is igible. The inside of this second mold is filled with the same silt, but the annular ring outside the small tube and outer tube is filled with sand ‘whose coefficient of permeability is ka 27 x 10° ft/min. “The fost setup is a permeameter of constant head. Water ix placed in the mold and maintained at a level = 1.25 ft above the level of the outlet. It say be considered that the system consists ofa fictitious soi of thickness ‘=H + Haand coefficient of permeability ke hs egies te weee= FOL Wafiey Ca) +n (Hol b] ‘hag (L/L Maa Hs + big Ha +... + Rin Hy | 16. What is the ttal flow of water in ft}/min? ‘A. 1874 x 104 C. 2368 x 104 B 1517x104 pee neg 14 17, What is the equivalent coefficient of permeability, in ft/min? ‘A, 6,952 x 104 © 73 «104 B, 6.025 x 104 D, 8253 x 104 Civil Engineering Reference Vol. 2 307 1. What is the tolalamount of water that percolated after 55 minutes A186 ce © ec B 386 cc DB, Bec Situation 7 = Theo pipes, 2 and 9 are connected in parallel, If the combined Alischange of dh thre pipes is equal O61 m/e, computa the folowing Assumiagallhave equal vakies of faction factor Pipa s(n Diamar (en pstine ge se (on) 2 200 ow inpipeine Us = 76. Compute the rae of low in ints a hes Gs =a 20. ci pis temtect Tow ingipel obs ‘ompute theraie of ow in pipeline 2 in L/s 361 z cay a of ee oat paline3. in fs ‘ompute the ate of flow in pipeline3, in Ls ‘a0 m C38 Be Dal ting above the wale Situation 8 - a piece of wood floats in water with 5 cm proj eens wood floats with 75 22. Thetermune te height of the wood in centianetres. ‘A. 1258 ©. 1632 B92 D. iso 23, Determine the specific gravity of the wood. Rie oe © 043 B 072 D. 052 2A. Determine the weight of the wood if is cross-sectional area is 20 cm » 20 ‘A465 CIN B78N D.32N Situation 9 Water flows at the rate of 12:m¥/s in an almost level channel that i= 3 m,_The depth gradually increases from 1.9 m to 1.10 m overa length of flow of Sm. 25, Calculate the head lost in meters. ‘A. 0.0623 © oo7is B. 00415 D, 00263 26. Calculate the slope of tho energy grade line. 00083 “ay em 000683 B, 0.0048 D, o.00918 2, Calculate the value of the roughness coefficient ‘A. 0.0085 ©. 00135 B om D. 00173 Mydrautics ana 308 Geotechnical Engineering ‘Situation 10 - A wooden vat isin the shape! a frusium ofa cone, 2m diameter at the fop and 4m ciameter at the bottom, and 3 m high. tix provided with: tho sel hoops one atthe tp end the other atthe boom Ife vat fed ‘with oll having specie gravity of 0.80: 28. Determine the hy drostatic force on the side of the vat. A 77 kN © 1563 kN B. 9259kN D. 1047 KN 29. Determine how high is Ehe said force above the Botiom of the vat? ALIS 07m B 085m D. 050m 30. Calculate the force in the bottom hoop, ‘A. 39.2 KN Cai B 482 kN D. 56 kN Situation 11 A tank of glycerin has a mast of 1500 kg and a-velume of 1.19 mi 31. Determine the weight of ghreerine in ki, ‘A Tse 1472 Badd D. 1623 32. What is the unit weight of glycerine in kN. A936 14s Bas D237 33, Whats the specifi gravity of glycerine? A132 C12 Br 126 D. 136 Situation 12~ A 4-mediameter open cylindrical tank. 6 m high. i filled with water. The tank is tilted to a position such that its water surface will cut the diameter of the base. 4. Determane the yume of water eft in the tani cubie meter Bub D8 85. Determine the depth of water in the tank when itis restored to its upright ¢ 112m. 3 145m D. 168 m 36, While the tank is full of water, how fast could it be rotated about its own. ‘axis such that the depth of water at its center is zero, ‘A. 52 a2 B 365 D.SLB Figure SM-23 Figure se496 309 rr Civil Engineering 310 Noweanber 2003 ceuecnsieartoginceriog | Reference Vel. 2 311 ee Tr Solutions to November 2003 Examination Situation: (1 to) Par tenvexG 28981 xC:G~ 284 cape GMC= Se) = = 040 asa (below the water table) 2824(040)= (I): ¢= 12086 _ 2Rd=1 ; oe a Ol) ~ B82 Ne’ ai Se jn eat Ine Ite Putt - pe=8.432(28 + 1040106 - 4.5 + 176(46) pen 7s6kPs = suger Tar We cee Part I For Narmatly consolidated clay- og 2h Pe = 0.009(LL- 10) = 009/45 - 10) G=0315 pox effective vertical stress at mic-height of elay layor Pom V75A KPa Pym Pat bp ‘ap = 12.N/cm? « (100 em/ mye ‘8p = 120,000 N/m? = 120 kPa py= 1754 +120 2954 kPa AH (759915 og B54 — 9.254 m AH=254em Re Sean ate Terie "8 a754 ws | Sa ‘DSituation 2 (4 to 6) Part I: 1 _ Acsin 33" Lesing Losin 3? Ke= 0.2948 RewRy he y= pg = 1600981) 4 15496 N/m = 15.696 KN/m? F.= Y4O20i8)(15656)(0)2 Fin 1481 kN Hydrautics and Geotechnical Engineering is 15405 - Sag OD C= 2.688 a 2588-1 OT aee 8" 0 19-9857 N/m 81) ‘som. ware pit Key H = 0.2048 (15,696)(3.5) = 16.195 kPa. y= 99(16.195)(3.5)0) = 28 4 KN, Fy = 16.195(4.5)(1) = 72.88 KN; P= K mH = 0.2945(9 857445) = 13.076 kPa ‘B= ¥9(15,076)(4.5)(1) = 29.421 KN; Fyn eye H=¥5(9 81). 5)P= 99.53 KN; Total thrust, F= Fi ++ E+ Fe ‘Total thrust, Fr= 2834 + 7288 + 29.421 + 9.33 ~ 22997 kN, yr 454 35/3 = 54667 a Yo~ VAS) = 225 m yuas/o=1sm yo45/3=15m Location: Frx 9 DFy ‘L997 F = 2B.34(5.667) + 7288(2.25) + 29.421(1.5) + 99.33(1.5) =2251m Civil Engineering Reference Vol. 2 Situation (710.9) Given) MC=057 Gaza 313 5=1 (saturated) h=2m (GMC=Se] 2841057) = 16) f= 16188 2 G4Se yy, 28k: M1 5188) lee) Vg telat Yu = 167026 KN/ im? (saturated unit weight) be Part: Density, p= 1/g~16,7026 /981 Density, p= 1702.6 kg/m? PAR 2 Totalstress.p: = Yah. ‘Total stress, p; ~ 16,7026(20) ~ 334 kPa Part Rffective stress = pr pw Effective stress = 334 981(20) ~ 137.85 kPa DSituation 4 (101012) Giver S=1 G=27 M=1526 grams,» 1053 grams Ree ca Me ISH —1s M, 1053 MC~ 0.4492 = 44.92% Pan? [GMC=5¢ 27(0.4492) = (Ie; ¢= 1.219 = 121.5% 2. 123 iH Tee Teas OSHS = 54.81% =0.10; Dy 0.17; Da =032 Uniformity coetficiont, Cy = 2 Goarse-Grained Seal Hyarauties and 314 November 2003 Geocectinical Engineering 315 Passing No-a seve = 100% (more than 50%) ; ae The soil is either SW, SP, SM, or SC ‘The soll is either SW, SP, SM. or SC eet Gen ape bebreen L148) G.#32<6 —C,=01908 (not between 1 and 3} Therefore, the sci is SW Since the soil does not meet both criteria for SW, the soil is SP a 5 Grain size Curve eee Tm ; ot il ! | = tHe I rel {I ese 5° il 3 a ®. : i- ® = T T 1 [ ® 0 f om om a Parle Diameter, mim Soil By i D015; Du = 0.66; Deo = 21 Dy . 24 Ini ‘ity coefficient, C= —2 = = = Uniformity v= BO = BE ag Pm a 08 ag Bre Dey — 0.15(21) 36 104 fi/min j/l= 125/05 =25 An Fd= 3 (15/12¢ 00127 1 Q.= 3.6 104 (25)(0.01227) = 1.1085 x 105 f/ min (Coefficient of curvature, C= ‘Classification: ‘Retained on No. 200 sieve = 96% (more than 50%) Path b: ‘Coarse-Grained Sit Oy kag iA E Passing No.4 sieve = 90% (more than 50%) ag” yg #75 108 r/min Sends 27x10? | 36x10 ! imh/L= 125/05 =25 | Angry bP y= 0075 4 316 Qs = 75 x 1042.5)(0.075) = 1.40625 x 104 (077m # Qy= 1.1045 « 105 + 1.40625 x 104 (Q= 15167 x 10+ AYmnin Part 2: QmkiA = (= h/L= 125/05 =25 A= FD = (8)? = 008727 fe Q= 15167 «104 f8/min 15167 x 104 = ky(25}(008727) Ay 69518 x 10" fy/min Part 3 Qt 15167 x 10-4 FY/min x 58 min, V = 0.00824185 f » (100 cm/3.28 #1) V=2364em* Situation 7(19t021) Q4Q2+Q=081 > 9.4) a fy) 2OS2ELIG801Q.? _ 9.08264/)1600)0,7 Fast) ‘020° 015° Qr= 229510) > Eq.) 2 2 fap ny OOSAATSONS? _ 1026460), a0" 015° = 032460, > Eq. (3) InFq (Ik, Q,4 229510, + 032460) - 061 Q) = 0.1685 m?/s = 168.5 liters/sec In Eq. (2): Qy= 22051(1685) = 386.7 Liters/see In Eq. (3): Qs = 0.3246(168.5) = 54.695 liters/sec Geotechnleal Eagincering ydraniles and Civil Engineering Reference Vol. 2 ‘Disituation 6 22% 2) sp.gz.of wood Drafts Ix cn gr. of liquid Inwater h-5 het Neat =H = 5 In glycerine: 75 nase 135 ject ® 1.35 10/125 Eq, (2) Ut Smt = A Secsll T= 5 = 1.35h- 10125 j= 14.64en, Past I Inq. (1): 1.64 smut = 164-5 Somat” 016585 Part Weight Yeon Vox pht= 578 N ight = (9510 « 0:6585){(0.2\0.2)(0:1469)) In ghcorine (S = 135) tyarouties and 316 November 2003, \Geotechnten! Engineering (75x 1A 25NO.O7S) = 140605 x 104F0/ min ‘Qs Qe= L1015 x 10 + 1.40625 « 104 S167 > 104 e8in iam (mh/L= 125/05 =25 A= $0 Fp =008727 Q= 15167 10418/min 15167 x 104 = y(25(008727) ys 69518 % 104 fein Part: Ve Qu tm 1.5167 x 104 A2/min » 55 min = 0,00834185 fx (100.em/3.28 8 V= 2364 cm’ ‘Gi Situation7 (190-21) : 5 Se 20mm 2 o Qt O+0s= 061 > Eq. (l) t= ny DOSSALABO:?. __ neascN.EONO,? 020° 0.157 Qh=22951Q; > Bq. (2) snp) OBZ? 108266 (600), b= mil ‘010° 015" (Q)=03246Q; > Eq. (3) InFy (1); Q,+220510, +032460) = 061 (1 = 0.1685 m/s = 168.5 liters/sec (Qi =22061(1685)=3867tnery/see (Qs = 0.3246(168.5) = 54.695 liters/sec Inq, (2 In Eq. (3): Civil Engineering ‘Aeference Vol. 2 Dsinitions G2 1020) “sp. graf wood Draft= 4x55 gr of liquid In water: h=5 =p Seal 1 Bengt h-5 359.0) In glyeerine: ha 7.5= hee 135 qu = 1358 ~ 10.125 9 Eg, @) Pisco = Sal = 135h = 10.125 hm 1460em Pact I: In Eq, (1) 14.64 Spas = 1464-5 Soon = 0.6585 Parti ‘Weight Ynoet Vast Wei Weight = 37.8 N, Bisituation9 (25 to.27) Tn gyetvine (5 = 1.38) (9810s 0.6585)[(0.2)(0-2)(0.1464)] @anim 318 November 2003 Geotechnical Engineering .g 5,= 0 almost level channel) 08155 +1+0=0.4678+11+ 56) $20,008 Slope of ECL Head lost, HE = SL.= 0.0083(5)= 0.0415 m Value of r: . can Tamsin | v= SL» AABN eae ra ste AL 30M eaten Bion spay =f) oom Se send) Raw SETS n0si79m aaue- 1 oaap omey2 n= 00173 Civil Engineering Reference Vol. 2 319 GB Situation 10 (28 to 30) eral rjc of ered ace Pac: B-vhA F; = 31 = 08)1.5)[26)] = 70692 kN Fayhar F.= (OBI x 08)Q)[*40)8)] = 3.548 Fa Fi 2h =7062+203.544) Fei772kN Patz: Fey=Fiyt Fey y= £6)" Lm y= %0)=075m 117.72 y= 70682{0) + 225544)(075) y-09m Part: Emp) (3) = AiG ~ n) + 250-19) * 70,632(3 =) + 263.544) - 075) Ty= 41202 1eN Hydrautics and. Givil Engineering 321 320 November Geotechnicatinginecring [| Reference Vol. 2 ata Part aS ‘When the vortex touches the bottom of the tank: the upper surface will Weight, Ws Mass « g {ouch the top rim ofthe tank. This is because tee volume of paraboloid is W= 1500x981 = 14,715 N ‘one-half the volume of eircumseribing cylinder. Ws 14715 KN Part 2: Ww, 175 Unit weight y= T= FE Unit weight, y= 12.366 kN/m* Past ais. 12066 Specific gravity, = FE = Specific gravity, s= 1.26 4 2 Situation 12 (24 t0 96) 2 oF sg - 525 md/eee 2031) 425 rad sec x (1 nev/2x rad) x (60 sec / 1 min) = 518 rpm Figae (a) Foun (2) Powe (0) Part 1 InFigure (6) Volume left Vi= EAs +4 Aa + Aa] Arm A= 0 An HQHO)= 6 me Volume lf, i= = [0+ 46) +0] Volume ie, Y= 16m Part2: In Bigure (¢} Vin Anna b= E@ped datz 322 November 2003 “Seat No: ‘CIVIL ENGINEER Licensure Examination ‘Sunday, November 23, 2003. 05:00 a.m, - 01:00 pa ‘sTRUC NEE ID CONSTRU: ie SELA INSTRUCTION: Scloct the comrect answer for each of the following questions ‘Mark anly-one ansyrer for each item by shacling the box corresponding to the leer fof your choice on the answer sheet provided. STRICTLY NO ERASURES ALLOWED. ise pencilno. 2 rly. ‘MULTIPLE CHOICE Situation 1 - The block of mast m= 1.6 slides slong the horizontal ough plane ‘ts showniin Figuze MESS" The potion eoorciate + is meagared from ta tivleformed. postion of the ideal spring, which hae a slips of k= 30 Njm At 70 the block is inially moving tothe righ with speed =~ 6 mys. 1. Which of the following most nearly gives the valte of x when the block first ‘comes to nest. A13m Cc 16m Lim D.09m 2. Which of the fotlowing most neatly gives the acceleration ofthe block when. the block first comes to rest. © 36m/st A T2m/st B. 18 m/s B. 24:m/st 3. Which of the following most nearly gives the speed of dhe block when it reaches = O for the second time. ‘A dm/s Sms B. ém/s D. 4mn/s Situation 2 = Figure ME28 shows 5-kg slider which i lensed at A ond slides ‘along a smooth circular rod until it reaches point B. ‘The stifess of the spring = 480N/mand has an outstretched longi of 750 mm Tihick of the following most nearly vee the change in grevitational potential energy ofthe sider ‘256 Jo © 26s Joules 3, 303 Joules B59 Joules ‘ 5. Which of de flowing most nearly vs the change i eat potenti ioe ©. -t12 Joules Bi joules D.-88joules 6. Which ofthe following most nearly gives the velocity of the slider at 3 A Ss6m/s © 624m/s B 7Am/s D.358.m/s Situation 3 = The rigid beam shown in Figure AN-12 is suspended by thee cables, The beam is horizontal prior to being connected to the cables and is 10. remain horizontal after being attached to the cables. The heam weighs 1764 and Construction TE Reference Vol. 2 323 eee KiloNewtons, 2. Which of the following: most nearly gives the ratio of the force in the bronze ‘able ta the force in the stcel cables. ‘A047 C0389 B. 154 D. 257 & Which of the following most nearly gives the stress in the steel cable in MogaPascals AI C168 B. 358 D. 98 9. Which of the followitig most nearly gives the elongation of the steel cables in millimeters ‘A. 0285, C oaa2 B ome D. 0615 ‘ Situation 4 ~ a structure js shown in Figure AN-60, 10. Which of the following mostmearly gives the reaction at A. in kiloNewtons. A. O0kN 7S, Teak B, 385KN BD. 281 kN Which of the following, most nearly gives the shear ata section 7m from C. ‘A. ZEN C 7O5EN B. S56 kN D. 98.5 EN BE Which ofthe following most nearly gives the moment at a section 3 m from A. 625 kN © S7kNm B 420kN. D. S47 kNem. Situation 5 A simply supported timber beam has a span af 6 m long and carries a uniformly distributed load of 25 kN/m aver its entire span This Toad stray incutes an allowance for beam weight. ‘The timber is 80% sts grade Apitong. The allowable deflection ‘8 1/240 of the span The Fraparie ot thuppine woods are prenan Tabi 43 % Heh of the following gives the smallest dimension of the beam such that the allowable shear stress is nai exceeded: “A. 20 mum x 440.mum © 250mm 500mm B, 150mm x 300mm 200 mm » 400 mm 1 Which of the following gives the smallest dimension of the beam such that the allowable bending stress is not exceeded 4, 220 mm «440mm ©. 250 mm x 500 ram B, 150 mm = 300mm D, 200 mm x 400mm “15. Which of the following gives the smallest dimension of the beam such that the allowable deflection is not exceeded: A. 220 mm 440 mm ©. 250 aum x50 mm B. 150 man «300 mum 200 mm x 00 mm 8 6 ~ A simply supported beam with span of 8 m is subjected toa ounierclockwise:moment at the left support and a counteclockiice ‘moment at the right support equal to 25% of the moment at the left support, oth acting in the plane of tht minor axis of the beam, The bos i not restrained against lateral bucking. The beam is A36 steel with Fy ~ 248 MPa. The provisions of the 1992 NSCT is given in Figure ST-64. ‘The beam E6121 62 whose relevant properties are: 324 November ‘and Construction Ry 0053 [001m a2 0833 = 0.00207 me by= 021m c 16. Which ofthe following most nearly gives the slenderness ratio, below which is short with respect to lateral buckling. ‘ASB ~ C. 863, B. 759 DB. 957 17. Which ofthe following most nearly gives the allowable flexaral stress in the compression flange acording 1) equation 46a or equation 45-6, as apphieabie. Pho MPa © 687 MPa B, 863 MPa D. 754 MPa 18. Which of the following most nearly gives the maximum value of the ‘moment atthe left support in kiloNewtor-meters ‘A 2547 © 3021 B 2758 D. 2574 ‘ Situation 7 A simply supported rectangular reinforced concrete beam has a span | Of 6 m, width of 290 mim, effective depth of 800 mm and total depth of 600 mmm. "The beam is reinforced with 2Saan-diameter bars. The concrete strength f= 28 MPa and the steel yield strength fy = 415 MPa,. In addition (Ie bad ae a ope eigen tie sot id coor slab that is 5m wide. ‘The unit weight of concrete is 24 KN/m’. The beam is tobe analyzed and designed using the Strength Design method. 19. Which of the following most nearly gives the total service (anfactored) ‘uniform dead load acting, on the beam, in kN /m: A216 283 B 152 D. 923 20. Which of the following most nearly gives the nominal moment capacity of the beam, in kNem. AS cm B74 D. 354 21. Which of the following most nearly gives the maximum uniform service: (unfactored) live load acting over the entire span that can be supported by ‘beam in kN/m. ‘A 3652 C446 B 747 D. 785 Situation 8 ~ A tension member in a bridge truss ts a welded built-up Heshape 13 mmilong and 400 mm deep. ‘The member is composed of three plates welded, together. The maximum load on the member is 1,600 kN tension and the runimwur load is 260 KN compression. For this problem, it can be assumed that compression is not critical. The member should be designed for a design load equal to the maximum load plus one-half the minimum load. ‘The allowable tensile stress is 185 MPa. The maximum slendemess rato (Lf) is 200, 22. Which of the following most nearly gives the minimum area of the member. ‘A. 8756 mm? C. 9867 man? B. 10786 mm? D. 9351 mm the approximation that the radius of gyration 7 fe approwimata qual to onevourth the why of the ange whch oft foley nea used as a preliminary value of the lange width: ‘A. 250 mm C240 man Bk Using te pr de of the Mange wid sing the preliminary value of the flange width obtained above, which of the following most nearly gives the required plate thickness A 87mm © 104mm B63. mm 0.128 mm Situation 9 - 4 bil of quantities is to be prepared for ths isclated column footin shown in Figure DW Unit weight of sels 7860 kg/tns Binion coon a (reoyrehlrsrt Om = ich lowing most nearly gives the unit weight of the 1é-mm- alam reinforcing ba, In hg/om fg ee © 324 B 0.95 B. 158 26 Which ofthe following most nearly gives the quantity of 16-ma-diameter ‘A199 ca BMS D. 147 27 Which of th following most nearly gives the quantity of 16smm-diameter A 3L cm Bas bits Situation 10 ~ Government constracton contracts are governed by Presidential decree 1584 which prescsbes that change orders See linked wo Segre eriginal contac ree provided tha na pajr py ie sal be neal by more than 100% of ts original cont. Major tems ts dedined es pay less representing at least 20% of the AAE or the first two items heelap the Highest pervenlage of the AAE or those indicated or specified te {he Instrvetionto Bidders For this problem, dhe AAE of «cern poner mnllion pesos 2 Witch of the following most nearly gives the amount at which ain i becomes a major item? © nome ‘Pa Milion ©. PS Million we abfsilion . 3 Milion Certain major item has an original quantity of 4,000 units. The winning Wer Bl at price of 0 forts tes. I the only change eens this item, which of the following most nearly gives Uhe marci cee ees ing most nearly gives the im amount of A, PS Million © Pa Million B. P3 Million D. P6 Million SI. Which ofthe following most ney gves the maximum increase in ofthe ean ifthe un price is uncharged? er A. 4,000 ¢ 3,000 ‘B. 5,000 D. 6,000. 326 November 2003 and Construction ; a } : p= 03 3|_ FgweME35 202 e # a 2 i é “a ; s aye 3 5 £ am i 8 i Figure 08 i : | (c) 8 Figure AN-60 328 November 2003 Figure 1-68 Section 451.45 of the 1992 NSCP gives the allowable stresses, in MegaPsscals, for beams not restrained against lateral buckling as follows: 1. Tension: A=06F 2. Compression: ‘The larger value computed by Formula (4.5-6a) or (4.5-6¢) md GEA coapoliahie bulast ore un Ga0E- oe FesnoOc, . L , [SSz ONG, Ry Bi wf > ated When 3 1055x10°C, L, pe When = 3 ® 1,170,000, eee 35-62) w/Ry hea) ‘Or, when the compression flange is solid and approximately feclangular in crosesecion and ir area isnot les thant that of the fenvion flange: pe BOS «sn it Where = ih Gy = 175 + 105(M,/Mtg + 030(04/M2 fat nat mote than 23, Sehore Mis the smaller and Me tho largor bending: momont [ fat the ends of the anbraced length, taken about the strong axis of the member, and where Mi/M: the ratio of end moments is Prstive whey "and Ma have the same gn (teers Parvature bending) and negative when they. are of opposite Signs (tngle curvature bending), When the bending moment ft-any point within an unbraced length is Langer than that at both eres of this lengthy the valine of Cs shall by taken as tity Cimay conservatively be taken as unity for cantilever eae radius of gyration of a section comprising the compression flange plus 472 ef the compression web ares, taken abut af sais inthe plane of the web d= depth of beam R Ty= width of flange p= thickness of flange ‘Table 2: Working Stresses for Visually Strese-Graded, ‘Unseasonedl Sructural Tunber of Phil ne Wands BSSESSE8 Figure AN-12 arate ‘Reference Vol. 2 331 TL Solutions to November 2003 Examination “DSituation 1 (1 t0.3) ‘Note: Work dlone in stretching a spring from x to x>8s ¥ak(ar- a2) We mg = 16(981) = 15.96 w ain won et Work-encrgy equation from @ to @: Eye Werks Wie» KE ALE) 40) - 0.201569), = 0 15n+3.1998n -288~0 n= 1285 m f020 New Part Hh: When x= 1.285 m Force on spring, F, = kx = (30)(1.285) = 38.55 N Friction, f= 0. Note: The friction is zero because this is the stage where the block reverses its motion, hence it’s still at rest Unbalanced force, Fy =F, = 38.55 N REF = m= Fy 3855 — 1.60 a= 24.09 myst Part I Pat Work-energy equation from @ to @: Ex Works ihm KE, (+ ¥4(30)(1.285) -0.2(15.696) (1.285) = V4{1.6)o3? fm02 oo 509 avs New 332 November 2003, 1 Situation 2 (4 to6) Natural length of spring = 750 mm = 0.75 m Part i: APES! ha) With datum at A: ha =Ohg = 0.75 m BIY0.75- 0) = 36. 7875.N-m Part I: APEgng = ¥aK(¢02— 242) 34 = 7512) ~ 750~ 750 mun = 075. y= 10007 ~ 075 = 03107 m APEsyong™ ¥4480)(0.31072 — 0.75%) = 111.83 N-mm ‘APEspomg = “E1183 Joule Part OL ‘Work-Energy equation from A to B: ‘KEq:+ Work Wh = KEp 0+ 11188 + 36.7875 = YS)ox? by = 77 mys Situation3 (7 109) 28+ Pi 17644N > Eq. (1) 1-6) P1000) F600) 600(200,000) ~ 900(83,000) Be 0.389 > ratio ofthe force in bronze to tel Pmasr, Eq. Subsite Fe ~ 0.299 Pto Ea): 2P, + 0569 P= 1754 F.73896 KN ne ‘Stress in steel cable, 5, BE = 125 Mra 600 Vol. 2 333 Elongation ofthe cables Note: Members EC, EB, and ED are truss members, Its subjected 19 pure axial load only [EMp= 0] Ay(5) = 50004) + ¥485)05) x 3 6.5) Ay= 21225 kN [EFy=0] Ay=50+%6(8.5)(15) = 113.75 kN, Rym fagteay® = ffai225)* (11375) Ry=24081KN > Past I vant 28,0- 7 AtjointD: Foy =212.25 kN AtjointE: [2Fy=0] FoxsinO= F; Feesin 47.7263° = 212.25 Fey = 286.848 kN 334 November 2003 Shear 7m from C (atsection y-y) =. 5.2 20 Y 7a ae ee (Vp)ogie 970.515 kN Moment m from C: i 7. 4 (at section x-1) (Maden = Fee cos 8(3) ~ 50(3) = 286 BAB cas 47.7263° () ~50(3) (Maden = 428,865 KN-mn 2 Situation 5 (13 to 15) (Vshngn = 132) = 113.78 { * Aen LS =e From Table 4-3 E=731% 10 MPa f= 165MPa. Fea 173 MPa 4 r (sem Pant: Shear, VR Hw FAO) w 751 av B Bat ‘Chsce B:150 5900 = 55.000). D> ‘ SO, = 25 6a > Fe agua) Choice 2 200 mn x 400 mn 375,000) 1 = fo™ ——____ = 1.406 MP < F. (OK) Franc a00) Bane Part I Manx 3 3 T125kN-m OM oe ome SP & bd? Note: F's Fi depending on the dimension of the bea. j= Hs Since Fy = 16.5 MPa and F'y is even less than 165, the section is roe ‘Ghoice D: 200mm = 400mm pw tttz sea) -200(400)? Since F = 16.5 MPa and F's even less than 165, the section is inadequate, =2109MPa ‘ - ae T= 15.548 mira Solve for F aceording to the code: ‘This fs asingle-span beam with equal end moments: = 1.84, = 1.84(6000) = 11,080 G- ear ae =10510 Thus Fi.= Fi= 165 MPa> fi (OK) Pat DE eS) sm. 2546)" 1000)" SSHEL 240 384(731x10°)F 1 =2,308,481,532 mt tree 23] ‘Choice B: I = 150 3/12 = 337,500,000 < 2.308,481,532 Choice D: {= 200(400)/12 = 1,056,666,667 < 2.308,481,532 Choice A: J = 220(440)}/12 = 1,361,706,667 < 2.308,481,552 Choice C: J = 250{500}'/12 = 2,604,166,657 > 2,308,481,532 (OK) By formula (45-7) 8, Fp~ 830006, ee eerie 220406) opty O.6F, = os(2as) = 1488 Pa ‘Therefore, F.~ 1928 MPa Part 3, maximum monvent at lef support: G= 175 + 1.05(0h/ Ms) + 050(My Ma)t ‘Mi=0.25M Allowable bending stress: = M Intension: = ‘Mh/fe= +(025M/M) = Fy= 06h, = 148.8 ‘Mi/Mo= +025 (positive because "reverse curvature") iGiipoties (G= 1.75 + 1.05(0.25) + 030(0.25)? = Fy= 1328 MPa G=201 <23(0K) Use F; = 132.8 MPa frosococy _ [TOSOOOOM) _ 5 28 fsszo.o00c, _ [SSa0.o00(z05) 5 28 Past 1 Slendlemess ratio, below which the beam is short with respect to Ia buckling: Slenslerness ratios OEE a 75.25 ua 2 since PEOOICE flange width art Am 2brt) + (€- 25) bu ty 260mm ‘= 400 mn Situation 9 (25 v0.27) Part I: Unit weight, w= punaAy Unit weight w= 7860)5} (0016) = 1.58 kgym Part U: Length in footing = 05% 4x2=4m Length in column = (0.15-+ 382) x4 =15.88 m Total length, L'= 4+ 15.88 = 19.88 mm Part IL Weight = arx L = 1.58(18.85) = 31.4 kg Situation 10 (28 to.30) Part I: Amount = 20%(15) = Fa Part tk: ‘Griginal contract cost = 4,000 x 7,000 Pant Maximum amount of change order = PAM + 25%(P4M) = PSM Pest Maximum change in quantity = 4000 + 25%(4,000) Maximum change in quantity = 5,000 units ING, d& TRANSPORTATION ENG'G, SECA, Ni Select the correct answer for each of the following questions Snbone ansaver fr enc item by shading the box coresponiding tthe letter four choice on the answer sheet provided. HICTLY NO ERASURES ALLOWED, Use peneil no. 2 on TIPLE CHOICE U1 hing, 6 = 1.2025, whats the Vilas of log, 11? A165 C1 B17 DB 1SL ‘What is the value of E in the following equation? 2et vax? +723 4100410 | A, Bre, Drok (e-Det+3)? aT 743 G43 A? co BS D-l ‘A salesman started walking from office A at 930 am at the rate of 25 kph. He arrived office 8 12 seconds late, Had he started at A at 900 am and Walked at 15 kph, he would have arrived et B one minute before the dred Kime. At what time was he supposed to be at 5 10:13am © 1022am B. 10:16am D. 10:18am Whatis the value of xif1, %4 1/%,1/10, form a harmonic progression? a6 cs BF DS Find the area of the spherical triangle ABC having the following parts: Angle A= 140" ‘Angle C = 86° Angle B= 25° Radius af sphere = 4 m A328 mt ©3879 me B a1aine D. 3456 m? The upper and lowes bases of the frustum of a rectangular pyramic are3 m by'4.m and 6 m by 8 m, respectively. If the volume of the solid is 140 m3, how far apart are the bases? A65m cam B 45m DL35m 342 May 2004 ‘anportion Eainesae Ri neincring 45 7. (A closed conical vessel with base radius of I'm and altitude 25 m has is ‘xcs vertical. In upright position (vertex uppermost), the depth of waterin ‘ALSi00 am, boat O left the port in the direction N 45" E at 26 knots AP ea0 fim, boat & lft the same port in the direction S 30° E at 2 knots, iow tng iis 30 em, if the vse was inert rein Hower, How dept ezine besepthget aban, Fapeyon Se lane water ini? ‘45531 ous ‘A. 1968.cm © 2086 em carrey Bis7Sem D. 1748 cm a D. 24.58 &. A rright prism has a base in the shape of regular octagon inscribed in @ AF Bvaltnts the integral of cos? Sx with mts from 010 4/6, frumibcanby loco Hus wbtede of 5c find svekume tae A. 0817 © one ‘A 12i26 © 13897 B 0305 D. 0218 B 1183.4 Bane ° THE Find the voluine inthe first octane, bounded hy the following surfaces: 9. Find the equation of the line with slope of 2/3 and which passes through x= od point of intersection ofthe lines 4x ~2y +1 = and x = 2y += 0 atm y+ 22 A. dr 6y+9=0 C 6-810 A. 005 © 007 B Gr-4y+9=0 D. ax-6y-+11=0 B 00s . D. ang 10, Given the curve 362 + 9y2 - 360. Whats the length of its lnfus cect? BRR A 12-0n car moving ot 30 Klometers per hour bumpa the rent ofa 1:b4cn ie at Daring ec ago Meme tone np te rf B O75 Dr ‘The bumpers got locked after the collision. What is the velocity of the sare linmediately afer impact? 11. Find the area enclosed by the following curve: ‘B26 ou ge Ie= by 23 =D B Bo ‘A. 8547 square units ©. 7855 square units B. 95661 square unite D, 6853 square units 2, As iron balls projsted upward witha velocity of 300 m/s at an angle of 80" fom tho level ground. When wil ibe ball all back to the greets 1A triangle have vertices at (0,0), (6,307), and (9, 70"), What is the perimetat 4. $5.8 sec ©. 58sec ofthe triangle? ‘ B. 206 sec Di1smin oe 5 Beat Ea rm B. 2674 units ne L Anequipment costin, 1000 has an estimate scrap value of 25,000 at the endl ofits economic life of 10 years. Using the Double-Declining Balance jai ie (ges psiow ma ciel ed pick wil pesoe pre eeey Make eae ald a te Dbl Dect Bale 50 4" thowund peace per gear, while a sand vestment plan AL PS3SS4 C. P3a758 generate 200+ i thousand petos por Year" What i he not exes profi) B. P2282 D. Pa5712 Jou invest an the second plan instead of the fist plan up to time the 8 aa Se equal prolit? shoes factory has a production eapacity of S000 units per month. The Paseo ©. Pisano BA shoes (actor has « production capesity of 9000 units per orth. Th B. P1687 500 D. PuS#9.100 Fined monthly operating cost = P00,000.00 Material and labor cost = P201.00 per unit 14. Given the function y = x(e + 1)’. Detormine the value of its second Selling price per unit = P500.00 derivative at © What isthe profit per month if the factory has an efficiency of 80%? 4. 36 3 A. PLA50,000 P1,280,000 Bd D3 B. P1360000 D. PIss0,000 15, ‘Triangle ABC have sides measuring AB = 7 m, BC = 5 m, and AC = 9 The present value of an annuity of R pesos payable annually for 10 years, ‘with the first payment at the er ares wort ayient af the end of 10 years Is P10070120, If mency i % © Pony B. P2500 Brana What isthe width ofthe largest rectangle that can be inscribed int with Vngerelde i he revangie dang the Se ide A 247528 © 1984m B 131d D.a5m TA lot & bounded by the following three straight sides. AB N 45" E 160 Jong: BCsand AC 190 m long, in clockwise direction, From point E, 100 from A and on side AB, a dividing line runs to D which is on side AC. area of ADE is to be 2/5 of the total area of the lot. Calculate the length DEA the total lot area is 11, 643.88 square meters Acme 22m B 933m D945 m. 25, A rectangular lot has a correct aroa of to hectares. ts length is twice ‘width. If the lengths of the sides were measured with a SO-m tape that (0.02 m too Fong, compute the error in the area of the lot in square meter. ‘A, 15,9904 6.095, * B 16.0032 D. 7.0003. 26, Acarves road 74 m in radius has superelevation of 0.12 and a design sp ‘of 80 kph. Determine the coefficient of friction between the tires and th aa A O38 068 ‘ Boss D036 27. Find the degree of curve of a central simple curve if it has a spiral curve If ‘mang on two sides, on which a car travelling at 75 kph will not skid, are basis. A Bas° 755 B 635" D, 4.65" 28, A downgrade of 3.2 percent meets a rising grade of 46 percent at Sta 73 10 wrhere the elevation is 996.” A sag curve, 440 m long, connects sgmadelines from A at the downgrade to Of Bin meters. A. 1008: © 100m B 101m B, 101d m, 29. The center height of the road at Sta, 5+320 is 425 mut. AtSta, +4201 ‘atthe upgrade. Find the elovati 18 mill. The ground slopes uniformly at 5% from Sta. 5+320. How far i eters, from Sia. $+320 toward Sta, 5+420 will the excavation extend?” ‘A, 7283 m_ C. 7387 m B. 7025 m, D. 797m 30, A line of levels was run from point 5 fo point 6, 8 km apart, The aven backsight and also foresight distance was 100 m. At every tuning pl the rod settles by Sem. Find the correct elevation of point 6 based om recorded elevation of 386 m. ‘A 383.79 mi © 38m B. 384.24 m D, 383.97 m ‘ANSWERS: 4 1B GC] «NC «BE OIA 260 2D 7A Me we we ae 34 8A 0B DZD 28a 4B 0D 4A 18D Be ScD sc _mB xc Solutions to May 2004 Examination log, 6 = 1.2025 HSE 6 red ein = aogcnt = ET = ray fod Bet t5x +774 10xe10 A, Bre , Dive 7 ee +3)? ws | aap Bat +900 + 728 + 103 + 10= AleERS)+ (Bx-+ Qla~ 12 +3) +r Be -1) Sere 2(a}¢+ 9(0)4 7(1)2-+ 101) + 10= Ade + 38 ee + 9+ 7+) +10 Ade 5 2M 43:9 +7284 10e +10 = (Pat+ 12st 18) + (Bet + 9Ba?— Bs - Bx + G8 +3Cr- G2- 30} 4 (Dx?- Dx + Ex 5) Bet 4 329-47: + 10k +10 @ Aas (C_ Ayes (12438-C+ Dj? wge-as-De ees geese) , ‘Tivs equation is an Ideniity, Fach term at the left side of the equation smiust be equal tc each term at the right side, aes 2 = (2-4 Byes 98 =(C- Bix 2=253,2-0 Bec wared eos Constant: 10=18-3C-£ 10=18 -3@)- E:E (12+58- C+ Dye 2438 -C+D #12+30)-3+D; D= 2 2s Thus A-2.80,Ca30=-2, b= Ort 32° 47x? + 10r+10 NG? +3? 346 May 2004 ee Gs Mathernaties, Surveying, and ‘Transportation Engineering Lot $= the distance from A to B = required time of arrival. Lewving at $30 amand walking at 25 kph nda oftavele 9.5% 16/3600 =~ 949667 $= speed x time of travel Sa 25-9456) E90) Leaving a1 200 am and walking a 15 koh: Tee ottweel 1-9 1/eb= (9087 S=1.5(t- 901667) Eq, (2) [5-5] 25(t-9:49667) = 1.5(t-901667) 251237417 = 154-19525, 1=10.2167 he =1023.am q anal a ‘Harmonic progression: 1.%4 1/3 W198 5+ ge 5 3g “Their reciprocals 1 4, x, 10 form an arithmetic progression. By inspection, the common difference of A.P.is 3. Thus=™ 7 i 5 B= A+ B+ C- 180 = 140" +75" + 86° 180" ~ 121° Radm » May 214) 40- 4 [4 + (68) + [OxaVex8) | asm Lahti ees postn Lot Y= volume of the yessel Ve= volume of air volume of water = VV, =i Jn upright position . By stimilar solids (vessel and ait): zg Va Ve as SE necny In inverted position: By stmilar solids (vessel and water): aca) Volume = Ayn cos22.5*= 2 = 5341196 cm Ap=Yartsin ds ~8 y= 4641196) in 5° Au= 828127 com Volume ~ 82:8427(15) Volume = 1242.64 cm Solving forthe point of intersectin, PE dept Te 0 30) r-tys4eo 22) ‘Mathematics, Surveying, and Civil Engineering 348 May 2004 ‘transportation enginecriog WML meference Vol. 2 349 Subtract Eq. (1) ~ Eq, (2); =n pane ae E amose; are cor mya or-3=0 as 7 1s B= [lsoestyie = [sor } = 1875 (thousand pesos) =5/P Phisat(, 5/2) By point-slope form: ‘Second plan: y= mie= 7) a? 4-5/2 2/3061) AE 200+ 5e; dP = (00+ 50 dt Gy-15=4x-4 = ac 6y +11 =0 te Sz)” P= |(200+5eat =|200+=17| = 3562.5 (thousand pesos} ey fom ssoe «frond ] (thousand posh at. we The given curve is anellipse with standard equation of == 4 = 1 Excess profit » 3562.5 - 1875/0 1687.5 thousand pesos Badan? Excess profit = PL687,500 nets at _ 20y? _ (x + IP = x(29 + 3x2+ 32 + 1) Length of aus rectum = 2 = 20 34 9584 Sat x Vind #9 6x21 ait Potaeinee a+ y= 10r= 10y +25 =0 racine ate (2— Toe + 25) +(y2 — 10y +25) =-25 + 25 + 25 yf =120)2 + 100) + 6=36 (@-5)?+ (y-5)?= 25; Radius 5 4 1s Ares = aR? = (5)? 78.55 square units This isa maxima- rinima problony, and s az is similar to CE Board From the figure shown: : November 2003, s2= Mt + 6 — 249)(6) cos 40" am ie x= 5.8538 By principle, the rectangle will be Perimeter= 9+ 6 + 5.8535 gest ils = 49) an Pertmoter= 20889 unite am Soa ae ‘ om Solving for i B= 72+ §-2(79(9) cos; 0 =33.557° h=7 sin =7 sin 38557" Ji 3.8694 m fs oF ons Solving forthe time when the two plans yield equall profit: 50+ i= 200+ St B-5¢-150=0 (+ 10\t-18) =0 = 15 years - time when they generate equal profit Thus, y=¥4{5:8694)~ 194m > width x=4Q) =45m > length 350 May aie Lett be any time after 5:00 am, Time of travel of boat O=¢ Time of travel of boat = {0.5 Ste (26e) + (92K- 0.5) = 2(258[ 32(t - 05)] cos 105° Sm 676F + ORME — 102i + 256 + 450.6712 215.341 St= DiSa67e - 1039.34 + 256 s= faaae7e 1239341 +256 aS ___ $261 34r-123934 at aylisoert® — 123938 + 256 At6am,t=1 as 4261 34(1) -1239.34 aS =F.603 kaos ft 2y/2130.67(1)? =1759 34(1) +256, for sven air Since a a3= Fr thiscan be solved using Wallis’ Forarula. 7 pow 2 MORE) f& om" 3 Tey 1 For the surface xt = y-+ 2: Wheny=8; 2°22 > parabola Whenz=0; 2°=y > parabol Whenx=0; y=-2 3 stratght line By law of conservation of momentum: nO + ms = mm Oy mB 12@0)+ 18@0)= 1.204180 b= 24 kph ‘der ean 4, = vy v yr tof tag P; wherey=0 O= G00 sin 3071 = HOS) 150! 4.9058 = (150 - 4.9050) =0; 1 = 30.58 sec 352 May 2004 Production per month = 9000 80% = 7200 shoes Profit = Total sales = Total cost Profit = 7200(500) — (800,000 + 200(7200)] Profit P1,360,000, Ay = 19,984 00959 me Error = 20/000 - 1998,00959 = 25.9904 mt ‘The minimum radius (in meters) of curvature of a circular curve is 2 ‘ where» is the design speed. in kph, © if the aes fy" Superelevation- and fis the ébefficient of friction. a i orp t= f-20561 5 Rien 1 7 Ae ‘R{q@sa08)" (i+ 008)"7008 * ‘The desirable length of spiral in meters is given by the expression: p= DeB6u" 100,701.30 = R= P30000.00 where vis in kph and R in meteres. 100 = SSO: p= 151.875 201518750 is Aare 11,649.88 mt ¥(160)(190) sin © = 11,645.88 0-50 ;D=7585° Anwe= FAue YS{1OO)(y) ier = 2 x VA(L60)(190) sier y= 126 In tangle ADE: ean) 32 = 100 + y2~- 2(100)(4) cos 22 = 108 + 121.6" - 2(100)(121.6) cos 50° x= 95,676m_ ‘Truearea, Ay =2 hectares = 20,000 m? Measured area = Aa Length of tape, L3 = 50 ‘True length of tape, L; = 50 +0.02=50.02m T= som Elevation of B = Elevation of PI+ gx(L/2) Flevation of B = 996+ 0.046(490/2) = 1008.12 m ¥/4.25= 100/605, x= 70.25 m Number of setups = 8000/00 = 40 Number of tiring Correct elevation ot oints= 40-1 =39 (6= 386 - 003(39) = 364.83 m CTION: Select the correct answer for each of the following questions. uc ne anime foreach stem by shading the box corresponding tthe leer Bee ceercaet eect ae INO ERASURES ALLOWED. Ute pencil no, 2 only, ‘TIONAL, ga. — Given the laboratory réqults of the Atterberg Limits Test in Figure 15. “otthe water content versus the-fuamaberof blows Determine the nearest valuoto the Liguid Limit ofthe oi A 18% C45 4% B a2 D. 45.0% Determine the nearest value to the Plastic Limit of the soll, A 369% C. 358% Bas D, 36.5% Determine the nearest value to the Liquidity Index of the sail A12 C08 B13 D.05 tion 2 The results of a consoltdated-drained triaxial test conducted on a ‘normally contolidatedt a8 follows: ‘Chamber confining stress = 250 kPa Deviator stress at failure ~ 950kPa Calculate the angle of fi ‘A, 2432" Cc iar B 2167 D. 198 Calculate te shoar stress onthe failure plane ‘labs EC’ Hoke 8 160 kes D.MSkPs ‘Gallculate the effective normal stress on the plane of maximum shear, ‘A. 207 kPa C. a6 kPa, B BOsEPs D. 353.kPa m 3- A confined aquifer underlies an unconfined aquifer as shown in Figure 63. Given the following: Dj = $9 m, Dz = 41 m, Hy ~ 45m, Hi ~33 m, Kj= 35m/day, Ky = 27 m/day, L = 2 ke. _Galeulate the equivalent coefficient of permeability in horizontal direction. ‘&, 978 m/day C3641 m/day B. 387 m/day D. 31.m/day Calculate the hydraulic gradient. A 0008 0005 B 0.008 D. 0003 ©. Calculate the flow of water from one stwam to another per meter width, ‘A. 18.21 m?/day ©. 11.87 m'/day B, 1425m"/day D. 1675m°/day Situation 4 — Given in Figure 98 is the borohole log in a project site. The propose Ti. Consilering shear failure only, calculate the safe bearing, prosnare on footing 6 mlongty 125 imide, usinga load factor 2 Giese = CNA +03(8/1)} + yD/N, + Yay BN - 0206/0) = Aur FS + Dp Wading will ona he stess of 12 Newton por aquave certo A 1574Ps Gmusun 10, Determine ihe nearest value to the buoyant Unk weight of the clay iy B 2876kPa Sak ijt BB. Whatistho safe eal ond ofthe footing? 1054 © 724 (vsti Pate Baye D, 28 2125 iN DASaLAN, Dotermine the nearest value tothe effective vertical stress atthe mid : ae ie cetaech pa Giistion 7 — the verte plate shown in Figure 85(2) is submerged in vinegar of the clay layer, in kPa. C1857. having a specific gravity of 80, shi of A. 165.3 185 AAS & $f ty of ‘use unit weight of water™= 9. im iss < ier 1, Which the folewang gives the react alse tote ection of enter 12. Determine the nearest value to the average settlement, in cm, of th of cree Aral eee eating aurtaeet Ar 624m i 7.78 mm normally consolidated clay layer. Use compression index C, = O00%EE tM B 652m fo een th ma eas. 21 Which of the following most nearly give the magnitude of the hydrostatic B 208 DBS. farce on one side of the plate? s A. 1102 kN een Situation 5 - A retaining wall S m high supports cohesionless soil having ad B Lan kN Crain BL, Which of the following gies the nearest vahar othe location ofthe cent of pressure from the vinegar sulace? ee gage Ms 575m B. 655m D. 465m Soe ees: 22 The water will just reach the rim of the vessel: ray of 170 S/n, ale of sowing ister Sard wo ae 0.68. The surface of the soil is horizontal and level with the top of the wa ‘Negiectung wall ition and using Rankines Formula for active preseute colesionlecs sol: 13, Find the nearest value to the total earth thrust on the wal in RN per ‘metor if the soil is dry. A. 592 Cc. 375 B, 83.6 D. 764 14 Find the nearest value to the thrust on the wall in KN per linear meter ‘owing to inadoquate drainage iis waterlogged toa level of 25 m below surface, , A. 157.2 rpm C 184m ‘A ai2 cm4 Sg nb O42 pm D. 119.6 mpm Baas D. B42 Bere apt olwains a tie center ofthe wean oe ase ous 15, Find the nearest value to the height above the base of the wall, the ‘acts during the waterlogged condition. B. 163.3.xpm D. 1555 [2k There is no water at the Botiom of the vessel within 200 mm from the A. 258m lim B. 142m D, 185m ‘vetical axis , ‘Ao 1459 rpm © 1897 2pm Situation 6 - A soil has the following properties B. 1023:pm D. 17421pm Unit weight, y= 192 KN/m! Coheston, c= 50 kPa tion 9- Duc to govemment water privatization, « bustling metropolitan city Se now home to substantial and continuous water supply While before supply was only 20lters/encon od pipes, tis non! 350 ern/ se sng Ang f friction = 10° rere sec rannis ‘Assume local shear failure and use Figure 38 to get the bearing capa pianew steel pipe network shawn in Figure 08-02(4), oe factors. sess 16. Calculate the net bearing capacity for a strip footing of width 1.25 m Gepth of 43 m._ The Teszaghi's ultimate bearing capcity equation is gi by gum eNe# 7D/Nq ¥ay8 N A a373kPa soars Bj 2518 4Pa D. a725kra $= 0015 (for all pipes except te outlet pi fo oid erouthtpipe ue PS) B ime following closely approximates the flow af water in the upper “152 /s Cis B. 1351/5 B. 100 5 «358 May 2004 ceonecnnieal Engineering Volz 359 Dh. Which of the following closely approximates the flow of water in the lower branch? A. 150s cmos B, 198 V's D. Way/s 22. Which of the following closely approximates the energy loss per unit mass Hetwecnthe ilctand the elle Rs ‘A 25K ke © 20urks BOLT K kg D13Ki/ke Situation 10 - The pump shown in Figure 34-2(4) draws 20 liters per second off ‘water from reservoir X and lifts #49 resorvotr Y_ The head lost from X tot | Ie three times the velocity head in the 150-mm pipe and from 2 to Yas | {seenty times the velocity head in the 100-9 pipe: 28, Whats the nearest value to the power output af the pump in kilowatts? i a eee Ph Lint ad Natal Water Contest A386 C554 % 484 D. 363 29, What is the nearest value to the pressure at point 1 in kelopaseals?” A. 194 cme oo B 175 D. 27 ‘30, Whats the nearest vale to the pressure at point in kilopascals? ‘A, 2612 C sta ‘ B 2145 D274 Situation 11 ~ A.90 degree triangular channel is to carry a flow rate of 12 m/e: } Using » = 0014, compute the following: 31, Closest value tothe critical depth in meters. A162 C146, 197 D. 088 Site oisial {82 Closest value to the critical velocity in meters per second, A 22 can fe B 252 D.a9 33. Closest value to the critical slope. ‘A. 10085 ¢. oma i B. 0.6064 B, 00031 ia f Situation 12 - A 250-mm diameter jet of water discharging at the rate of 491 i second impinges normally 8 fat rata plate 34. Which of the following is nearest ta the foree exerted by the jeton the metal plate in KN. Ee 1 a. 43 8 39 55s * ai 95. Find the force exerted by the jet on the plate if the plate is moving, at 2 m/ in the same direction as the jet ‘A 335KN © 315kN AG 1B. S75KN D.214kN Ed ee 10 36. If the jet strikes a series of flat plates moving at 4 m/s in the same direction emer bons logit: scale fas the jet ind the work donc-on the plates. 10,500 watts © 12400 watts B. 9.800 watts D, T7awatis $80, 3827 ne Figure 08-0264) Zo 30 Sol fretion angle, 6 (dea) Figure 38 362 May 2004 ‘Figure 85(2) ry 2m 1c 60 {18 (A 28 2B 318 360 25° 7¢ 40 mc mo mB xc SB GR «A A A mB 3D a 90 MD A HA HA MC $8 WD 158 2D 2A SUA _35C 364 May 2004 Lh = 45.35% PL 3651% MG, = 52.52% MC, =PL 523 S TE=PL ~ 4535-3551 D Situation 2(4t0 6) Part 1 R= 350/2= 175 C=2806 175 = 495 sin g= R/C = 175/425 = 20316" Part 2 t= Rain (90-4) E> 175sin (90 -24.316) r= 1595 kPa Part 3: a= C= a25kPa © Situation 3(7t0 9) Pact KH = 3K KYG3 + 8) = 35669) + 710) Ry= 51615 nyday Pant ii L= 18/2000 = 0.008 Past Qa Kaa A shows = S38 (1) = 50me (= 31.615(0005)(50) = 14227 myday {Situation 4 (10t0 12) Part » 73 wk 2 23 22; eule GMC= Se, whores =1 278A} @ Ie e= 1113 2765-1 ‘ om BES a) 8.278 enn Part “ Fe = (82789643) + (103972) + 17.640) fawoatkee Past Be Ge ey A= Hp eh Ss 000e40-10) =0.27 foie PrP ap ‘p= 12cm?» (100 cm/m}= 120,000 Pa ap 120kPa py 17922 + 120 = 29922 kPa npr 075, BO aH = 82 OE lop ee = 0288m AH =23.3em DSitoation 5 (13 to 15) Part |: 2 Lesing _ dosing Lesing ‘Tesina K-0.2073 er a y= pg = 15700981) y= 15,4017 N/m! = 15.402 kN/m? F.= ¥40.3073)(15.402)(57 F,=59.16 KN Parts 11 & Ul: pee Tre Situation 6 (16 to 15) Part a= N+ ¥DyNy + ¥59B N, For local shear failure: ro yD) N+ YoyB N'Y, = § (60) =3333 Dy= 45m; y= 192 KN? Bai2sm From the figure, for § = 10° N= 8; Ny=1.9%) N= 025 qu= 33.3348) + 1924 5)(1.98) + 4(0912)01.25)(0.25) = 37256KPa “5, oma = 437.256 ~ 19.2(4 5) = 350,856 kPa Part 2: qe= EN {1 + 038/L)] + 1D/N', + Mey B NLL 02(8/L)] qe = 33.35(8))1 + 0.3(1.25/6)} + 19.2(45)¢1.94) + Yf19.2)(1-25)(0.25)[1 + 0.2(1.25/6)] 4. = 454.05KPa uate Bem gu=7Dy 454.05 ~ 19.2(45) = 367.65 kPa son. warer :- aes ce pum Key H = 0307(15.402)(2.5) = 11.82 kPa s y Fi=14(11,82\2.5)(1) = 14.775 KN; y= 25+25/3=333 m eee se Fam 11.82(25)(1) = 29.55KN> ye= (25) = 1.25 m a pe= KewH = 0.207(9563)25) = 734 kPa Fy= "(7 34)(25)(1) ~ 9.175 KN, y= 25/9 = 0.893 m. = 9 Aig = 253.46(1.25 6) ju HE = YOB1)25)%= 3066KN; y= 25/3 =0833 m = 1750.95 kPa Total thrust, P= F, + Ft Fat Fy {Sitnation 7 (19021) Total thrust, Fr~ 14.775 + 29.55 + 9.175 + 90.66 = 84.16 kN Foy, Ai = 0.81 x 08)55)(3* 7) = 906.404 kN Loca ee = rain "egy ay @x7K63) BA.167 = 14.755(3.33) + 29:55(1.25) + 9:175 (0.833) + 30.66(0.833) gee om y= G81 « 08\7)(2 « 4) = 499.458 KN PF, + B= 1345.93 ON 2a)? 412 a) = 0.19088 mi. 367 363 May 2004 Yen 74 e= 719088 m Fy-Fiv+ Fey 1345.53 y ~ 906 444(6.28) + 459,488)(7. 19018) yiny-25 epee By squared property of parabola: . wy D Situation 8 (22 t0 24) xx gees ee 02% os* y=25 = 016y ‘ym 29762 m 2g ya (05) 219.81) o= 15,283 rad/nce x 2 = 14594 rpm 29762= Sinsation 9 (25 to 27) Q-2+0 3) Os= 04 20 Taminthy = >) Iq MELSOMIOOT «505m 2 0.0826(0.015)(150)9," 221m = I 543.102 co ST HO #805)? z _ 0.0826(0.015)075}0, _ Bo Seay? 07 128tB ad/sec x 2 =118.642pm In sa 2716 Qs (0.0826{0.015)(300)2 .7 : gn ROSIN 463.25 Fat 2 y=25mr=05m Reet ne 005)! fam eae =249m 25 ~ Ser; 014.007 md /sec x B= 138.76 rpm 03002 2981) InFq (3) MES] os=z1608+46307 Bat Qe 2 543.108 =2716Q)+463.Q2 Qs" 1.307 O: InE9. Qin @.+ 13070: 035 = 25070) (Qa 0.517 m’/s™= 1517 litersee Qs=1.307Q51.7) = 1983 liter/sec HL hn Hat ig = 133.06 + 543.1(015177 + 24.9 HL = 17046 m = 170.46 N-m/N = 17046 J/N_ HL= L6ra)/ig = 1672 kik £2 situation 19 2910 30) Q= 0.02 m/s 02 0% = = 11318:m/s; = 0.0653 m Foye 25 c 0.02 Po? =" =2516 m/s, “= 033m mer 00 MEA as Energy equation between X and ¥ (datum at elevation 0) Ex Hlasa + HA ~ Hla Ev (0+ 0 + 10 ~3(00653) + HA = 20(033) = 0+ 0+ 250 HA = 216.796 m Past 1: Power output of the pump: Power= Q yHA = (0.02)9.81)(246796) Power = 45:2 kW Part 2 Pressure at point 1 Energy equation between X and 1 (datum at elevation 0) E,- HLve= Et d+ 0+10=300659) «0089+ + (20) o 19:739.m of water pi=19.739(0,81) = 193.64 kPa Part 3: Prossure at point 2 Energy equation between 2 and ¥ (datum at elevation 0) Ex- Hla = Ey 039 + FL + (10) -20(038) =0+ 0+ 250 7 Bi = 266.27 m of water ¥ pe= 266270981) = 261211 kPa ae Sak Pa2x al =2(1 966) 3 5.561 m Re A/P= 3864/5561 = 0.6948. vet sv 1 a= a $= 0.00303 (Qosssp0sv2 Iz (24 to 36) Patti 2, ? Fa Bye F1 . 4/781) _ £ gh 9Btes(0.25) eH Part? ra Bh (0,0 Sua cost) va 2m/s a 0.491 oar eve ene yw =2m/s Q=au= 302576) (= 03927 m/s 039271381) 49 0 See (10-2)= sas Fe 372 May Note: Ifthe jest strikes a series of moving vanes F- Suc cosa) z Part 3: s v= am/s u=10-4=6m/s Work done.on series of vanes: Work = = fl ~ cos Oe! 0.491(9.81) SE 100 - cons0-y0) ‘Work = 11.784 KN-my/ = 11,784 Nem/s Work = ct Select the correct answer fox each of the following questions. Solcone answer foreach tem by shading the box corresponding tothe eles 2 on the answor sheet provided. NO FRASURES ALLOWED. Use pencil ne.2 only. {LTIPLE CHOICE fon ~The pene re ULPD 1894 defines an unbalanced bid as « Bid containing, one or more pay {tems that are 30% h w than the unit lovable government climate (AGE) in respect to nifer tare whereas Oller pay densa priced lower un the rat AGE. ACE defined ts oes BBG sss of GH AAE cad tan svoruge of al sescreive wide’ tee Yelp dering he sceage a a ope Bi phy 120% of the AAE or lower than 60% of the AE shall not be considered. Mires are fined spay tons representing at ect 9% ce RAE ‘or those indicated or specified in the instruction to bidders. The responsive bids for a government project with 8 bid items are as follows (all cor figures ined thotands ol poss) Qty ARE Bde Bid Hem! © 10010001000 500 800 fiem2 100 70007001600 19.000 500 500040 3000S 500 0 6000 S00 goo.) 1m 300 = 20002500 a ‘00 1600 2.000 Ts 1.200 300030000 soo 2800 2 -75000 «2400012600 33,000 Which of the following most nest gives the asnount, in thousands of [psos, above which an item becomes a major item? A PIS530 © Pusan B, P2920 D. PIa550 Which of the following give the complete lst of najor item? A Htoms2 and & . Htems 24, and 8 B. tiem D. Htems 2,9; and 8 Which of the following gives the complete list of unbalanced bids? A. Bid B C. Bids.A, Band B. Bids A and D. Bids and c 375 4 A column resting on a base plate fe shown in Figure ST-76, The itmn transmits a moment of 78 kN-m to the foundation aid the aval lose ‘on the column is negligible. ‘The moment is transmitted through two angles Situation 2 - For a gi reinforced concrete pipe can be ‘equipment and labor: A project, it is estimated that 4m of 6l0-mnm-diam stalled in one hour using the folla Rate per hour Welded to the column flange, then to two anchor bolls The weld hag on Veh nen Pboo00 Alona sos of 11D MPa and healt ses ont elt 150 {plate compactor P 12000 Which of the Following: most nearly gives the force that will be wansmited foreman P5000 the anchor bolts, n KloNewtons. 2 skilled laborers “P4000 © 1733 TOunskilled laborers. P3500 D. 62 lowing most nearly gives the moment acting on the welds, meter A 867 607 B 925 D. 742 na L Which of the following mast neal gives the minimum nominal sie of the ‘weld in millimvetres. ‘The pipe cost PS70.00 per meter, For every meterf pipe, the follo vane ig of porand cent P1200 par bag cement et 0.15 cam ofsand @ P20000 per cum Value added tax is 10% of the cost of equipment and labor. Profit, ove A525 C721 contingency, miscellaneous, and insurance is 30% of the cost of equipm B 448 * D654 labor, ad enatersals, : ¢ tion 5 - The shaft shown in Figure ME-12 is made of bronze and steel and 4. Which of the following most nearly gives the cost of labor and equi 4s fixed at both ends. It is subjected to a concentrated torsional at the per meter, in pesos? funtion The stu segment 1 meter Long ‘A. 587 C350 ich of the following most nearly gives the ratio of the Tength of steel to B. 425 D. 235 the length of bronze ich that the allowable stress in each material s 5. Which of the following most nearly gives the cost materiale per meter, simultaneously? pesos? A. 098 © 1284 ‘A915 C538 B 1185 D. 1.354 B 883 D. 624 Wich of the following most meaty gives the reaction at he lft nd the maaimum torque is applied to the shai, in KileNewtons Ad? Cow B. 718 D. 536 ‘Which ofthe following most nearly gives the angle of twist at the interface Of the two materials, in degrees Ale © 436 B22 B38 6 Which of the following most nasty gives the unit price for installing meter, in pesos pee n ctiee C1650 B 1510 D. 1300 m3 A block sides down. plane inclined downward at an 1 of 9 the horizontal. The coefficient of kinetic friction between the i the plane is 0.25, The block is initially at rest and. its bottom is 6 m ki than the ground. ain 6 reef concrete boa has a wih of 250 mom ae fetve 7. Which of the following mast nearly give the acceleration of the block dept ‘mm. ‘The beam is simply supported over a span of 5.50: ane m/s? ree a face sevice ive load of 35 N/m ans service dead load of 13 EN/m ‘A. 278 398 {including the weight of the beam). Steel yield strength f, = 276 MPa and 8. 185 D. 45a Goncrete strength /, = 207 MPa. If compression steel required, it will be laced 75 mm from the extreme compression fiber. 8. Which of the following most nearly give the velocity of the block afler ich of the following, most nearly gives the tela factored uniform load travels a vertical distance of 3 m, in m/s? A412 © 325 fcting on the beam in kN/mv. B. S78 D288 ‘4822 cou 9. Which of the following most nearly give the time it will take before B72 D. sea Iblock hits the ground. in seconds. Which of the following most nearly gives the nominal moment capacity (in ALS C258 Nem) if the beam is singly reinforced with maximum steel reinforcement B. 358 D457 Allowed by 1992 NSCP. ‘A185 C0 B38 B40 376 May 2004 sé Constrocto as 377 18. Which of the following most nearly juired to resist the total factored loa ‘Which of the following most nearly gives the vertical deviation of pomt D Fre Oe wal sen of eon relative to the tangent drawn at A, in millimetzes, 580 © 3360 A072 C273 B 3510 D. 4550 B38 D1 Which of the following, most neatly gives the vertical deflection at Din Situation 7— A projet i nanched upyra at an inital slope of 4 vertical Hhoctoonial hw grourvddloped dawned such tant tor nec pou at point ower at ote oi. The Tight is attained ae horizontal distance of awry from the ogi 19, Which of he following most nearly gives the inital velocity of the pro in meters par second rillimetres, AU? c 005s B 0302 D. 025 ion 10 - A reinforced concrete column with dimensions b = 350 mm and lt 700 mm is reinforced with six 32-mmdiameter reinforcing bars, arranged A 832 c 756 as shown in Figure RC-85, The eccentricity ¢ = 250 mm. Cancrete strength B 1279 D. 3214 {= 21 MPa and stoel yield strength f, = 419 MPa, . Ay Wink iaf ee tolaning ripe memriy Pires the. taal heriaadia ich of the following most neatly gives the reinforcement ratio, én travelled by the projectile in meters. erent. ‘A 257 C856 A 1587 Cima B 3658 Bus B 1559 © D200 21. Which of the following most nearly gives the maxifiim height attained J Which of the following mast nearly gives the fatio of the eccentricity to the from the origin. 3 dianension h © 533 A. 0.095 © 0357 D298 B 0174 D. 0.289 Using the interaction diagram, which of the following most nearly gives the ‘Situation & - Too steel plates, each 950 mum wide and 12 mm thick, are fob aaximum axial load (P,) in KiloNewtons, that the column can support jpined together by welded lap splice, aa shown in Figure ST-82 A. 3870 120 ‘electrode used for the weld has « nominal tensile strength of 550 MPa. B. 4560 D 2810 22. Which of the following most nearly gives the maximum weld size that te used actording tote 1992 NCP. ‘A 105mm © 115mm Figure ST-82 B mm D. 9mm 23. Which of the fellovring most nearly gives the effective net area of the fl ‘ed yang the mosinam weld nize allowed bythe code, 7 ‘A. 3960 mm? 5200 mia 8. 4570 mm Deis0 mms 7 P 24. Which of the following most nearly gives the maximum load that ean] resisted by the weld using the maximum weld size allawed by the code. ‘A. 857 kN C TAN B. 964 JN D. 620kN i 6 ‘Situation 9 - a simply supprted beam is acted upan by a concentrated load: ‘concentrated moment as shown im Figure AN-66. The product Ef a 1,000,000, where £ is the modulus of elasticity in Pascals and [ is moment of inertia i meters tothe fourth power Analyze the beam the Area-Moment method. 25. Which of the following most nearly gives the vertical deviation of ol rele tothe tangent dear at A, in euler, A 197 C386 B. 0.75 D247 INSCP Requirements on Fillet Welds [Allowable shear on elective area of filet welds 0.30 mos the romninal tensile -Aringih of te 378 May Tiaras FES not groav wed sl and on elon ] Thine elcine aro at melts el te ake aor scive length tes etoenr ices 2 Teele tel tle op twin os sh eval ache ekease ln edly ar, 5. ReaD ttre Sadat tal ha beh shri ree om Bs SPN ont ke it i dageamati we cept fore TES iy al Slomatgel a pecan fe rte tt renee all be anaes tesa fet os teal Be SE pu Mino at eddseper tn i “Limitations of Filet Welds "the munnam sizeof et welds sb be is shown in Table. Minimum, 2c B dependent upon the ticker of the two pars abel, exepe that the Weld od nf Cran a chess ofthe nae prt For ot e=con, pric SRSA an rows preenoc oso Tee a Wal ran to tamer part fed ate perl eid by alae ty Ine nevelded comdiion the ditance betvesn the edge the bare mel fe a te el ye thn Ln pov he eld ae ‘Table3 - 6 Minizmum Size of Fillet Welds Material Thickness of Minimum Size ‘Thicker Part Joined ‘of Fillet Weld ‘To6mmincusive Simm ‘Over 6mm to 12m 3mm ‘Over Hem to 201m mms ‘Over 2 mum os 2. ‘The mainwam sice of fillet Welds that ts pormitad! along edges of connected p fate faeries han 6 ra thick nt greater tant hicks of the mat 3) Mazin C'mon or more in hicks no greater than he thickness of i ‘atrial tuba 15 sea. urkess the weld expecially designed on eninge oe bat ou fo obian full tsk ‘he nnn elective leg of Ele wells deigated nthe basis of Bunvbe er es that tnt the orl sa, or ar te ano ho wel Rae a dee A Dyfi elective eng login it ‘Sea sane fend Sonmmetonac ft to rsion ember he leg of sch ‘as ‘be'not es ante perpen feveran fTeueert spacing of tngitadil bt wee ued in en conection Trembers en exceed 2 uns the member is designed on he ttecve net are «Ber cae meters ore eae ing surlae when ee fas tan tat cove chaaous fle weld of the sullen porated sen and 1 on com Ge cater’ The dice lng of any sepwert ol Anereent eaUESE dhufborouiss tun tines te weld se wis minum o¢ 38 ty 5, Riigp ts he miriam lp thal be 3 times the Gichnew of the Sent Fenelon 25 uh. Lap ems ig plas of brs ube see al belt weed along the end fot lsped parts expt whe ici he lapped pt uly eta wo prevent opening off pnt under main nding erence Vol. 2 381 Solutions to May 2004 Examination 380 hay 2008 A 11 (1103) 10s AAE | 0% AAE “LF an) 240000 | r2go000 0000 (Pam, ea 2 Patt Amount for major Pay lem: ‘Total AE = Ps4,600, 20% of AAE= PI2.92000 Pant: Complete list of Major Items: ‘Major items are Item 2 and Item 8 so 80 To a0 8 10 110 120 APIA, sa BAAD, MPa 1 pad a0. Pat3 baa ea Ace | KAGE jacaco | sco) wee stb 9 ‘sorooo | 1sc0000 1575000 4eonce | soono0 | sooo 45000 i | swore | sone sano | Somer ] sore | zsma | sons | rso00| 22000 B | 20mm | tse | same} 135000) tarsco ] soem | asorse | 2sms0 | —soonc0} samoo 1200000 i) sem | ssi | sissoto From the Table above, the ones with gray background are major items ‘Higher than 130% of AGE and the anes in iaies sre items Loner than the “unit AGE, 382 May 2009 Bid B ia balanced bid because allhough some items are lower than tunit AGE, its major items are lower than 130% of AGE. (The bids consistently ow) Bidders A and C have major items higher than 130% of AGE and items lower than AGE, thus itixan Unbalanced Bid. Part 2: when k =3 m,S= 3 zi WP= 0+ 212786) 0-578 ys D Situation 2 (4 to 6) Part 3 art 1: Cost of Labor and Equipment P mretkoeat sats Cost of iabor and equipment per hour: Peete Backhoe = F800.00 12=0+4O7He Plate compactor = P12000 {=294s0e Foreman = 1(60) = PS0.00 Skilled laborer = 240) = P80.00, ‘Unskilled laborer = 10(85) = 350.00. Situation 4 (10 to 12) Force on anchar bolts: 120 + 5 5 From the FBD shown: Total labor and equipment cost = 800-4120 + 50+ 80 +950 ee Total labor and equipment cost ~ PLAD0.00 per hour in aye mG re Labor and equipment cost per meter = 1,400/4 = P350,00 Een Part 2: Cost of materials, Pipe per meter = P570.00 ‘Cement = 0.07(130) = P9.10 ‘Sand = 0.15(200) = P4500 ‘Total material cost = 570.00 + 9:10 + 45.00 Total material cast = P624.10 per meter Design of weld: > ne a PN% Saf Ferman Fe 17333 m | Part 3: Total cost: Value nlded tax = 10°%(350) = P35.00 Insurance ~ 30% (250 + 624.10) = P29223 Total unit price of pipe installation ~ 624.10+350-+ +35 + 29228, Total unit price of pipe installation = P1,301.33 Lert “te teal Moma on weld = 80 =17 Menton Weld = 850650 Renan 8.87 EN The moststrsed ponisored and N= Weos ao" Considering point (Note tt there ae tw lines of weld long A Analyze 1 mm length of weld: ies Si (Fry <0) Dineet load, Ry = £ — 173333 ~ 255.59 ny, REF + f= Wsin 30° ect eed, Ras T= “Sonay Nain Wa + oesw=05w Fue Bay =150mm, 3 a= 278 myst J = TLU3/32.» gt yall 200[200/ 12 + 0+ 08] x2 J = 4,500,000 min? 8.566 5501050) 4,500,000 Fa = 288.89 N/mm Fam Ra= {Egy Pt = (288.890)? +(288.89)" Ry =408.55N/mm Allowable force on fillet weld = 0.707 tL Fy t=? F,= 110 MPa L=1mm 408.55 = 0.7071(1)(110}: t= 525 mm jon § (13 1015) Part 1 According to the problem, the allowable stresses in each material reached simultaneously. 16r s= 1 opr! 16%, Forthe bronze; 60= OU; Ty= 4,970,098 N-mm #05) 16f, Forthe ste: 80 ST. 7, = 1,963,495 Nemun (50)? The rotation atthe junction must be equal: = Tih Tobe mal see 4970,058(0)__ _1,963,495(0) rs) 35,000 3 (60) 83,000 ‘ 4211857 SmtioofL tol, Pat Reaction at the left ene = Tj = 4,970,098 Nm. Reaction at the left end = Tj =4.97 kNem. Part &: Anglo vist at theimefoce= 0,» Zi Note: L= Tm, Angle of twist at the interface = 0, ~ 262495(1000) (G0)* «83,000 Angle of twist at the interface = 0, = 0.03855 radians x 180°/ ‘Angle of twist at theinterface = 0, = 2209 m6(i6i01) Part: Factored unions Wp," 15 kKN/im ee" wu = 36KN/m Wwa= Ta woe + 17 eu = 1.AC15) + 17(56) t= s2akNyin Pat 2: Maximara moment (romina e Poe yy I) for singly reinforced 85/600 _ , 085(2077(0.85)600 = 0.75. =075 SELB 97 08520 7)085)600 Pra” 0750-0757 cons fy arenas 276) Pan 0.0078 00278276) 27 (10.5%) = 207(03707f1 0.590707) crore ] Manas = Rab dm 5895(250)(400) = 29907,919 N- mm Moon = 238381 kNom =03707 Pat 3: ‘Maa: ~ ¢ Mame 0.90(239.81) = 215.83 N-my Required M,: meat ea , = 31082 kN-m, Since the required Ma > Maa the beam must be doubly reinforced Me™ Ma+ Mao Ma = Man 205.83 KN-m ‘Mg = 310.82 21583 = 94.99 kN-am ‘Aas Pu bd = 0.0278(250)(400) = 2780 mm? 386 May 2004 ‘Solve for Aa: Mag 9 Ta (d= a 9499 106 = 0.90 A (276)(400-75) Age 1477 mm? Required terion sti Required tension steel, A, CD situation 7 (19 ta 21) ‘Atany point in the pr =O Aaf (ad), d=73mm ) ad Aah (dod); ‘Maximum size of weld: ‘Miieral 6 man or more in thine, not ruterial minus 13 mm, [eee ees ee Oa ty Gees lta] ess the raid expecially designated om the euwings fo be but ot weber ltanat Neioas Maximum size, f= 12-15 = 105 mm An Ant Aa 2740 +1177 = 3,957 me Part 2 Effective area of fillet weld: ‘The offeetive arch cE fillet welds shall be ken as the eecthvw lengih times the efective throat thichnss, fective length = 950.2 Poo mn Elective sat thickness = 7071 07070105} 7.4255 mm Ective area = 75007285} - S30 a jaca Part &: 7 > Fy (t) ‘Considering the upper weld only: baby see Pa Fede ee eek Cah ee i= 0.7071 = 07 (350) = 2558 mm? ae i se P= 5302596) = 857414 N = 057.4 KN 2 x > E. eee eT) ia 9(25t0.27) Part: ‘At point B: By symmetry 1=802)= 16m 2 -. 981006)? From ig 6) d= t6unS818- Ee Pah: anon fap= 2 (Areayn Z5) ie At point C , 120 yram fa, 1) (000)2/3) 98x 10 Bere wa boise ——— etic +¥49)(1200)0) From a @) Rese tan S218 - Span} cos? SAID? Va} O00}5/)] | lias 0.08583 xe -1.3832c-2~0 195657 N-m° | re= 17:38 m fae SRN om Part 5 x [Atpoint A: fan = DEEN =m? From Eq, (a) 08 (12787 sin 58334)! = 2(981)yx Eco Nm Mia 0.0019667 m ~ 1. yo~ 5333 om ty 9667 m= 1.9667 mm 388 May 200¢ Fate )* = 2/3) ; (Arennn 50) ™ pq apy 2}1200 2) = ce(1000)2/)] foya> 0001139 m “1.133 mm toyam Part $0" yo- tes e = SA yo = 141.9667) = 1.475 mm fo= 1475-1133 0.52 mm. CD situation 10 (28 to 30) ‘Avm 6x 4 (@2?=4625 mm? y= Aa/Ay= 4825 / (350 = 700) = 0.01969 = 1.965%, ‘y= efhe= 250/700 ~ 0.357 = 21 MPa. Pm, MPa 3000 10 8090 Wo 120 Cy PHA eh = AMAA MP. 8 2 30 «0 From the interaction diagram: Seales «709 = 80 Peet stON = 28175 EN Seat Noe fnTERAGTION OAGRAM ‘Vol. 2 L ENGINEER Licensure Examination November 20, 2008, 08:00 a.m. 02:00 pam, VEY! ENG'G. SETA i: Select the correct answer for cach of the following questions. angar fr each tem by shading the box corresponding othe eer ice on the answer sheet 7ED. Use pencil no. 2 only, YO ERASURES ALI (CHOICE 1 —The fourth term of a geometric progression ie.6 and the 10% term is ‘Whats the common fittio of the G.P.? ALS Se spray acing ine DE parle! eB ed Oe lo he aa te BC and 150 ma long. The lofscgmient BCDE w GOST a be ca the aren lot ABC. In square meter. 62368 O57 254 “3 S331 fe the area of lot ADE, m ate the arexo si square meter. TA BA ee 560 B 14475 D. 11,546 fe the value of angle C, in degrees, “A 3T ea ia pB. 42 D, 68 = A swimming pool is shaped from two intersecting circles 9 min ith eco Sayan 4 #5 thearea common to the two circles, in square moter? KAS2 C187 B87 5 45 the total water surface area in square meter? A 0094 3873 at 5th poo inter perimeter of the pool, in meters? AGS 824 ‘BBA D963 Situation 4 — A closed conical versel has a base radius of 2 m and & 6 m ‘When in upright position, the depth of water in the vessel is 3m. 410, What ic the volume of water in cubic meter? 391 fnatfon § — A right circular cylinder of radius + and height his inscribed in fight ciscilarcone ofradiusé mand height12m 2 Deter the radius ofthe cylinder such that i volume sa maximum, AB c C3m BS DB. 32 B. 4m fi D.5m 11, Ifthe vessel is heldsin inverted position, how deep isthe water, in mieten Deterinine the maximum volume ofthe cylinder A335 632 A155 72) © 2531 me B. 578 D. 4.25 B. 821.12 oo D. 201.06 m* 12. What is the weight of water in quintals. Unit weight of water is 9,800N/i ‘Determine the height of the cylinder such that its lateral area is. maximum. A. 2084 Cc 297 A10m Com B 1954 D 2472 B8m Dam tion 9— A right circular cone has a base diameter of 24 cm. The maxigugm Situation 5 - Given the ellipse 16x? +25y? = 400. 18. Compute ils perimeter ‘A 32.65% © 1785 B. 28.448 D, 368% 14. Determine its second eccentricity, ws ‘A073 < 067 B. 082 D.06 15, What is the equation ofits diameter bisecting the chords having equal of 1/5? ‘A. Sx- 16y BL Bx+1éy=0 Situation 6 ~ For the ellipse 1622 + 25y?= 400: 16. What is the slope of the ellipse at point (4, 12/5)? i tis ooh 3 ae b.18/18 17, Wht ada of curate of hlipnat (12/57 A SA? 324 hat he equation of the th alipe a (1275) 1. What the equation ofthe tangent to the elipan a AL Vx +1 100 = 0 oF . 1bx- Iéy- 120 =0 ter ny 100 0 Bt t5y-1a0-0 Situation 7 Given the elipse dx +9 = 144 or Whats the equation ofthe cps with rxpect othe pole whoee coord 44). SOM coyes sore 7ays1e4=0 Cen tse rays es Bde +09 +32c-ly +1840 D. dot + Oye as. 56y- 144 =O 20. What is the equation of the diameter bisecting the chords having, sail slope of -1/3? BR 3r-ay =a © 4r-ay=0 B are dy=0 D. 4r4 3y=0 21, What ig the equation of the diametor bisecting the chords halving 4] slope of 2? Bact oy—0 C or-8y=0 B. &r-9y-0 B. 95+ 8y-0 farea cf parabolic segment that can be ext frorn ths cone is 2078 S Determine the base width of the parabola = ic te 55.0 ; Se D.ab8cm Determine the altitude of the parabola.” A item © 15am B. ism D. 6am Pp Diticrine the alitadeof the cone Amen ©. 16em B item PS tion 10 Given the parabola 3x2 + 40y - 4800 =0, Win: es aror boned by he pobole andthe Xasb? Ae 6200 i B $200 DB. 6400 What & the mosent of inertia, about the X-axis, of the area bounded by the parabola and the X-axis? ol een D. 21/055,000 What is the radius of gyration, about tho X-axis, of the area bounded by parabola arc! ths Xan? bounded by the mies cas § Gd 3756 Bh — An icvestment of P2500 is made at the end of each year wil Mnerest of 25% compounded annually. BS < mn B Determine the cqualpayrentaeres Compoundamoun’ factor alter sar rare 9.632 B 10578 D. 8736 Determine the total amount ofthe investment after B years, A. P23863.12 P2z456.21 B P25,786.95 1D. P21.340.29 ‘Hw long (in yous) will it take for the investment to amount to P59,865002 AW C9 au D2 393 cars traversing a 2-kan 392 November 2004, Situation 12— A d-degree simple curve has an angle of intersction of 51%. Th of the curve it tation 34 +523, ‘4. Determine the length of the long chord, in meters. 15 - The following data were taken on five highsy Common ratio jon3 (7 10 9} 608 (0/2) = 45/9 Oi ay ag F496 x48 6 GB simation 2(610.0 - VaR sin @ y proportion of triangles: By proport nl ra OP sin 120" Ay=84.5230-35.074 9.749 mt Ascoe™ Ase - Aspe 50,974 ~ ¥4{400)(o) sin 50° ‘Common area = 24; = 99496 me 24(350)60 sin 50" i‘. = 977.4 = 200(8d/3) sin 50° of water surface = 2x aR? - 2A; Cae Area of water surface = 2x (3)? ~ 99.498 = 408/44 mt sog7ra=351. 10874 SORE moth Perimeter =2x 288 9, 3O)@60-=1209 | = 387.18 ie ~? a 75998 ‘Ajse = ¥{€00)(87 18) sin 50° ~ 59,319.42? jon (10 to.12) ‘Anant = ¥4150)(145.19) sin 50° = 8,341.65 mt Vee= 3 0245) Sekt Veone= 8 m1? HS ‘Solve for sie bby cosine laws (= AOU + 387.182 - 2(400) 367.18) cos 50” Inprigh postion, b= 39288 m Vane 6 vs 8" fe Solve for angle C by sine law: E bh. 38718 _ 332.88 Gn ainB’ sinc sin5” sin C= 0891; C= 63° Thisvetal poste Veter HS, Te a Weight of water = fy Visinr™ 9800 » 21.991 = 215,511.8N Weight of water = 215511.8 N x (1 kg/981N) x (1.quintal/100kg) Weight of water = 219.686 quitals Situation 5 (13 10 15) 6x2 + 2542 400 Second eccentricity, # = £ Equation of diameter eat + 254" = 400 Didfenentiate with respect 0 x 32+ S0yy = 0 y= slope of chord 1/5. 52x + S0y(1/5) =0 S2r + 10y=0; 1ée+ Sy =0 {2 Situation 6 (16 10.18) jon of ellipse: 16x + 25y2 = 400 Point (4, 12/5) or (4,24) Slope of the curve at any point = dy/ax= y° Tix? + 25y2 = 400 32r + Sy’ = 0. 1ee @ (4, 2.4): eo me Ba Slope = y' = -16/15 Radius of curvature: lox vyeP? px Lith Iv] foe 16/1592)? [-07a07a] p= a2isr Equation of tangent: y-yi= mie a) (a yt) = (4, 12/5) ne “l6/is y- B-Hee- ‘15y =36 = ~16x + 64 Ast + 15y- 100 =0 ‘Anotiver way to solve for the equation of the tangent Text + 2542 400 = ee Torn + 35y y= ‘Isx(4) + 2512/5) = 400 Ge +25y ~100 = 0 art Equation of ellipso with respect to point (-6, 4): (on) % be Gk=4 Goh? , (ik? af oe (a-6" , Ged)? e e Alaa ~ 122, +36) +9492 + 591 # 16) 144 Ant Bny * 148+ 94+ 72 #1 = 14d Sys! 4d + 7ay +1 = 0 Fart 2 e+ oy = 144 8x + Ly’ = 0 Ax + 18y(1/3)=0 &x-6y=0, dx=3y=0 Part 3 dat +99 = Ar + ly’ =0 Ax 4 1841/2) =0; are 9y=0 Situation § (22 to 24) For maximum volume: ven ah 12 12-h= or he 12=3r v= n2-2) V= n(l2rt 23) ar 6A] =0 dar -6r?=0 r=4m i= 12-2(4) dedi ‘Maximum volume: Vines 7h Viens = x(0%(4) Vine: =201.062-m? For maximum Jnteral surface area: Ay= tart 2ny(12 = 27) er tne ante 4a, =O; r=3m h=12-29)=6m |Situation 9 25 to 27) Aun = $.20)(H) = 207.8 frm 15585/h > Eq (1) het x td hw Ls /24 BaP (x 1y Bel (e-2ix +1) dire bm te? A> F200) A= 4 Voie-2 24) Am giao "To maximize A, we get the partial derivative of A with respect tox Base of parabola = 2+ 20,784 em = 185.85/10392= 15m (i= Lx/24] 15 = L(18)/24; L= 20 H- (2-12? - f20? 22? =16em Note; For maximum ares of parabola that can be cut from a right circular cone: b= Dein oo? h=O75L x=078D 2 Situation 10 (26 to 30) ‘Area, A= 30h Area, A= 3 (60/0120) Area,A= 6400sq,units | (40,0) [ace Vol. 2 401 aA = Pedy A = 2 fico Ry dy ee Using the calculator, 4, = 21,064,000 [oie To make the integration much easier for manual integration, lets translate the axis of the parabola. ere aa anaty 1 (40, -120) . $08 ="4(120) sey te he fo an dhs zedy tan 2f-Hy ty ee Joo-u 2a 4=2-= oss 2092/2997) 4-2-2 [s-soovr? sy"? yf a] a Tom 21065143 ar T _ (ener Radius of gyration, = JL = |E065.188 5 ae a ang 77 {= 01025; n= 8 Equal-payment-series compound-amount factor = (F/A i») Equal-payment-series compound-ariount factor = 241)" =1 402 November 2006 ‘Equal-payment-series compound-amount factor = Equal-payment-series compound-amount factor = 8.736 ra agjAcin) Fm 2500(6.736) = P21,880.29 Part 3: +f pean 2,500((1 + 0.025)" = 1] SPC 0.025 1025 = 1.59865 i rilog 1025 = log 159865; m= 19years 2 Situation 12 (3410 56) no= MD Ter" Long chord, L = 2R tan (I/2) = 2286-479) tan (54"/2) Long chore £ = 291.94 m R-286479m kl, 88.09)60) peptrare te» IN Length of curve, Le= 270 m Stationing of the midpoint = 34523 + L,/2-= 34525 + 270/2 Stotioning ofthe dpi = 34458 = 34+ 638 (a situation 1 (571038) Wyo hy = 0.00730 = 60.7 $= Mem OO67SSO+ SO" 432 one (1+0.095)8 1 0025 Blofe=325415=3265m Elo b= 250+ 12+ b= 250+ 1.2+.60,75 =311.95 be= 3265 -311.95= 1455 m ide bc Be 7 ay ~ CO/30)01455) = 38.8 m El ofe = 2504124 gd de= 2504 12+ 452+388 El. ofe =722 m ‘Height of tower at B= El. of ¢=El. of B Height of tower at B = 722-685 = 37m Latitude Departure SSE 26741 15763 SI8°SI'2"E 34917 0.04 124.78 SaPW 582 -66:12 St es Sum = “232 _19:53 Eatituce of Da = 32.32 Departure of DA = 19.53 Length of DA= 4[(32.32)?+(-1953}* = 37.76 m Bearing of Da: tan = 19.53/3292; [= 311438" = 31°8° 36” Bearing = N 31° 8°36" Ww ‘Area of traverse: os Length Lainds _Deperre MD A AB 25.36 222.67 47h Ma F779 BO 15763-14917 50841035 -17.95355 CD 127% 4058262518 11,2972 4 DE 3076 3232 1953 19.53 631.07 Double Arca -20,724 21 ‘Aten = ¥:(20,724.21) = 10,362.1 m* = (I acre / 4,046.87 m2) ‘Aiwa = 2.56 acres 404 November 2004 raneporaion Sea 405. (2) Situation 15 (43 to 45) MIL ENGINEER Licensure Examination ‘Traffic density, k= the number of vehicles per unit length a eee cae ‘Traffic density, k= 5/2= 25 velyan, ‘Space mean speed, = D= length of road n= number of passing vehicles ECHNICAL ENGINEERING SEEA ETION: Select the correct answer for each of the following questions. Konly one answer for each item by shading the box corresponding tthe letter ice on the answer sheet provided. HY NO ERASURES ALLOWED. Use pencil no, 2 only: ECHOICE ath) P(l8+14+16+15 +13) 9152 min Hon 1 The gate AB shown in Piguse 34(01)A is hinged at A and kept closed w= P= 2 = 795 kph ie force Pat fe UR Determine the nearest value to the total force exerted by oil on the gate. A152 EN C12 kN Time mean epeed or tpot speed - The arithrtetc mean of the 5 B. 1687 EN DL ges kN vehicles plasing pois during» given interval of tre Deleratine the nearest value to the location a this total force from A. Re eee 4422m C 4am em (ays) 354m D, 501m " Diternincthe nearest value to the force P needed to keep the gate closed A 1365) cab B I7IN Boat kN Fp scrage ot having density of 86g/ mand dynamic viscosity of 72 ADE Pas Hows in a 7>-mm-diameler pipe 1250 m lang at the rate of 012 a+ Whit isthe nearest value tothe Reynolds Number? A. 1236 ¢ 1070 wt? D. 1860 Time mean speed, ui= 4 (66.67 + 8571.9 75. 80+ 9231) eee cin Bee Time mean speed, w= 79.938 kph B 0.0598 D, 0.0421 What isthe nearest value to the total head lost in meters? A064 C0515 B 0421 D031 3 The low of water fram reservoir Ain Figure 562) s6001/s Be Tissier sure ceeaton of servoirB is neuest os A VA8m C1965 m1 B 1625 m D.1813m The flow in line 2 in liters per second is nearest to; A 230 © 250 Lea D. 260 “The flow in line 3 in iters per second is nearest to: A330 C340 % 350 D320 406 November 2004, Situation 4~ A lead welghing 110 KN/or is tied to the bottom of a LEN eyl 10, How tum EN of ead het makes the einer Flot with water uo B25 C22 topof the cylinder? a ‘Wat prepare increase in the sls espcint when he tank led with Determine the required depthol footing in meters : ies ‘p'20 TA IS2SKN B. 1253 kN D. 13.258N : q aes cee 11, How many KI of Jeed that makes the cylinder float with 05 m = above the ae {FL What the expected settlement ofthe clay layer i A, 92K ©. 835N 4 182 C12 5. 75kN B97 kN ay B. 528 D399 12, If lead is placed inside the cylinder, how many KN of lead will make Cylinder float with water just flush on top of the cylinder? ? cea i ne es tion § ~ A retaining wall 7 m high supports a cohesionless soil having shy : nv density of 1600 kg/m’, angle of shearing resistance is 33° and Voi ratio of B. TLIOKN ey (068, Tho surface of the soi is horizontal and level wv tse top of the wall Neglect wall fiction and use Kanhine's formula for active pressure of a eohensionless soul 5 - A steel pipe 275 mm in diameter and 4_mm thick i used an hegh theses ME Determine the nearest value to the total carth thrust on the wall in KN per 12, Glen forthe tensile sess in dhe pipe when theres head egal soeter ire solo ary = en ae Aiba c. 157 B. 1255 Daa, ey Dees ee i eaare ica SO eee he ee | Detormine the nearest value tothe thrust onthe wall KN per Lnel meter ype thickness in ram i the pressure head is 500 m of water, Assume Homing to snadequate drainage, its waterlogged to level 3 m below the theeticency ofthe pipepint 60% ans ean eS Di 5ai Bertin the rene vac to height eave the base ofthe wal where ¥ See ne Bc part & Geter the acta wall ee ae the thras acs during the waterlogged condition 158 m © 197m A. 1062 ie B 1m D. 254m Boa D, 904 Situation 6 - A consolidated-dreined triaxial test was conduct is set-upas shown in SM-23, A cylindrical mold 4° in diameter ‘consolidated clay. The results are as follows: 03 ~ 138 KPa, (62\(= 256 Re rt coat Wane cocticient of permeable, Hii 16, Whatis the nearest value to the angle of friction in degrees? eer ani. A a7 © 2889 B 2214 D 2881 17, What is the nearest value to the angle @ that the failure plane makes the major principal plane in degrees A 6232 57 B. 59.45 D. 8212 0 eee ee ee ee Th est setup is permeameter of constant head. Water is placed inthe told and maintained ats level h = 14 ft above the level of the outlet It ee Bee 7 be considered that the system consists of a fictitious sail of thickness H Situation 7-The tank shown in Figure 36(12)-2has an insite diam | =H + Hand coefficient of permeabity f- ‘O82. THE coated Weight of enpyy acanet te corerets oeege Thetlowsing general equations may be we EN, It is required to excavate an amount of soil such that it will comy Koeg™ at (Hs en) + (Ha/ Ba) + 2. + (Haken) to the dead weight of the tank and concrete. Bog (L/E)( hus Hi + has Ha * sa he Hie A second coansal mold is placed on top. of the first mold whose inside diameter is d= 15" and whose height is Hy = 030 ft. Its thickness is hepligible. The inside of this second mold is filled sith the same sit, but the annular ring outside the small tube and outer tube is filled with sand [whose coefficient of permeability iska= 2.7» 10° ¢/min 408 November ‘Geotectntea! meemeering 409 25, What fe the total flow of water In(@/ min? A. 25149 « 104 C1699 «104 Calculate the net bearing capacity for a strip footing of width 1.25 m at a depth of 3m, ‘The Terzaghts ultimate beating capaci oquation is given B 005 « Wh D.1785 «104 if ee Seen e 26 What & the equivalent coefficient of permeability. kin ft/min? gene Net yyy Mya A 6952 «+ C7325 "104 A us2kra © 4875 kPa B 60% «104 = D. 8253 « 104 B h7 kPa D, 3288 kPa 27, Whats the total amountof water that percolated aftr 55 minutes? Considering shear feilure only, calculate th safe load on a footing 6 m long A. Bbc © 1580 by 125m wide, using a load factor of 25. Given: B bcc D, aé2ee geome NAL * 03(8/1)]-+ yD)N,* 4y BN, -0.2(8/0)) Situation 10 ~ A borehole log profile in a construction project ix shown in Pl 7 tune f FS* yDy M56. The proposed construction will impart a net stress of 15 A. 2258.1 KN 118655 Use unit weight of water equal to 931 kN/mt |B 1a537 kN D. 3/2004 5N as 28. Determine the nearest value to the buoyant unit weight of the clay’ Which of the following property of soil is necessary inthe design of footing? KN/ am A Permeability = Alterberg limits A, 958 © 857 > B. Plasticity inclex + D. Compress B. 1021 D8 : 29. Determine the nearest value to the effective vertical stress, in kPa, at mid-height of the clay layer. AWE 17 B 183 D. 164 30. What is the average settlement of the normally consolidated clay layer centimeters. Compression index C, = 0.009(LL~ 10). A. 364 C212 B 209 D. 463 Situation 11 ~ A dense silt layer has the following, propertics: void ratia = effective diameter dis = 10 ym. capillary constant C= 0.25 cm?. Free ‘water level is 80 m below the ground suriace. 31. Find the height of capillary rise the silt. Capillary rise is given as h= C/) did ‘A, 735m © 625m B. 475m 1B, 555m 82 ind the vertical effective stress in kPa ot Sm depth, Assume unit weight solids = 265 kN/an? and that the soil above the capillary action 1ise {ground surface is partially saturated at 50%, ATA © t64 B. 136 B18 33, Find the vertical effective stress at 10 m depth Assume unit Weight solids = 255 KN/m? and that the soil above the capillary action rise ‘ground surface is partially saturated at 50% A155 Cot B 182 D211 Situation 12- A soil has the following properties: Unit weight, Cohesion, ‘Angle of friction = 10° ‘Asmume local shear failure and wie Figure 28 to get the bearing factors wo Noes 20 ‘Soil friction angle, + (deg) Figure 38 asm Tank ne 20 a8 Ma 450 se a8 BA 1c BA Concrete Fg Figure 36(42)-2 A Ba HC Di we Ro Be 2D HA MC BA HD Be we 38 wD p= -10.erra ? rr Pa™ (281 « 08)(4) 18:46 fre= 12.992 KPa Be pig A= 12.902 (2% 6) Fe 185.184 kN fe=rha) 1 ral a3t4 EM, =o) 155.184 = (9.81 « 08) F (2* 6) ¥ =1.6078m 2 " Geoaem em y=3+192= 492m P= Fly) (6P= 155 184(482); P = 124.68 kN R= 0120.55) 7x10 Re=1,070<2000 (Laminar flow) f- = 010598 64 1070 414 Be Ay jy_ 0.0598(1,250) 012% = =0731'm D % 0075-2881) (y= © Situation 3 Q1-06 m/s 820m o26(0.018) 2000.6)" 06 hin + hy = 200-178, Mq= 2-826 =1374m = 0.0826(0.02)(1500)(05)? hn= = 826m hn a #1374; Q.= 03199 m'/s= 51996 [Qi-Q:+ OQ” 6= a+ 03199 (0.2801 m/s = 26031 Ys = £1.0826(0.015) (1,100) (0.2801)? oat Ele Ela = hin = ha Ely ~ 200 -8.26- 10-44 =1813m ha = 1044 m, i Sitiuation Fart BF 1eVo BF, = 9.81(4 x(13)(2)]= 15.41 KN BF=9miVi Wands = 10V, EFv=0 We Wi = BFe+ BF 44+ 10V,=15.41 + 981V; Vi= 0.1139 mv ‘Wi = 110(0.1139) = 1259N 4 415 = 9811 4 n(13(1.5)] = 11.564 BF,- 981¥, We= nve~ 110¥; 0) Wet Wes B+ BR 44 110V,~ 11561 981¥1 Vi 0.0755 am Wi= T0(00755) = 83kN re3: : BE = Vo e BR,= 981[+ 2(13)(2)] «1541 kN Wma fro TWh w= are pe Se ere, atm HS tog Pe 4 Se Tre, "8p, MTs jon 8 220 24 Part iss! Tesing To ina3° K=a208 Fe iRoy it ¥= pg 16000 81) situation? alae N/m = 15.696 1N/t sl: = 0: e Weight of sol removed ~ Weight of tank and concrete ee ort eky Yt Vea 3,200 Parts 1 (185) * $0} Dy= 3.200, Dy= 22m : ne So Pan? = Weight of oll = ya Vos = (9.81 x 0.82) 5 (1096) = 3790.74 KN 13606 Tg OSI G= 2609 ; A 2688 —1 ny : Pressure increase « AY at =sn.2662kPa tem SEES (1) = 9.857 LN/o or, Pressure increase , hat = (9.1 * 0.82)(6)~ 482652 KPa From th figure below. = Ky H = 0.2649(15.696)(3) = 13.882 kPa Pan = (13.882)@)(1) = 2082; 4+3/3-5m o™ initial effective stress at midheight of clay layer R= 13.992(0(1) = 55528 jada) = 2m po 172014) + 1850.5) = 68:83 kPa po Ken ~0.2948(6857)(4)= 11.625 KPa ‘hy = Pressure increase due t ol = 48.2662 kPa Fe= i 11623\a)0)= 2346 AN, w4/o~123m Note: The weight of the tank was no longer included Because Fay. nesg=aiin, faassen it was compensated by the weight af soil excavated. Me Total thrust, F= Fy + Ft Fs+ Fy ocR= al 2.2; pe= 195.426 kPa Total thrust, Fy = 20.82 + 55.528 + 23,2464 78.48 = 178.074 kN Note: p. = preconsolidation stress en Pym pet Spm BB.83 + 482652 = 137.0952 kPa es T7RAOMAF ~ 70.8245) + 555282) + 25.246(1.53) +78.48(1.33) 418 November 2004 £2 situation 9 @5 to.27) k=3.61064t/min i=h/t=14/05=28 = F@= $05/17=0m1227F0 (Q1=3.6% 104 (25) (0.01227) = 1.2868 x 109 f0/min Path b Qs hey fA Ee as * Sa 7751104 nin 27x10 = 3610 imh/L= 14/05 —28 A= $= n= SCG = (38 Qe (75 x-10-9(2.8)(0.075) = 1.575 «104 F0/ min = 0, Oy 1.2368 » 1054 1575 x 104 = 166987 + 10° ft/min Part Qania i= h/L~14/05 -28 = FD~ FC 4 - 0.08727 08 = 1.5987 « 104 f6/min 1.6987 x 10° = hy(28)(0.08727) y= 69517 10* fymin Part: V= Q x 1 = 1.6587 x 10+ fe8/min x 55 rin, = 0100934285 Fx (100 cmn/3.28 8) V= 264.76 cm? 10 (28 to 30) Pant = 1026 267= 981 xG;G=272 {emc: MC=w= 040 S=1 (below the water table) 272(0.40) = (Ife; e= 1.088 Ine Pan pen s0i(Zh)+ 104006 46) +17.644.0) Aso neice pom 176.07 kPa -< 5250004 | = 20.35 ey | am Past 11 For Normally consolidated clay: AH = HS tog EL Com 0009{LL - 10) = 0.009445 ~10) C0315 yo effective vertical sess at midcheight of clay layer pea t7A07 KPa 17 Fo? BP, ap ISN/cméx(00cm/ms ‘j= 150,000 N/m = 150, jy TAO7+ 150 = 324.07 KPa oss |, 32407 AH = (76) 999 tog 407 = 9.308 m AH Cees Saag? AH = 309m 2 Situation 11 ($1 to 33) Solving for yarn e First we solve G: ote: 265~981~G6-27 Below the water table the soll is saturate and the water is. a eae raiter, ‘The water ‘is positive (compression) and Tia 1 tem *9.81~ 2.729 kN/m with depth below the water tal ee 1404 Bam i] = SE OWA) gg) = 2035 N/m Tasos bie wai able: i Yachtghe or expla ca Tse * ae fm capillary zone and the water is called capillary water. The ¥ Pressure is negative (less than atmospheric) in the capillary and ineseases negatively with height above the water table Nertical effective stress at A Total stress at A r= 21.720(3.25) + 2033{1.75) = 105:197 kPa Capillary rise: n= Pore water stress at 4: Cd Pe -BBI(S) == 29.49 kPa senna Ier=pet pe] 105.197 pe + (29.43 dip = 10m = 10 x 104 m= 10> 104 em eee ste ex 0d bo 03 ners ca 6m Vertical effective stress at 8: pe~ (21.728 ~ 981)(2) + 21:729(6.25) + 20.3(1.75) pe= 195.22 kPa 12 (34 to 36) Part 1: He CN. + DN, 4% 1B oaciox 10") or local shear fature gum EN + 1DjN',+ YeyBN, b= 2c=260)=2053 Dye 4.5 mj y= 19.2 KN/m* B=1.25m From the figure, for $= 10" Numa; NY =1.94: N= 025 4 = 33.3348) + 19.2631. 94) + 1619.2) (1.25)(0.28) qe = 381.384 kPa janet = gu ~ Dy ~3B1.364'~ 19.263) unet * 323-784 kPa Part 2 = EN‘ {1 + 0.3(8/1)) + yD,Ny +7 BNL -0.2(8/L)) = 33.33(8){1 + 0.3(1.25/6)] + 19.2G)(1.94) + ¥a(192)C1-25)O.25)f1 + 0.2(1.25/6)] jo = 398.174 kPa Gone 4u-¥Py = 398,174 - 19.23} = 340.574 kPa = SE + 1920)= 199.85 kPa Safe load, P= q, x A= 193.83(1.25 x 6) = 1453.72 KN Part 3: Among the choices, compressibility of soil is necessary i design of footing, Big ru 15 Frese fh Cl 265= 9.81" G:G= 2701 = 2A a1 7291s 27005 + OSNO4) oa Tot Ys ne 2033 KN/m? tress at A Total stress at A: pr 21.729(3.25) + 2035(1.75) = 106.197 kPa, Pore water stress at A Pe=-981(3)=-29.43 kPa [rr=pe* re] 106.197 = pr + (20.43) r= 135.63 kPa Vertical effective stress at = (21.720 - 981)(2) + 217; + be eee ee Ot Ze) +5075) 12 G4 1036) Part: = cN, + 1D/N, +44 YBN, 422 November 2004 For local shear failure: qe EN'e+ yDyN +4 7BN,, z= he= 3 60)=3533 Dj 4.5m; y=19.2kN/m! B=125m From the figure, for ¢= 10" NimB, N= 1.94 qo= 22.3308) + 192G)(1.9)+ ‘381.384 kPa Gana gu 7Dy™ 381.384 - 19.263) ‘oo = 523.784 kPa ‘Seat Now RING AND CONSTRUCTION SETA JCTION: Select the tvireet answer for each of the following questions ne foreach item by shading the box corresponding tothe letter SP oeeer hewcwemectperuet SVNO ERASURES ALLOWED, Use pencil no. 2only. 025 ¥a(19.2)(1.25)(0.25) shown in Figipe ME-16 is hinged at the Teft end and ted ble cable atthe right ena DEES ce the folowing meee needy groea the teraon the cable in Newtons: Past? a a= ENGL + 03(8/1)] + 1D.N + Yo BNL “0.208/0)) 4. 33.33(6)[0 + 03¢1.25/6)] + 19.2)(1 98) CS & 38 + 95(19-2)(1.25)(0.25)f1 + 0.2(1.25/6)] 4 hee : {Which ofthe flowing most nearly gives the reaction at in Newtons: nomen A176 icmmmme Stren B 1864 D. 1957 ura = x ~7Dy = 398.174 - 19.208) = 340.574 KPa 28574 1929) =193 47a Safe load, P = 4, * A = 193.83(1.25 * 6) = 1453.72 kN Which of the following most nearly gives the angle thatthe reaction at A aks with the horizontal (positive cusrterclockwee) A lise C365" B 1973" Daa Part 3 ‘Among the choices, compressi lesign of footing, flan 2 bear 26 m lng is simply supported by a roller support atthe ket fd and hinge support at the night end. ‘The beam carries a concentrated Toad of 500 N, 5 m from the left end, a dockwise couple of 800 N-m at the Hidspen, and a uniformly distributed load of 50 N/m ever 10 m from the nghtend , ity of soil is necessary I the following rics nearly gives the reaction at the Kft end, in Newtons A 3308 ©. 3546 B 4692 D. 6547 Wisch of the fotlowing most nearly gives the maximum shear in Newters. ‘4. 3565 4602 Beata D. 5308 Which of the (ollowing most nearly gives the maximum moment in Newton ecter A 2100 : 2900 i 2508 B26 on 5~ The rigid beam shown in Figure AN-28 is suepende! by thee cables The beam is hocizontal prior to being connected tthe cables and is to Femain horizontal ater bing attached tothe cables. The bean weighs 1764 “KiloNewtons TWhich of the following mast neaily gives the ratio ofthe Yorce i the B cable to the fone in the tel cables, Which of the following mast nearly gives the deflection st point Bduc to the em oad, when the ler support vem malin, 4. 047% © 0389 A 4125 Bist 3.257 : B. 7225 D. 5625 8. Which of the fellgwing most nearly gives the stress in the steel cable Which of the following, mast nearly gives the deflection at point B due to a1 MegaPaseals lunit load ot 8 when the roller support at is removed, in millimeters: 13 168 ‘A015 C06 B. 358 D.%8 Bas D.03 9. Which of the following most nearly gives the elongation of the steel Which of the following most nearly gives the vertical reaction at B in in mullimetors KiloNewte A 0.285 ©. ose A 1235, C10 0936 D. ass B 1575 D. 1375 Situation 4 A flor is made up of « 200-mm thick concrete slab’ and thick floor finish both of which has a unut weight of 24 KN/caum. The aise caries acelling whose weight is 820 att «lve load af 2400 Pa. floor is supported ‘by simply supported’ steel beares with s'open of il spaced at Ein on centers with Compression flange rigidly attached cence Moe, The sels AS6 with yd suengh (f,) = 208 Ma allowable shear stress specified inthe 1992 National Structural Code of Philippines is O40F, The allowable deflection £360. 1 10. Which of the following most neatly gives tie toial presouce on the a {6 A.50 mm x 100 mm tniber is used asa purin. The trusses ae spaced 32 3 nd he op cor orp the Rezo The rn tan be assumed simply supportid atts ends. Spacing of putts along the top chord e430 on cea. The purln is teat a vera ive ota Paya vertical cell load of 140Pa and a vertical dead load of 120 N/en including ts own weight. The timber used is 80% stress grade Apiiong The propertics of Philippine woods are given in Table 43. Which ofthe following most nearly gives the uniform vertical lod on the ‘pucknin Newtons per meter: Pascale ‘A, 5212 < 457 ‘A 10520 C9858 B 468 D, 3956 B a563 D, 9220 E Which of the following most neasly gives the maximum bending stress in 11, Considering the weight of the beam, which of the following sections i Ui Pascal mest economic (lightest) section for the given load, assuming that dfle 74 cis gaverns the des eB. 13 B16 ‘A, W18 x11, ur 4555.14 N/m, d= 5669 :mm fe ~ 38.6 mm, Which of the following most nearly gives the maximum deflection in y= 289657 ~ 10 m4 eters, B. WB x 234, w= 342507 N/m, d= 5349 mim, te=29.5 mm, ‘A653 c a9 y= 2.03958 x 106 mum“ 356 D589 © W852 130, 27= 1,901.7 /m, d= 489 mm, fy = 17 mm, y= 1,023.93 x 108 mm D. WB x 158, ar= 2304.96 N/m, d= 500.9 mm, fy 206 mm, Ty = 1.27862 » 106 mm an 7 An amount of one million pesos is needed after one yeur, Given an sg. leet rate OF 15% compounded monthly Which ofthe following ot realy gives the single amount tobe deposited 12 Considering the weight of the beam, which of the following sections it “how in order to raise the amount: ‘most economic (lightest) section for the given load, assuming that (A, 836,569.67 ©. P361,508 60 stress governs the design. DB. P786352.25 D. Pas23366.42 ‘A, W10 x26, w= 378.87 N/m, d= 2624 mm, l. = 6.6 mm, 1, = 50.94% 10° mend B W10% 112, w= 1637.88 N/m, d= 288.5 mm, f= 192mm, T= 298 * 106 mun" 4 W103 60, 20 876.2IN/m, d= 2506 nam, ty = 10.7 mum, D. | Which ofthe following most nearly gives the required monthly deposi to Be isle at nd cf eh mom (eewelve mocha in onder Yo teas te fence B Pama 5, Proeatae FPSB 36 DW ich ofthe following most nearly gives the required monthly deposi to He ind athe ong ofeach monn fr oweve mons nore 0 se Bae I Fras 26 Preseoas B Pes 31 D, F7678.38 [y= 141.94 » 108 mm W10 x49, w=7I685N/m, d= 2535 mm, to~8.6 mm. J 113.215 © 105 mm" Situation 5 - A two-span continuous beam is shawn in Figure AN-16, Use E=i (GPa ane T= 600% 108 sie, 426 November 2004 nna Situation B = Section S122 of the National Structural Code of the Phill states the following provision for development of bars in tension: 5122.1 Development lengh lu, in_ millimeters, for deformed. bor deformed vires inension shall he computed as the product af the evelopment lengtl lp ef See. 5.1222 and the applicable modi factors of Sec. 5.1823 thyough Sec. 5.1225, but ly shall not be less than P Which of the tres motors is mont eronomally advantageous 2 Moore tee eo B. None of these D. Motor A Table 4-1 Working Sires or Veuly Sens Gade Unscxsned Suter! Tinker of Figg Woods 5:12:22 Basic development length le shall be: Si im bar B-enulicrand deformed wines .-".007A 36 mm. e oe BAP =~ A wall footing 3 m wide and 550 mm thick seeps cat 600-eim. thi Peat haber tee ate che ere ee Talent pets Neen ee cola a oe mm avconing to 1992 NSCP ‘A 1880 c m0 Saat Se B, 860 B. 670 = ee] es 25, Which of the following most nearly gives the actual development le ao se | we | ue | ae mnllimetres. fe wa | 50 | su | oz A. 1350 ©. 1200 Sen aa | in | Ge | is B 125 D175 ee ae | a9 | op | 2A, UM the required area of of flexural reinforcement is per meter length of 8 a aw | ia | nm | te 2840 square mnlimetres, which of the following most nearly gives — um | rs | up | is ‘equine spcingof bars inline: is am | ose | um | ae 1859 1667 tous no | me | tte | te B 135 ©. 1728 Mees | cs | ie | one | an Tain uation 9 - A water department is considering the purchase of «md ‘Operate a pump for 4D years. Three motors are ta be considered as Motor | Fustcost ‘taneoee ~__|_P5s0,000 0,00, B | P770,000 Een | Frawo.000 | Assume interest rate of 9%. 25. Which of the following most nearly gives the present worth of Motor Ar A, P10,683,973 C, Fib350,739 B, P9856.682 D, PEL b13,289 26. Which of the following most nearly gives the present worth of Motor A. PLLSI3.269 C. P10350,739 1B PI856,652 B. PI0g93.973 Pprey ea Hy F i i 428 Novemlber 2004 Solutions to November 2004 Examination ibe Figure AN-28 ale ‘eg: a . [EMa=0) Tsin30°(10)=26042-6) 7-286 mee Ro nt aoe Sas fe ano Gea EMe=0] Rav) 2603 4) Ray= 96 PF O)Rey§ 260 = 288 cos 20° ‘ Ray = 149415 Ne Bem Va Ray?) Rem faa rem Ry= 1776N fan8= Ry /Ral tan 0 = 96/149.415 O=an7 ) 25R« + 800 = S00(21) + 50(10)(5) Ry = 409.23 N 26Re = 500(5) + 800 + 50(10)(21) Ry=530,77N From the figure below: ‘Maximum shear = 530.77. Maximum moment = 2900 N-m Allowable shear stress = 0405, F,= 040(248) = Pressure om the slab agen Situation 3 (7109) ; 27.4 P= 1764KN > Eq, (1) er 8 —P,(1000) ___F,(1600) (600(200,000) — 900(83,000) hse >is attire ono Mose, 3920) ‘Substitute Py ~ 0.389 P, to Eq, (1): 27, + 0.389 P,= 1764 P= 73.838 kN ‘Stress in steel cab ‘a 73838(1,000) (60070000) §=0615 mm ‘Conerets = x thickness ~24(0.25)= 6kPa = 6,000Pa Live Load =2400Pa Celling =820Ps Total =9,220 Pa > Part 1 Uniform toad on the beam, w= p x3 Uniform load on the beam, w =9,220 (5) Uniform load on the beam, w= 46gL00N/m = 46.1 kNi/en Part2: Allowable deflection = 10,000/360 = 27:78 mm Ty first the lightest section: Choice Cte = 46100 +1,9017 = 4,001.7 N/m Lt _ _5(48,001.7)(10)* 1000)" SE4ET ~ 35(200,000)(,023 93108) = 30:52 mm > 2778 (NOT ADEQUATE) Choice: 4100+ 250195 4440496 N/m _S(s840426,00)41000°_ 384(200,000)(1, 273.67 10°) $= 24.74 mm < 27.78 (ADEQUATE) Part: Allowable shar stress, Fy = 0.4F, = 0.40(248) = 99.2 ‘Choive A: i= 46,100 + 378.87 = 46,478.87 N/m, wh _ 46,47887(10) Viney = R= SE = SAVES) 2 2 = 22,3985 = 23239435 N 16,100 + 716.85 ~ 46,516.85 N/m wh _ 46816,85(10) z Van= R= = 234,00425 pee Te 53586) fo 10737 MPa > 99.2 (NOT ADEQUATE) ‘Choice: “w= 46,100+ 8752~ 46,975.2N/m pe BE = BOTS pen fino Ba vd 259.6(107) ‘fo~ 84.56 MPa < 99.2 (ADEQUATE) Vex Gi Situations (13 to 15) Part |; Deflection at B when the supportis removed: 10h 1 E=200,000 MPa 1 600106 mm! 1 2 Feeeethegi cm me) 3 i - AS N=m? 8= Epaanan)-40)606)]= = 435(1000)* 5 "200,000(600% 10") = 4125 mm 434 N= 619068 Nem 64, __ (619.058) (1000) ru ‘50(100) wt _ 253.4165(3.5)" Mg Er at AUG +0.0125)? =1] 0.0125 1,000,000 = AMT 75831 7.429 MPa Part3: Annuity due dang ui adage Ave a = TB = 357419N-m M, Fo oS - aad 8578 MPa Is mes* fic fiy™ 7429-4 8578 = 16.007 MPa ae eos eS Part; Deflection: Situation 8 (22 10-24) ai es SaryL* ‘BAF, 5(404.289)(3.5)*(1000)° ae 00) a4(7310) i= 25.94 mn an et sei, 4 ~ Sesanssyay 009? L = = ron tam 3047310) 82 tthe 6, = 59.896. mm 7 Sr ar Part 1: Cae <=<€ aus ((6,)? +6," ~ (69.806) 75.98) Bee ea ance bea 27 mm : : .02{5 (25)? Ja45) ae, Base deeopont ing a= SECS 40 FePLo0000n00 i= 015/12 =0.0125 l= 7393 mm paisheois nei Putt Part 2: at Arian development langth= 1125 mm po F_ -_1000p00_ Cees Gen" Geoorsy= Part 3 P= P361,5086 fina) (25)? N= 2.840, N= 5.7856 Part 2: (Ordinar wait) SHH esse rat pe Mso"=1) ! —— 436 November 2006 Situation 9 23 toa7) Amulet C= FO1+ Om + BERS For this problem, RC= FC For Motor A (life= 10years): (Aca = 50}00(005 + So p00» (2.000 82.503(005) (+009) ACa = P1,070,270.89 Present worth for 0byear peried 1,070, 270,991 +000)" 1] (1+ 0.089" (005) PWiy=PInsi326049 > Ratt Pl ACA(P/A. i, 40) = For Motor B (lite = 20 years): ‘ACa = 770, 0004008} + 880,000 + ACs = P962.20067 Present worth for 40-year period: 962,200671(1.+ 0.08)" —1] (+009) (0.09) PW,=P10,5507922 > Part2 (70,000~110,000(0.09) (140.09) —1 PW. ACAP/A, i, 40) = For Motor € life 40 years) omia+y"—1) sv ary | ai _, 885/000{(1-4 0.09)%"— 1] 350,000 0 + 00090 +009)" Po P10,688,973.42 PWe= FC+ PA ‘Thus, the most economical is Motor B. > Part 3 437 ‘Seat Now IL ENGINEER Licensure Examination y.May 11, 2005, SETA JECTION: Select the“correct answer for cach of the following questions. one aie foreach tem by shading the box corresponding to the I eeeeeer tas sansiver sheet provided,

You might also like